SlideShare una empresa de Scribd logo
1 de 48
Descargar para leer sin conexión
From 
Blogs 
to 
Boards: 
AAHPM 
2012 
Pre-­‐Course 
from 
the 
contributors 
of 
Pallimed 
& 
Geripal 
From Blogs to Boards: Answer Key 
Editors: 
• Suzana Makowski 
• Drew Rosielle 
• Paul Tatum 
• Eric Widera 
• Christian Sinclair 
Page 1 of 48
From 
Blogs 
to 
Boards: 
AAHPM 
2012 
Pre-­‐Course 
from 
the 
contributors 
of 
Pallimed 
& 
Geripal 
From Blogs to Boards 
Goals of this session: 
§ Prepare for the boards with an audience response quizz hosted by Pallimed & Geripal bloggers 
§ Review recent, clinically relevant literature on Palliative Care 
§ Discover three methods to obtain quality up-to-date information on palliative care 
Board Review Content: 
Page 2 of 48 
Board Content 
8% 
7% 
6% 
45% 
11% 
9% 
5% 
9% 
Approach to Care 
Psychosocial & Spiritual Concerns 
Impending Death 
Grief & Bereavement 
Medical Management 
Communication & Teamwork 
Ethical & Legal Decision-Making 
Prognostication
From 
Blogs 
to 
Boards: 
AAHPM 
2012 
Pre-­‐Course 
from 
the 
contributors 
of 
Pallimed 
& 
Geripal 
Notes Question HPM1 
Page 3 of 48 
Ms. V is a 68 year old with metastatic non-small cell lung cancer, congestive heart failure, and 
mild renal insufficiency residing in an inpatient palliative care unit for management of bone 
pain. Her medications include morphine IR, fentanyl transdermal patch, furosemide, senna, 
and Fleet enema’s prn. Ms. V did not have a bowel movement in 4 days. Basic labs were 
ordered for the next morning as well as a two of her prn enemas, although they failed to result 
in a bowel movement. The labs the next day reveal a serum sodium of 124, potassium of 3.0, 
creatinine of 1.4 (baseline of 1), low calcium of 6.5, and a very elevated phosphate of 17 mg/dl. 
What is the most likely cause of her electrolyte abnormalities? 
a) A medication adverse event 
b) Tumor lysis syndrome 
c) Bowel Impaction 
d) Osteolytic metastases 
Discussion: Correct answer is (a) 
a) Sodium phosphate preparations should never be given to patients with renal 
insufficiency, heart failure, cirrhosis, or elderly frail individuals due to significant risks 
of adverse effect. Both oral and rectal sodium phosphate preparations can cause 
significant fluid shifts within the colon resulting in intravascular volume depletion. 
Furthermore, these preparations can cause electrolyte disturbances including 
significant hyperphosphatemia, hypocalcemia, and hypokalemia. A significant 
clinically important rise in serum phosphate can even be seen in elderly patients with 
normal renal function. (J Gastroenterol Hepatol. 2004;19(1):68). Lastly, phosphate 
nephropathy may occur due to the transient and potentially severe increase in serum 
phosphate combined with volume depletion from the fluid shifts. 
b) Tumor lysis may indeed cause hyperphosphatemia and hypocalcemia, although it is 
generally seen in with cytotoxic therapy in patients with a large tumor burden with 
rapid cell turnover (ie. Non-Hodgkins Lymphoma or certain leukemias). It is also 
associated with hyperkalemia. 
c) Bowel impaction alone should not cause these electrolyte disturbances 
d) Osteolytic metastases generally cause hypercalcemia. 
References: 
• http://www.geripal.org/2012/02/dangers-of-fleet-enemas.html
From 
Blogs 
to 
Boards: 
AAHPM 
2012 
Pre-­‐Course 
from 
the 
contributors 
of 
Pallimed 
& 
Geripal 
Notes Question HPM2 
Page 4 of 48 
Walking into a room at your hospice inpatient unit you see a tired appearing female patient 
lying in bed with soft moaning, holding her abdomen. She has end stage CHF and no history of 
cancer. Review of your notes show decreasing oral intake and increased time in bed. Her nurse 
reports she disimpacted her yesterday after suppositories and enemas were ineffective for 
worsening constipation. 
Medications include: Fentanyl 50mcg patch (on for several weeks), Senna 2 tabs BID, Colace 
daily, Recent enema, and docusate suppository 
Exam: Cachectic female, Scaphoid abdomen, hypoactive bowel sounds, formed (but not hard) 
stool on rectal exam. 
What is the next best step? 
a) Write an order for methylnaltrexone 8mg subcutaneously x1 now. 
b) Switch her from a fentanyl patch to a morphine pump so you can better manage her 
abdominal pain. 
c) Write an order for octreotide 200mcg subcutaneously twice daily for three days 
d) Place an NG and give her polyethylene glycol daily until she has a bowel movement or 
regains ability to swallow and you can remove the NG tube 
Discussion: Correct answer is (a) 
a) The patient likely has opioid induced constipation (OIC). Methylnaltrexone is a mu-opioid 
receptor antagonist and is related to naloxone. After ruling out bowel 
obstruction, fecal impaction and any other abdominal process, you give 
methylnaltrexone at 0.15mg/kg subcutaneously, usually 8 (patients < 136lbs) or 12 mg 
(patients over 136lbs). About 60 percent of patients will have a BM in under 4 hours. 
Usually within 30 minutes of the first dose. Number needed to treat was 2.2 (pretty 
darn good). One barrier is cost. At $48 per 8mg dose this is a costly way to manage 
constipation. 
b) While controlling abdominal pain is important relieving the cause of the abdominal 
pain takes precedence. Opioids may be the cause of her pain – increasing them is not 
indicated. With the exception of imminently dying patients, proper treatment of OIC 
will lead to its resolution and function can be improved. 
c) Octreotide has a role in palliative care for malignant bowel obstruction (MBO), not 
constipation. This patient does not have a cancer history and sudden onset nausea and 
vomiting that may be signs for a MBO. Octreotide also is expensive-costing between 
$40 and $80 per dose. 
d) Placing a nasogastric tube should be avoided whenever possible when there are less 
invasive measures available. The patient can swallow oral laxatives, and does not 
have an MBO and so does not have any minimal indications for an NGT in any case. 
Polyethylene glycol is helpful as an osmotic laxative and is often employed as a first 
line option for OIC. It is often more helpful as part of a maintenance regimen or for 
mild to moderate constipation. 
References: 
• Thomas, Jay et. al. Methylnaltrexone for Opioid Induced Constipation in Advanced 
Illness. 2008. NEJM 358 (22): 2332-2343. 
• Yuan, Chun-Su. Methylnaltrexone Mechanisms of Action and Effects on Opioid 
Bowel Dysfuction and Other Opioid Adverse Side Effects. The Annals of 
Pharmacotherapy, 2007. 41: 984- 993
From 
Blogs 
to 
Boards: 
AAHPM 
2012 
Pre-­‐Course 
from 
the 
contributors 
of 
Pallimed 
& 
Geripal 
Notes Question HPM3 
Page 5 of 48 
During a hospice interdisciplinary team meeting, you hear about a 53 year old resident of the 
local nursing home. He has ALS with bulbar attributes, and is starting to have difficulty 
swallowing and speaking. He is bedbound most of the day. He has had two episodes of 
aspiration pneumonia in the last month. 
His nurse describes the scene with the patient’s wife, Sally, at his side, squeezing his hand with 
one hand and her rosary with the other. 
He explained to the nurse, “I told Sally that I don’t want a feeding tube. I’ve had a good life 
and have few regrets. I saw my father-in-law die on a feeding tube and I would not want to go 
through that, or put my wife through that. But I am Catholic. Our friend at the parish said that I 
have to ‘do everything’ to prolong my life – especially when it comes to nutrition - or I will go 
hell. I don’t want to go to hell.” His wife nods emphatically. 
During the interdisciplinary care meeting, the chaplain (in his role as teacher) asks you to 
explain to the team what your understanding of the Catholic doctrine is as pertaining to this 
patient. 
What do you say? 
a) My understanding is that medically assisted nutrition is obligatory for patients who are 
unable to take food by mouth. 
b) My understanding is that medically assisted nutrition is morally optional for most 
patients at the end of life. 
Discussion: Correct answer is (b) 
a) Some interpret the teachings of the church to mandate artificial nutrition at the end of 
life, especially with the media coverage of Terri Schiavo. But the doctrine is more 
nuanced than that. “58. In principle, there is an obligation to provide patients with food 
and water, including medically assisted nutrition and hydration for those who cannot 
take food orally. This obligation extends to patients in chronic and presumably 
irreversible conditions (e.g., the “persistent vegetative state”) who can reasonably be 
expected to live indefinitely if given such care.” (from: section 58. Ethical and 
Religious Directives for Catholic Health Care Services.) 
b) The discussion is actually more complex then that: Medically-assisted nutrition and 
hydration become morally optional when they cannot reasonably be expected to 
prolong life or when they would be “excessively burdensome for the patient or [would] 
cause significant physical discomfort, for example resulting from complications in the 
use of the means employed. “59. The free and informed judgment made by a 
competent adult patient concerning the use or withdrawal of life-sustaining procedures 
should always be respected and normally complied with, unless it is contrary to 
Catholic moral teaching.” (from: section 59. Ethical and Religious Directives for 
Catholic Health Care Services.) 
References: 
http://www.pallimed.org/2010/01/catholic-directives-on-artificial.html 
http://www.pallimed.org/2008/08/media-coverage-of-terri-schiavo.html
From 
Blogs 
to 
Boards: 
AAHPM 
2012 
Pre-­‐Course 
from 
the 
contributors 
of 
Pallimed 
& 
Geripal 
Notes Question HPM4 
Page 6 of 48 
Mrs Dole, a 68 year old with 20 year history of Diabetes Mellitus Type II is referred to 
Palliative Care from Oncology with Stage III Nasopharyngeal carcinoma. Nausea is the key 
concern. For last 3 years she has had early satiety but maintained weight. Since initiating 
chemotherapy, she has had nausea for the first 2 days of her chemotherapy cycle, which then 
resolves. 
1 week after the last round of chemotherapy she required intravenous fluids for dehydration. 
Now 2 weeks later is having intermittent severe nausea. It can be provoked by sudden changes 
in body position. She fell once because she lost her balance. Usually she does not vomit, but 
occasionally does. She describes a feeling of the room spinning associated with the nausea. 
Of the following options, which drug is most targeted to this patient’s specific nausea type: 
a) Ondansetron 
b) Prochlorperazine 
c) Metoclopramide 
d) Diazepam 
e) Meclizine 
Discussion: (e) are the correct answers 
This patient has had multiple types of nausea, however currently her major nausea type seems 
to be vestibular. She may have developed an otolith while dehydrated. Some chemotherapeutic 
agents are ototoxic and can cause vestibular symptoms including hearing loss, tinnitus, 
vertigo/nausea. She also has had chemotherapy induced nausea, as well as diabetic 
gastroparsis. For the boards, probably the default choice for nausea will be D2 blockers, 
however there are certain types of nausea for which D2 blockers are not the best choice. 
a) Ondansetron and the other ‘-setrons’ are HT3 receptor blockers and have excellent 
evidence for the treatment of chemotherapy induced nausea, and post-operative 
nausea. While used widely for other types of nausea including opioid-associated, there 
is less evidence to support them for these practices. They are exceedingly safe and 
well-tolerated; they are constipating. 
*** Chemotherapy induced nausea/vomiting is considered acute when it occurs <24h 
after chemo infusion, and delayed if >24h. Delayed n/v usually occurs in the several 
days after chemotherapy, but not weeks. First line treatments to prevent acute CINV 
including 5HT3 blockers and steroids. NK-1 blockers such as aprepitant and 
gluclocorticoids are also used, especially for mod-highly emetogenic chemo. NK-1 
blockers and steroids also prevent delayed N/V; 5HT3 blockers less so. D2 blockers 
are no longer first line agents as 5HT3 blockers have clearly shown superior efficacy 
and safety. Doses of metoclopramide needed to be effective are 1-2mg/kg IV! 
b) Prochlorperazine and other D2 blockers such as haloperidol target the Chemoreceptor 
trigger zone and D2 receptor. They are the work-horses of nausea treatment. 
c) While the patient has some component of diabetic gastroparesis suggested by satiety 
and long history of DM, he is not bothered by emesis with meals. Metoclopramide 
targets D2 receptors primarily in the gut, and has some prokinetic features, but its role 
long-term for gastroparesis is controversial as it causes EPS such as tardive dyskinesia. 
d) Diazepam and benzodiazepines are effective for anticipatory nausea/vomiting which 
occurs in ~25% of chemo patients. Behavorial/cognitive treatments, and integrative 
modalities are probably helpful too. Aggressive prevention of CINV can help prevent 
anticipatory n/v. 
e) She has what seems to be vestibular symptoms. Anticholinergic drugs such as 
meclizine, scopolamine, promethazine, and even diphenhydramine are potential drugs. 
CNS side effects such as sedation, confusion; as well as orthostatis and xerostomia are 
worrisome side effects. 
References: 
• http://www.pallimed.org/2007/09/vatican-tube-feeding-more-on-abigail.html 
• http://jama.ama-assn.org/content/298/10/1196.full.pdf+html 
• Hain TC, Uddin M. Pharmacological treatment of vertigo. CNS 
Drugs. 2003;17:85–100. 
• http://www.oncolink.org/resources/article.cfm?c=16&s=59&ss=224&id=1004
From 
Blogs 
to 
Boards: 
AAHPM 
2012 
Pre-­‐Course 
from 
the 
contributors 
of 
Pallimed 
& 
Geripal 
Notes Question HPM5 
Page 7 of 48 
In hospice IDT, you discuss the case of a 68 year old female with ovarian cancer with abdominal 
pain and sudden onset nausea and vomiting. She has had no recent bowel movements and is on 
minimal opioids. You suggest a trial of octreotide for a likely malignant bowel obstruction and the 
nurses say “Doctor! You say we can use octreotide for everything! Is there anything octreotide 
can’t be used for in hospice?” 
Which one of the following is not a potential scenario to use octreotide? Choose the best answer. 
Answers 
a) A 37 year old male with end stage alcoholic hepatitis who starts vomiting blood 
b) A 90 year old with a severe diarrhea with a history of a rectal tumor and radiation burns to 
the perineal area 
c) A 42 year old female with a tense distended abdomen leaking a small amount from a 
previous paracentesis site. 
d) A 27 year old male with a malignant wound with copious drainage 
e) A 31 year old female with abdominal pain from opioid-induced constipation 
Discussion: Answer is (e) 
a) Octreotide is the Swiss Army Knife of palliative medications. It is a synthetic analog of 
somatostatin and has many mechanisms of action: in general, it has a global effect to 
decrease secretions primarily in the GI tract It can be costly as a medication alone but it 
could reduce the system cost by avoiding hospitalizations. You should talk with your 
local pharmacist to see about availability and cost in your local programs. It is typically 
administered via intermittent subcutaneous dosing. 
Study published in 2000 compared octreotide infusion with sclerotherapy and found that 
octreotide to be as effective as sclerotherapy regarding hemostasis at 48 hours and on day 
7 after the index bleeding episode. So for the patient looking to avoid hospitalization with 
acute variceal bleed this may be a helpful (but expensive) medication. 
b) While it does not work as a prophylactic treatment to prevent chemo and radiation 
induced diarrhea a few studies have shown that it can treat existing diarrhea related to 
these two common cancer treatments. 
c) Rapidly accumulating ascites or situations where repeat paracentesis or drain may not be 
readily available have been shown to be responsive to octreotide. It also has been 
reported for use in pleural effusions related to cirrhosis. 
d) Tumor related secretions have been show to respond to octreotide 
e) Indications for octreotide include (via palliativedrugs.com) : symptoms associated with 
unresectable hormone-secreting tumors, e.g. carcinoid, VIPomas, glucagonomas and 
acromegaly; prevention of complications after elective pancreatic surgery; †bleeding 
esophageal varices; †salivary, pancreatic and enterocutaneous fistulas; †intractable 
diarrhea related to high output ileostomies, AIDS, radiotherapy, chemotherapy or bone 
marrow transplant;†inoperable bowel obstruction in patients with cancer; †hypertrophic 
pulmonary osteo-arthopathy;†ascites in cirrhosis and cancer; †buccal fistula; †death rattle 
(noisy respiratory secretions); †bronchorrhea;†reduction of tumor-related secretions. 
References: 
§ http://cases.pallimed.org/2008/06/am-i-really-going-to-have-to-live-like.html 
§ http://www.pallimed.org/2008/11/octreotide-for-radiation-induced.html 
§ Freitas DS, Sofia C, Pontes JM, Gregório C, Cabral JP, Andrade P, Rosa A, Camacho E, 
Ferreira M, Portela F.... (2000) Octreotide in acute bleeding esophageal varices: a 
prospective randomized study. Hepato-gastroenterology, 47(35), 1310-4. PMID: 
11100339 
§ Kalambokis, G. (2006-01) Octreotide in the treatment of refractory ascites of cirrhosis. 
Scandinavian Journal of Gastroenterology, 14(1), 199-121. DOI: 
10.1080/00365520510024043 M 
§ Martenson et al. The efficacy of octreotide in the therapy of acute radiation-induced 
diarrhea: a randomized controlled study. International Journal of Radiation 
OncologyBiologyPhysics, 54(1), 195-202. DOI: 10.1016/S0360-3016(02)02870-5
From 
Blogs 
to 
Boards: 
AAHPM 
2012 
Pre-­‐Course 
from 
the 
contributors 
of 
Pallimed 
& 
Geripal 
Notes Question HPM6 
Page 8 of 48 
You visit a patient at home receiving hospice care for cancer. Her pain has been well controlled with 
long acting morphine 60mg BID and occasional PRN doses of short acting liquid morphine (10mg) 
over the past few weeks: she had been tolerating this well. She has had recent progressive functional 
decline and is currently at a PPS of 20%. In the last 24 hours the patient has vomited and has been 
more lethargic and having difficulty swallowing pills. She appears uncomfortable. In your 
examination you see a very thin patient who appears to be dying with a prognosis in the few days to a 
week range. 
The patient’s son is a respiratory therapist at a hospital and is insisting you change the patient’s opioid 
to a fentanyl patch because “it is less sedating than morphine.” 
The best response is: 
a) Because the patient is cachectic, you tell the family that fentanyl transdermal patches are not 
indicated because the medication will not be absorbed. 
b) Agree with the son and convert the patient to a 37.5mcg/hr fentanyl patch with oral morphine 
liquid 10mg q1 hour PRN 
c) Because the fentanyl will not be effective for over 24 hours, continue the long acting 
morphine sulfate 60mg BID but give it rectally instead of by mouth 
d) Suggest starting a morphine infusion via her port at 1.7mg/hr basal with a 3mg q30min bolus 
PRN after talking with the son about his concerns about sedation. 
Discussion: Answer is (d) 
a) Cachexia has not been show to be a CLINICALLY RELEVALANT factor in absorption of 
transdermal fentanyl. Cachexia will decrease the amount of subcutaneous fat which is where 
fentanyl is stored AFTER absorption through the dermal layers. In 2009 Heiskanen did a 
study comparing blood levels between cachectic and non-cachectic volunteers and found no 
significant difference, although cachectic patients had a slightly lower mean concentration. 
There was no difference in VAS score. 
b) Fentanyl is not less sedating than morphine at equianalgesic doses. Also there is no 
37.5mcg/hr patch or 12.5mcg/hr patch. As written, and described by the manufacturer, the 
“12.5mcg/hr patch” is labeled and Rx’d as a “12mcg/hr” patch to prevent confusion with 
Rx’ing 125mcg/hr. As for the conversion, it could be acceptable to use a 25mcg/hr & 
12mcg/hr patch (total 37mcg/hr) per the Fentanyl transdermal product insert. It recommends 
25mcg/hr for someone on OMDD of 60-134mg and 50mcg/hr for someone on OMDD 135- 
224, so this is right in the middle. The Breitbart/Donner conversion of 2mg morphine = 
1mcg/hr transdermal fentanyl which would be 60mcg/hr of fentanyl (You could choose 50 or 
75 depending on other clinical circumstances). 
c) The pharmacokinetics of fentanyl do not warrant switching to it if otherwise indicated. 
Morphine still has time to circulate and get out of her system, and fentanyl begins to reach 
significant blood concentrations 8-12 hours after application. If needed, she can be bridged 
with a few doses of liquid morphine. In addition, people do not prefer rectal administration if 
it could be avoided. 
d) A morphine continuous infusion allows for the continuation of the current effective opioid in 
a patient who is likely not going to regain swallowing function. The conversion is most 
direct (120mg OMDD = 40mg daily IV = 1.7mg/hr (1.5 if your pumps are limited in decimal 
rates). A 3 mg IV morphine bolus most closely replicates the 10mg oral morphine doses that 
were effective prior. If you did not choose this answer because your hospice doesn’t use 
continuous infusions (expense, nurse familiarity, not available from local pharmacy) then 
start talking with your hospice to decrease these barriers to an effective and essential tool to 
good pain management. 
References: 
• http://www.pallimed.org/2009/05/cachexia-and-absorption-of-transdermal.html 
• Heiskanen, Tarja. (2009-7) Transdermal fentanyl in cachectic cancer patients. PAIN, 70(1-2), 
928-222. DOI: 10.1016/j.pain.2009.04.012 
• Mercadante, Sebastiano. (2012-01-09) Sustained-release oral morphine versus transdermal 
fentanyl and oral methadone in cancer pain management. European Journal of Pain, 7(Suppl.
From 
Blogs 
to 
Boards: 
AAHPM 
2012 
Pre-­‐Course 
from 
the 
contributors 
of 
Pallimed 
& 
Geripal 
Page 9 of 48 
A), 320-1046. DOI: 10.1016/j.ejpain.2008.01.013 
• Weissman DE. Converting to/from Transdermal Fentanyl, 2nd Edition. Fast Facts and 
Concepts. July 2005; 2. Available at: http://www.eperc.mcw.edu/fastfact/ff_002.htm. 
• Tatum IV WO. (2002) Adult patient perceptions of emergency rectal medications for 
refractory seizures. Epilepsy & behavior : E&B, 3(6), 535-538. PMID: 12609248 
• Colbert SA, O'Hanlon D, McAnena O, & Flynn N. (1998) The attitudes of patients and health 
care personnel to rectal drug administration following day case surgery. European journal of 
anaesthesiology, 15(4), 422-6. PMID: 9699099 
• Mercadante, Sebastiano. (2012-01-09) Sustained-release oral morphine versus transdermal 
fentanyl and oral methadone in cancer pain management. European Journal of Pain, 7(Suppl. 
A), 320-1046. DOI: 10.1016/j.ejpain.2008.01.013
From 
Blogs 
to 
Boards: 
AAHPM 
2012 
Pre-­‐Course 
from 
the 
contributors 
of 
Pallimed 
& 
Geripal 
Notes Question HPM7 
Page 10 of 48 
JY, a 28 year old woman with advanced cystic fibrosis and Burkholderia cenocepacia 
colonization is hospitalized for a cystic fibrosis exacerbation. She has chronic chest wall pain 
from coughing and pleurisy, and recently broke 2 ribs from coughing. She is on IV 
glucocorticoids, IV ketorolac, IV ketamine prior to vest treatments, and lorazepam. Prior to her 
hospitalization, she took oxycodone ER 30mg q12h. Currently she is on a hydromorphone IV 
PCA at 2mg/hour, with 2mg q30 minute boluses. She used 72mg of IV dilaudid in the last 24h. 
Despite this she is becoming drowsy, and reports her pain is minimally improved and still 
severe for most of the day: 7-8/10, and ‘nearly intolerable’ during vest therapy 
The best next step is to: 
a) Increase her PCA basal and ‘bolus’ doses by 50% and monitor for 24 hours. 
b) Add a 5% lidocaine patch to her chest wall over her rib fractures 
c) Discontinue hydromorphone and switch the patient to another opioid 
d) Advise the primary team to stop vest therapies 
Discussion: Correct answer (c) 
a) Indications for opioid rotation are 1) dose-limiting side effects such as sedation, 
nausea, pruritus, myoclonus from the patient’s current opioid, 2) need for a new dosing 
route (patient cannot swallow), 3) costs/insurance changes, 4) inadequate analgesia 
despite ‘adequate’ dose-escalation of the current opioid. There is no consensus on what 
4 actually means, however rapidly escalating someone by an order of magnitude (as in 
this case) without good response, is generally a scenario in which you’d consider 
rotation (if not long before). Is not best next step given the above discussion 
b) No data at all suggesting the lidocaine patch is effective for pain from fractures 
c) Is the correct answer: Morphine, methadone, or fentanyl are all reasonable options. 
Some prefer methadone in these sorts of settings, but no actual data to support that and 
probably not tested on the boards. Another reasonable approach in this situation 
would be to consult a pain interventionalist for regional options. 
d) Opioid rotation is reasonable first, before advising this, as it will likely affect the 
patient’s ability to recover. 
References: 
• http://www.pallimed.org/2008/07/methadone-methadone-methadone.html 
• http://www.pallimed.org/2010/01/outpatient-rotations-to-methadone.html 
• http://www.pallimed.org/2005/07/transdermal-fentanyl-to-methadone.html
From 
Blogs 
to 
Boards: 
AAHPM 
2012 
Pre-­‐Course 
from 
the 
contributors 
of 
Pallimed 
& 
Geripal 
Notes Question HPM8 
Page 11 of 48 
Mr. Smith is a 72 year old patient was admitted to hospital from his nursing home for 
respiratory distress due to CHF exacerbation. Despite aggressive diuresis attempts, his 
respiratory distress continued and his urine output remained minimal (~30ml/day). 
PMH: heart failure, moderate dementia, renal insufficiency 
Home medications: furosemide 40mg po bid, metoprolol 25mg bid, donepezil 10mg daily, 
olanzapine 5mg qhs. 
After a conversation with his son (health care proxy) the patient was "made CMO" (comfort 
measures only) by the hospitalist service and resident team two days ago. He was then started 
on a morphine drip “titrate by 1mg as needed for pain or shortness of breath”, his donepezil, 
olanzapine and diuretics continued, other medications stopped. 
His intern calls in a panic: “We promised to make him comfortable, that he would die in 2 days, 
but he is still alive and the family does not know why he is in such pain – even with light touch 
– crying out & jerking.” 
What is your recommendation? 
a) Stop morphine drip and start fentanyl and lorazepam prn 
b) Increase morphine and olanzapine 
c) Increase morphine and add lorazepam prn 
d) Stop morphine drip and start fentanyl, increase olanzapine 
Discussion: Correct answer (a) 
Key points: 
• Opioid neurotoxicity in the setting of renal failure/azotemia is the most likely answer. 
Morphine metabolites build up disproportionately in the setting of renal failure. 
Morphine 3-glucoronide is a neurostimulant that can lead to agitated delirium, 
myoclonus, hyperalgesia, and even seizures. Morphine and hydromorphone are the 
most common culprits. Morphine 6-glucoronide is a metabolite that is active on the 
mu-opioid receptor, and thus is not a major player in terms of inducing agitated 
neurotoxicity. 
• Fentanyl does not have the same metabolites and thus has a lower risk of agitated 
neurotoxicity. Since there are no active metabolites that build up in renal failure, it is 
the safest of the “pure” opioids for patients on dialysis or who are oliguric. Methadone 
is another opioid that is nearly ~100% excreted in the stool 
• The treatment for this is to rotate off current opioid. Fentanyl is safer option in renal 
failure. 
• Antipsychotics can worsen the symptoms 
• Benzodiazepines can help treat myoclonus and prevent seizures 
References: 
Robin K Wilson, David E Weissman; Neuroexcitatory effects of opioids: patient assessment, 
2nd ed. EPERC# 057 http://www.eperc.mcw.edu/EPERC/FastFactsIndex/ff_057.htm 
http://www.aahpm.org/apps/blog/?tag=boards 
Smith, H. S. (2009). Opioid metabolism. Mayo Clinic proceedings. Mayo Clinic, 84(7), 613-24.
From 
Blogs 
to 
Boards: 
AAHPM 
2012 
Pre-­‐Course 
from 
the 
contributors 
of 
Pallimed 
& 
Geripal 
Notes Question HPM9 
Page 12 of 48 
BJ, a 65 yo woman with known non-small cell lung cancer, metastatic to her mediastinum, 
contralateral lung, and supraclavicular lymph nodes, returns to your clinic for follow-up for her 
cancer-related pain. She is getting chemotherapy, and has always expressed a desire for ‘the 
most aggressive’ treatments available for her cancer. 
She complains of 2 weeks of worsening, midline low back pain. She has noticed difficulty in 
rising from chairs/toilet, and needed a wheelchair to make it into the clinic area today from the 
parking garage due to weakness. Examination is notable for an unremarkable back/spine exam, 
and 4/5 strength bilaterally in her lower extremities both proximally and distally. 
You obtain a stat MRI which shows a T12 vertebral metastasis and cord compression. 
In addition to administering glucocorticoids, then next best step is to: 
a) Arrange an urgent radiation oncology consultation for the next day. 
b) Admit her to the hospital, and arrange a stat radiation oncology consultation. 
c) Admit her to the hospital, and arrange a stat spine surgery consultation. 
d) Adjust her pain medications appropriately, and instruct her to contact you immediately 
if her pain or disability worsens 
Discussion: Correct answer is (c) 
This is a medical emergency. 
• Vertebral metastases, putting a patient at risk for cord compression, should be 
considered in any patient with new back pain and cancer. New or otherwise suspicious 
back-pain can be evaluated urgently with a non-contrast MRI of the entire spine. 
• If patients have neurologic symptoms of LE weakness and/or bladder, bowel 
dysfunction, it is a medical emergency and patients needs stat imaging, steroids, and 
intervention. Neurologic deficits, once present, can rapidly progress to permanent 
paraplegia within 24h. 
• The role of steroids + XRT vs steroids + surgery is unclear. A recent trial indicated 
better outcomes with immediate surgery, especially for patients who came in with 
severe weakness. 84% of patients vs 54% were ambulatory after treatment course with 
surgery vs radiation without surgery. Actual practice has not necessarily caught up 
with this, and will depend on local, institutional resources. 
References: 
• http://www.pallimed.org/2005/08/surgery-better-than-radiation-steroids.html 
• http://www.pallimed.org/2008/03/spinal-cord-compression-copd-prognosis.html 
• http://www.eperc.mcw.edu/EPERC/FastFactsIndex/ff_237.htm 
• http://www.eperc.mcw.edu/EPERC/FastFactsIndex/ff_238.htm
From 
Blogs 
to 
Boards: 
AAHPM 
2012 
Pre-­‐Course 
from 
the 
contributors 
of 
Pallimed 
& 
Geripal 
Notes Question HPM10 
Page 13 of 48 
Mr. G. Da Salva is a 68 year old construction worker who has metastatic non-small cell lung 
cancer involving his right femur and pelvis. 
Medications include: Morphine ER 200mg bid, Morphine IR 30-60mg PO q2 hours prn, and 
dexamethasone 8mg daily. At rest his pain is well managed, 2/10. 
However, he fears movement due to severe pain and spends most of the day in his recliner, 
avoiding showering or changing or helping with the meals. He uses approximately 5 doses daily 
of 60mg short-acting morphine for this pain but once it starts to work the pain has often 
spontaneously subsided and he becomes sleepy and confused. 
Which of the following is LEAST appropriate? 
a) Take a short-acting morphine prior to a clustering his activities: showering, changing, 
fixing a meal. 
b) Add sublingual fentanyl 200mcg to take prior to his activities. 
c) Increase his long-acting morphine to 200mg tid. 
d) Single-fraction radiation therapy to his pelvis and femur. 
e) Intrathecal pump with morphine and low-dose bupivacaine. 
Discussion: Correct answer is (c) 
This is an example of incidental pain. It differs from breakthrough pain in that it is associated 
with movement, and diminishes as soon as the activity ends. The challenge with this form of pain 
management is that the pharmacology of systemic opioids does not tend to match the timing of 
this type of pain. 
a) Clustering his activities together so that they all take longer may better match the t1/2 of 
the short-acting morphine, but he will still need to take the medicine approximately 40 
minutes prior to starting the activities. 
b) Sublingual or buccal fentanyl has a shorter half-life and shorter time to onset than other 
oral opioids and is a better option. 
c) Is the correct answer: Increasing the long-acting morphine is the least appropriate 
because this will not help the incidental pain and may worsen his confusion when he is 
not moving around. 
d) Single fraction radiation therapy would be very appropriate in this setting and would 
likely be one of the most preferred interventions as long as he had not previously been 
irradiated at the site of pain. 
e) Intrathecal pain medication delivery is another good option. However, this is an 
expensive procedure and requires a prognosis of at least 3 months to assure coverage by 
insurance plans. Because the dose of opioid is a fraction of systemic opioid delivery, its 
risk of side-effects is lower and is a more effective means of managing incidental pain in 
the lower back and lower extremities. 
References: 
• http://www.aahpm.org/apps/blog/?p=809 
• http://www.pallimed.org/2009/12/poll-results-palliative-care-experience.html 
• Bruera, E., & Kim, H. N. (2003). Cancer pain. JAMA : the journal of the American 
Medical Association, 290(18), 2476-9. 
• Chow, E., Harris, K., Fan, G., Tsao, M., & Sze, W. M. (2007). Palliative radiotherapy 
trials for bone metastases: a systematic review. Journal of clinical oncology : official 
journal of the American Society of Clinical Oncology, 25(11), 1423-36. 
• Smith, T. J., Swainey, C., & Coyne, P. J. (2005). Pain management, including intrathecal 
pumps. Current Pain and Headache Reports, 9(4), 243-248. Current Medicine Group 
LLC.
From 
Blogs 
to 
Boards: 
AAHPM 
2012 
Pre-­‐Course 
from 
the 
contributors 
of 
Pallimed 
& 
Geripal 
Notes Question HPM11 
Page 14 of 48 
Mr. Z is a 87 year old with advanced dementia living in a nursing home. At baseline he cannot 
recognize family members, is dependent on all ADLs (dressing, toileting, bathing) but does not 
have urinary or fecal incontinence. He speaks about 1-2 intelligible words per day and he has had 
progressive loss of ability to ambulate. He is now admitted to the hospital after sustaining a hip 
fracture from a fall. 
When discussing treatment options for his hip fracture, his wife asks you how long he likely has to 
live. 
Given his current state of health, what would be the most appropriate answer: 
a) Given that he does not meet FAST 7C criteria his prognosis is likely greater than 6 
months 
b) He meets NHPCO Guidelines for hospice eligibility which means he likely has less than a 
6 month prognosis 
c) Given his advanced dementia and recent hip fracture, his 6 month mortality risk exceeds 
50% 
d) As with most individuals with advanced dementia, his life expectancy is likely weeks to 
months 
Discussion: Correct answer is (c) 
References: 
a) The FAST scale measures functional status in dementia and consists of 7 major stages 
split into 16 different sub-stages. Hospice eligibility criteria for dementia are based 
largely on whether a patient meets or exceeds Stage 7c on the FAST and whether they 
have at least one complication from their dementia. Unfortunately, these criteria do not 
accurately predict 6-month survival. 
b) The current National Hospice and Palliative Care Organization (NHPCO) guidelines for 
hospice eligibility are of limited accuracy in predicting death within 6 months. In 
addition, NHPCO guidelines relies on the FAST staging, which fails to account for the 
observation that dementia often does not progress in a sequential pattern. The patient is 
dependent on ADLs (dressing, toileting, bathing) but does not have urinary or fecal 
incontinence (FAST Stage 6d and 6e). His speech has declined from less than 6 
intelligible words per day (7a) to one or less (7b), and he has had progressive loss of 
ability to ambulate (7C), however since he does not have 6d and 6e, Mr. Z is not 
considered Fast Stage 7c, rather he is Fast 6C. 
c) Is the correct answer: Individuals with advanced dementia that are either hospitalized for 
either pneumonia or for hip fracture have a very poor prognosis. In one study, six-month 
mortality for patients with end-stage dementia and hip fracture was 55% compared with 
12% for cognitively intact patients. 
d) Advanced dementia is a terminal condition; however estimating prognosis is difficult due 
to the prolonged period of severe functional and cognitive impairment that occurs prior to 
death. For those with advanced disease who reside in a nursing home, the 6-month 
mortality rate is 25% with a median survival in one study of only 478 days. 
References: 
§ http://www.geripal.org/2009/10/there-is-important-article-in-current.html 
§ www.eprognosis.org 
§ Mitchell SL, Miller SC, Teno JM, Kiely DK, Davis RB, Shaffer ML. Prediction of 6-month 
survival of nursing home residents with advanced dementia using ADEPT vs hospice 
eligibility guidelines. JAMA. Nov 3 2010;304(17):1929-1935. 
§ Morrison RS, Siu AL. Survival in end-stage dementia following acute illness. Jama. Jul 5 
2000;284(1):47-52. 
§ Mitchell SL, Teno JM, Kiely DK, et al. The clinical course of advanced dementia. N Engl J 
Med. Oct 15 2009;361(16):1529-1538.
From 
Blogs 
to 
Boards: 
AAHPM 
2012 
Pre-­‐Course 
from 
the 
contributors 
of 
Pallimed 
& 
Geripal 
Notes Question HPM12 
Page 15 of 48 
Mrs. A is an 88 year old with advanced dementia who lives in a nursing home. She has at 
baseline some difficulty with eating as she pockets food in her mouth and occasionally coughs 
after swallowing. She is now hospitalized for an aspiration pneumonia. In addition to the 
antibiotics she is on in the hospital, her only other medications include HCTZ for hypertension 
and a baby aspirin. She has never taken a cholinesterase inhibitor . 
What is the best next step? 
a) A trial of both a cholinesterase inhibitors and memantine 
b) Feeding tube insertion 
c) Careful hand feeding and good oral care 
d) Addition of olanzapine to treat her pocketing of food behavior 
Discussion: Correct answer is (c) 
a) Acetylcholinesterase inhibitors, such as donepezil, galantamine, and rivastigmine, and 
the N-methyl-D-aspartate (NMDA) antagonist memantine have some evidence for a 
statistically significant improvement in cognitive, functional, and behavioral outcomes 
in indivudals with moderate-to-severe dementia. However, these improvements have 
marginal clinical significance. Adverse events are common with these agents, most 
commonly nausea, vomiting, and diarrhea. There is no evidence to suggest that they 
decrease eating problems or risk for aspirations. 
b) Placement of PEG tubes often occur after transfer to an acute care facility for eating 
problems or pneumonia, despite the fact that feeding tubes have not been shown to 
improve survival for individuals with dementia. There is also no evidence that tube 
feeding prevents aspiration pneumonia, decreases the risk for pressure ulcers, improves 
patient comfort, or prolongs life. 
c) Oral care has been shown to decrease incidence of pneumonia, number of febrile days, 
and death from pneumonia in nursing home residents. 
d) Antipsychotics have not been shown to improve eating behaviors in dementia. There is 
a moderate short-term efficacy when treating agitation, serious side effects that include 
risk of stroke and death limit their clinical use. 
References: 
• http://www.geripal.org/2011/07/decisions-on-feeding-tubes-in-advanced.html 
• Finucane TE, Christmas C, Travis K. Tube feeding in patients with advanced dementia: 
a review of the evidence. JAMA. Oct 13 1999;282(14):1365-1370. 
• Yoneyama T, Yoshida M, Ohrui T, et al. Oral care reduces pneumonia in older patients 
in nursing homes. J Am Geriatr Soc. Mar 2002;50(3):430-433. 
• Schneider LS, Tariot PN, Dagerman KS, et al. Effectiveness of atypical antipsychotic 
drugs in patients with Alzheimer's disease. N Engl J Med. Oct 12 2006;355(15):1525- 
1538
From 
Blogs 
to 
Boards: 
AAHPM 
2012 
Pre-­‐Course 
from 
the 
contributors 
of 
Pallimed 
& 
Geripal 
Notes Question HPM12 – Part 2 
Page 16 of 48 
The family is concerned that Mrs. A’s aspirations will continue if she continues to be fed by 
hand in the nursing home. They would like to know about more about the risks of a feeding 
tube placement. 
The most appropriate risk to include in the discussion is: 
a) She will have a 1 in 10 chance of a major surgical complication in the perioperative 
period. 
b) She is unlikely to have a tube related complication after the perioperative period 
c) Once the tube is placed, it would be technically difficult to electively remove the tube 
d) She will have a 1 in 3 chance of requiring chemical or physical restraints to prevent 
tube removal 
Discussion: Correct answer is (d) 
a) Up to one-third experience transient gastrointestinal adverse effects (ie, vomiting, 
diarrhea) but major complications like bowel perforations are rare (1%) 
b) Tube dislodgement, blockage, and leakage are common (4%-11%). One in 5 tube-fed 
residents experiencing a tube related complication necessitating a hospital transfer in 
the year following insertion. 
c) Removal of feeding tubes is not technically difficult. An important pearl to know is 
that if a feeding tube is inadvertently removed, the stoma site will close in a few hours, 
so put in a Foley catheter to keep it open until a new one can be placed. 
d) Over a quarter of tube fed dementia patients are physically restrained after feeding tube 
placement, and nearly a third are placed on sedating medications to prevent them from 
pulling out the feeding tube. 
References: 
• http://www.geripal.org/2011/07/decisions-on-feeding-tubes-in-advanced.html 
• Teno JM et al. Decision-making and outcomes of feeding tube insertion: a five-state 
study. J Am Geriatr Soc. 2011 May;59(5):881-6
From 
Blogs 
to 
Boards: 
AAHPM 
2012 
Pre-­‐Course 
from 
the 
contributors 
of 
Pallimed 
& 
Geripal 
Notes Question HPM13 
Page 17 of 48 
A 54 yo man with a 7 month history of metastatic bladder cancer presents to the cancer center’s 
palliative care clinic. He complains of low mood, anhedonia, feelings of guilt, shame, and 
worthlessness most days for the last 2 months. He says, “Of course I’m depressed – who 
wouldn’t be? I’ve got a cancer that the doctors tell me is terminal. What good am I to my 
family? They’d be better off without me.” 
The best next step would be to: 
a) Tell the patient that he is depressed and recommend a treatment plan for it. 
b) Ask your team’s social worker to see the patient for grief counseling. 
c) Provide emotional support and counseling with the patient that what he is experiencing 
is part of the expected adjustment to having a terminal illness. 
d) Refer the patient to psychiatry for complicated depression. 
Discussion: Correct answer is (a) 
a) Psychiatric disorders including depression and anxiety disorders occur in only a 
minority of patients at the end of life. Published numbers have ranged from 10-40%; 
while higher than the general population, they are not the norm. Persistent low mood, 
feelings of worthlessness, guilt, and shame are highly suggestive of depression. The 
patient should be counseled about this and offered treatment 
b) While this may end up being appropriate for this patient, his symptomotology is most 
c/w depression and a) is the better answer. 
c) See (a). This is not ‘normal.’ 
d) Complicated depression is not a diagnosis. He has untreated depression, and HPM 
specialists should be able to initiate appropriate therapy! 
References: 
• Rayner L et al. Antidepressants for the treatment of depression in palliative care: 
systematic review and metaanalysis. Palliat Med. 25(1):36-51. DOI: 
10.1177/0269216310380764 
• http://www.pallimed.org/2011/02/what-i-learned-at-aahpmhpna-annual.html 
• Irwin SA. Oral ketamine for the rapid treatment of depression and anxiety in patients 
receiving hospice care. JPM. 2010; 13:903-8. 
http://www.ncbi.nlm.nih.gov/pubmed/20636166
From 
Blogs 
to 
Boards: 
AAHPM 
2012 
Pre-­‐Course 
from 
the 
contributors 
of 
Pallimed 
& 
Geripal 
Notes Question HPM14 
Page 18 of 48 
The patient agrees to pharmacologic therapy for his depression, and declines offers of 
counseling/therapy. Your best estimate is that he has 4-8 weeks to live based on performance 
status and tempo of decline. 
Which of the following are appropriate drug approaches for his depression? 
a) Methylphenidate 
b) Ketamine 
c) Dronabinol 
d) Sertraline 
Discussion: Correct answer are (a) and (b) 
a) Is correct: Methylphenidate and other psychostimulants are rapidly-acting, with onset 
of mood elevation occurring ~immediately (if they are going to be effective at all). 
b) Is correct: Ketamine has been described as a depression therapy for decades, albeit one 
on the margins of accepted medical practice; there is a recent resurgence in interest for 
its use at life’s end, because its effects are immediate. Patients usually receive a single 
infusion, which stabilizes mood for weeks at a time. Can be used orally too. 
c) Dronabinol has no defined role as an antidepressant. 
d) Most SSRIs take at least 4 weeks to become effective, which is 50-100% of the 
patient’s anticipated survival. While SSRIs are first-line antidepressants in the general 
population as well as for patients with advanced illness, this is not as true as prognosis 
shortens. 
References: 
• Rayner L et al. Antidepressants for the treatment of depression in palliative care: 
systematic review and metaanalysis. Palliat Med. 25(1):36-51. DOI: 
10.1177/0269216310380764 
• http://www.pallimed.org/2011/02/what-i-learned-at-aahpmhpna-annual.html 
• Irwin SA. Oral ketamine for the rapid treatment of depression and anxiety in patients 
receiving hospice care. JPM. 2010; 13:903-8. 
http://www.ncbi.nlm.nih.gov/pubmed/20636166
From 
Blogs 
to 
Boards: 
AAHPM 
2012 
Pre-­‐Course 
from 
the 
contributors 
of 
Pallimed 
& 
Geripal 
Notes Antidepressant Pop Quiz 
Page 19 of 48 
I say depression, insomnia, anorexia, nausea – You say: 
a) Trazodone 
b) Paroxetine 
c) Mirtazipine 
d) Escitalopram 
I say depression, anxiety, insomnia, neuropathy, You say: 
a) nortriptyline 
b) duloxetine 
c) fluoxetine 
d) venlafaxine 
I say activating antidepressants, You say 
a) fluoxetine 
b) paroxetine 
c) buproprion 
d) citalopram 
I say depression, anxiety, neuropathic pain, advanced age, You say: 
a) duloxetine 
b) nortriptyline 
c) paroxetine 
d) mirtazapine 
Discussion: Correct answers in order are: (c) – (a) – (a, c) – (a) 
• Mirtazapine has side effects which include drowsiness and weight gain, plus some 
antiemetic effects 
• TCAs are the only drug class that directly treat all those symptoms (SNRIs – serotonin 
norepinephrine reuptake inhibitors - like duloxetine and venlafaxine aren’t known to 
be directly effective for insomnia). 
• Paroxetine can be sedating; citalopram more neutral 
• Duloxetine seems to be better tolerated than TCAs, especially in the elderly. For the 
boards, would avoid giving elderly TCAs. Mirtazapine’s role in pain is not well 
defined, especially compared to SNRIs and TCAs.
From 
Blogs 
to 
Boards: 
AAHPM 
2012 
Pre-­‐Course 
from 
the 
contributors 
of 
Pallimed 
& 
Geripal 
Notes Question HPM15 
Page 20 of 48 
Mrs. Phillips is a 91-year-old hospitalized patient who is now actively dying due to end-stage 
pulmonary fibrosis and asbestosis. She has been well palliated during the last several months at 
home where she lived independently, until she developed a pneumonia and was hospitalized. Her 
home medications had not been adjusted in over six weeks. This included: albuterol and atropine 
nebulizers, dexamethasone 2mg every morning, 25mcg/hour fentanyl patch for dyspnea, 
oxycodone concentrate (20mg/ml) 10mg q2 hours prn dyspnea or pain, senna and Colace. She is 
on day 7 of oral antibiotics for presumed pneumonia. She is on oxygen 6 liters via nasal cannula. 
Her last bowel movement was yesterday, and her urine output has been good (250ml or more 
daily.) 
Yesterday she was still oriented, between periods of increasing fatigue and sleep. She showed 
signs of mottling and new secretions causing respiratory rattle. A scopolamine patch 1.5mg was 
started for her increased secretions. 
You are called by the resident who explains to you that this morning Mrs. Phillips is now agitated, 
moaning, and even thrashing at times. This is causing family and floor nurses distress. He asks 
you for advice. 
Which of the following is appropriate? 
a) Stop scopolamine 
b) Start lorazepam 
c) Increase the fentanyl 
d) Stop the fentanyl 
e) Counsel family about the inevitability terminal delirium 
f) Order soft restraints 
Discussion: (a) is correct 
Delirium is a common condition at the end-of-life. It often is considered “terminal” even if 
reversible, however. Terminal delirium should be considered a diagnosis of exclusion or even one 
made in hindsight. While conducting a battery of exhaustive tests to evaluate the cause is not 
usually appropriate or necessary, causes of delirium should be addressed if possible. The most 
common causes of delirium in this setting remain constipation, urinary retention, medications, 
infection, electrolyte abnormalities. Constipation, urinary retention can be ruled out in this patient. 
a) With this patient, the addition of scopolamine is the most likely cause. This is a tertiary 
amine anticholinergic agent, and commonly causes confusion in the elderly. 
b) Lorazepam is not the best option for delirium; neuroleptics, in addition to treating the 
underlying cause (if feasible) are appropriate. 
c) Fentanyl, on the other hand, is less likely the cause. She has been on a stable dose since 
home and was previously tolerating it well. Stopping the fentanyl will likely increase 
delirium, dyspnea and withdrawal symptoms. Increasing the fentanyl, similarly, is 
unlikely to address the agitation – unless the patient has been responding to breakthrough 
oxycodone. 
d) As above 
e) Family members should be comforted, but not that it is an inevitable part of dying. 
f) Restraints should be avoided. 
References: 
• http://www.geripal.org/2010/06/ny-times-article-on-delirium.html 
• http://www.eperc.mcw.edu/EPERC/FastFactsIndex/ff_001.htm
From 
Blogs 
to 
Boards: 
AAHPM 
2012 
Pre-­‐Course 
from 
the 
contributors 
of 
Pallimed 
& 
Geripal 
Notes Question HPM16 
Page 21 of 48 
Mr. J is 58 year old diagnosed with ALS 6 months ago. He is referred to your clinic by his 
primary care doctor to help discuss options to treat a progressive weight loss. He currently lives 
alone in an apartment, is independent of ADLs although he has been having difficulty feeding 
himself due to proximal arm weakness. He complains that he occasionally bursts out crying or 
laughing, but denies feeling depressed. His forced vital capacity (FVC) has remained at 70% for 
the last 3 months. 
The best next step to help treat his progressive weight loss? 
a) Riluzole 
b) PEG Placement 
c) Mobile arm supports and modified cutlery 
d) Non Invasive Positive Pressure Ventilation (NIPPV) 
Discussion: (c) is the correct answer 
a) Riluzole is the only available disease-modifying therapy for ALS. Based on clinical 
trials, riluzole likely prolongs median survival in patients with ALS by 2-3 months 
compared to patients taking placebo.(1) It does little to improve functional outcomes 
or bulbar symptoms. There is no evidence to suggest that it is beneficial for weight 
loss. 
b) PEG placement should be discussed with any individual diagnosed with ALS, 
although attempt to reverse other common reasons for weight loss is warranted before 
PEG placement. Ideally, PEG tubes should be placed before FVC falls below 50%. 
While PEG may be indicated for this patient in the future, currently C is the best 
option. 
c) Individuals living with ALS may have difficulty with the mechanics of both cooking 
and putting food from the plate to the mouth. This may often contribute to weight loss, 
especially for those individuals living alone. Occupational therapy may help maintain 
adequate nutrition by supplying devices such as mobile arm supports and modified 
cutlery. 
d) NIPPV confers a survival benefit and improves quality of life in patients with normal 
or moderately impaired bulbar function, although it does not improve weight 
References: 
• Miller RG, Mitchell JD, Lyon M, Moore DH. Riluzole for amyotrophic lateral 
sclerosis (ALS)/motor neuron disease (MND). Cochrane Database Syst Rev. 2007 Jan 
24;(1):CD001447. 
• Mitsumoto H, Rabkin, JG. Palliative Care for Patients With Amyotrophic Lateral 
Sclerosis “Prepare for the Worst and Hope for the Best”. JAMA. 2007;298(2):207-216
From 
Blogs 
to 
Boards: 
AAHPM 
2012 
Pre-­‐Course 
from 
the 
contributors 
of 
Pallimed 
& 
Geripal 
Notes Question HPM17 
Page 22 of 48 
Mr G is a 74-year-old nursing home resident with coronary artery disease and end-stage renal 
failure (eGFR of 12). He is considering starting treatment with dialysis but would like to know 
more about what life will be like after starting dialysis. 
What would be the most accurate statement in regards to his prognosis 
a) His functional status is likely to improve with renal replacement therapy 
b) His functional status is likely to be maintained at his pre-dialysis level 
c) He is unlikely to have significant symptom burden if he elects not to initiate dialysis 
d) The majority of nursing home residents die within one year of starting dialysis 
Discussion: (d) is the correct answer 
a) Based on a NEJM paper (1) that linked dialysis registry data to activities of daily 
living measures reported by nursing homes in 3,702 patients, patients similar to Mr. G 
did poorly. Within 3 months after the start of dialysis, 61% of the nursing home 
residents had died or had a decrease in functional status as compared with their 
functional status before dialysis. Only 39% had the same functional status that they 
had before dialysis. By 12 months, almost all (87%) nursing home residents had died 
or had a decrease in functional status after starting dialysis. 
b) See a above 
c) The last month of life for individuals who elect not to undergo renal replacement 
therapy is associated with relatively high symptom burden, similar to that of advanced 
cancer.(2) Common symptoms include lack of energy, itching, feeling drowsy, 
shortness of breath, difficulty concentrating, pain, lack of appetite, and swelling of 
arms/legs. Therefore, clinicians should attend to these symptoms as aggressively as 
they do for patients with advanced cancer. We don’t know though whether symptom 
burden is improve or worsened for elderly patients with multiple chronic conditions 
who do elect for renal replacement therapy. 
d) In the NEJM cited above, 58% of these nursing home residents had died 1 year after 
initiating dialysis 
References: 
• http://www.geripal.org/2009/10/how-should-we-counsel-frail-nursing.html 
• http://www.geripal.org/2010/09/dying-without-dialysis.html 
• Kurella Tamura M, Covinsky KE, Chertow GM, Yaffe K, Landefeld CS, McCulloch 
CE. Functional status of elderly adults before and after initiation of dialysis. N Engl J 
Med 2009;361:1539-1547 
• Murtagh FE, et al. Symptoms in the Month Before Death for Stage 5 Chronic Kidney 
Disease Patients Managed Without Dialysis. J Pain Symptom Manage. 2010 
Sep;40(3):342-52.
From 
Blogs 
to 
Boards: 
AAHPM 
2012 
Pre-­‐Course 
from 
the 
contributors 
of 
Pallimed 
& 
Geripal 
Notes Question HPM18 
Page 23 of 48 
George Condi is a 68 y/o male is admitted to ICU for respiratory crisis and found to have renal 
cell carcinoma with a 13 cm mass in the R upper abdomen. He has severe pain, and dyspnea 
with large R sided pleural effusion. With drainage of effusion his dyspnea is improved; a 
tunneled pleural catheter is placed, and he is discharged to home hospice with a PPS of 50. 
The next day his wife calls saying she can’t manage the catheter and she is in tears because his 
pain is 6/10 and he is more short of breath. “You promised me it wouldn’t be like this!” She 
wants to take him to the emergency room for IV furosemide and a pulmonologist visit. 
The best approach is to: 
a) Arrange for a hospice nurse to meet the patient in the emergency room to disenroll him 
from hospice 
b) Set up in home continuous care to manage his catheter 
c) Immediately prepare a respite stay 
d) Admit the patient to a qualified skilled nursing facility for General Inpatient stay for 
pain control 
Discussion: (d) is best 
a) The family needs are symptom management and training about catheter care. Both are 
best provided in a controlled environment, not in the emergency room. This case 
represents a failure in transitions of care, and disenrollment from hospice will only 
lead to one more transition 
b) Continuous care could be a reasonable choice if the issue were only the catheter. The 
requirement for skilled care is the same as for gip and it is a good option for a patient 
who really does not want to be in a facility but in this case the patient’s intensive 
symptom need may need 24 hour nursing care and continuous care is not entirely 
provided by nursing level care 
c) Respite is for the benefit of the family and is generally a planned event, it is also 
appropriate to use in cases of caregiver breakdown when there is not a skilled care 
need requirement for the patient 
d) In this case, the patient has severe dyspnea and pain and is requiring both catheter 
drainage of an effusion and rapid titration of opioids to control his symptoms in a 
manner that his care givers are not capable of providing currently. GIP is to provide 
skilled care for the patient that cannot be provided in the home. Documentation for 
GIP based on pain must include: 
• Frequent evaluation 
• Frequent medication adjustment 
• Aggressive interventions to control the pain 
References: 
• http://www.aahpm.org/apps/blog/?p=1133
From 
Blogs 
to 
Boards: 
AAHPM 
2012 
Pre-­‐Course 
from 
the 
contributors 
of 
Pallimed 
& 
Geripal 
Notes Question HPM19 
Page 24 of 48 
George is admitted to GIP status in a skilled nursing facility with 24 hour RN availability. He 
has had a marked decline since he was seen 2 days ago. The hospice nurse is asking whether 
the plan should be to send him back home after the symptoms are controlled. The social worker 
doesn’t want to bring that up because it might upset the wife and because it might give George 
false hope. The entire Interprofessional group thinks he might die in the next week or two 
You reply that: 
a) Since he’ll likely die in 7-10 days, it will be fine to continue on General Inpatient 
Status for imminently dying criteria so discharge discussions don’t need to be raised 
b) Due to the wife’s burden of caregiver distress, the patient will be maintained on 
General Inpatient Status for caregiver breakdown so discharge discussions don’t need 
to be raised 
c) Once admitted to General Inpatient Status, one of the goals must be transition to a 
lower level of care 
d) Since General Inpatient status should only last 7 days, discharge discussions will start 
after the first 3 days to let the family have some relief. 
Discussion: Correct answer is (c) 
a) There is no GIP status for ‘imminently dying’. There must be some symptom that 
requires management 
b) CMS has clarified that GIP should only be used based on the patient condition and 
should not be used due to caregiver “breakdown”. (CMS Quarterly Provider Update 
April 2007, 
http://www.cms.hhs.gov/quarterlyproviderupdates/downloads/cms1539p.pdf) 
c) Direct wording from quarterly. The goal may not be achievable, but needs to be a part 
of planning and discussion. 
d) There is no specified time limit to GIP status, although some fiscal intermediaries do 
appear to increase audits after the first 7 days 
References: 
• http://digital.ipcprintservices.com/publication/?i=93241 (login required using aahpm 
membership id) 
• Medicare Benefit Policy Manual Chapter 9 - Coverage of Hospice Services Under 
Hospital Insurance 40.1.5 - Short-Term Inpatient Care pp15 and 16/32 
http://www.cms.gov/manuals/downloads/bp102c09.pdf
From 
Blogs 
to 
Boards: 
AAHPM 
2012 
Pre-­‐Course 
from 
the 
contributors 
of 
Pallimed 
& 
Geripal 
Notes Question HPM20 
Page 25 of 48 
Mrs. Tagliatelli is a 76 year old Italian immigrant and widow who has not missed a day of mass 
in her adult life until this past month. She comes to see her primary care physician in clinic 
because she missed mass, asking whether she should get hospice. She has heart failure, mild 
hypertension, and sleep apnea. 
She has noted that over the last month, her legs are more swollen and she is having increased 
difficulty walking to church and the grocery store. She still keeps an impeccable home, 
managing her housecleaning herself, but now is sitting down for a longer period of time after 
carrying the vacuum up and downstairs. She is also able to maintain her daily rituals of reading 
the NYTimes Health and Travel sections, cooking three small meals each day. 
She no longer wishes to return to hospital, and has not been admitted since her myocardial 
infarction 5 years ago, which preceded her diagnosis of heart failure. At that time, she had a 
successful resuscitation and wishes to remain full code. 
She uses CPAP at night for her sleep apnea, but otherwise does not require oxygen. She also 
tells you that because she lives alone, she keeps a gun in her home for self-protection. 
Her home medications include: Furosemide 10mg BID, Atenolol 50mg daily, lisinopril 10mg 
daily, simvastatin 5mg daily, aspirin 81mg daily. She also has nitroglycerine 0.4mg sl prn 
(which she has not used since her MI), and acetaminophen 325mg which she takes “once in a 
while for an ache.” 
Why would this patient not be admitted to hospice? 
a) She is full code. 
b) She lives alone. 
c) She has greater than a six-month prognosis. 
d) She is not homebound. 
e) She has firearms in the home. 
Discussion: answer is (c) 
Take home points: 
• Prognostication for heart failure is one of the more scrutinized and difficult under the 
hospice guidelines. Her current NYHA class is 2, due to increased symptoms with 
activity. In addition, her medical management can still be adjusted, likely with good 
response. General guidelines for heart failure include hospitalization within the last 6- 
months to a year, dyspnea with minimal exertion or at rest. Since she still is 
independent in all ADLs including thorough house-cleaning, without becoming 
dyspneic, it is not reasonable to say she has a <6mo prognosis. 
• While to qualify for visiting nursing services, a patient must be homebound, no such 
requirement exists for hospice. 
• Additionally, patients may choose to be full code on hospice. Medicare does not 
require a 24 hour caregiver to be present in order for the patient to receive hospice 
services. While firearms are concerning for patient and staff safety, they do not 
prevent hospice admission. Some hospices have adopted a stance to request patients to 
have their firearms in locked safe or gun closet. 
References: See Q21
From 
Blogs 
to 
Boards: 
AAHPM 
2012 
Pre-­‐Course 
from 
the 
contributors 
of 
Pallimed 
& 
Geripal 
Notes Question HPM21 
Page 26 of 48 
A couple of years and hospitalizations later, Mrs. Tagliatelli was admitted to hospice. At the 
time of admission to hospice, she was breathless with minimal exertion. Neighbors and 
members of her church visited her often offering her food, company, and rides to church. She 
required oxygen all the time. Even with this, at the time of admission to hospice, she 
experienced constant dyspnea. 
Her cardiac medications were continued, morphine ER and IR were added for her dyspnea. 
After six months on hospice, she is now well palliated, especially since she has been able to 
have her medications as prescribed and no longer spaces out her medications in order to make 
them last. However, she continues to require help from her friends and neighbors, and oxygen 
with minimal activity. She fell once and required a trip to the emergency department. 
You go to see her for recertification visit. 
What do you write in your recertification note? 
a) She meets criteria for recertification because her prognosis remains 6-months or less. 
b) She does not meet criteria for recertification because she has not shown decline in her 
condition. 
c) She does not meet criteria for recertification because her last hospitalization was 
unrelated to her hospice diagnosis 
Discussion (a) is the correct answer 
Take home points: 
• Some intermediaries recommend the demonstration of decline in clinical condition for 
a patient to be recertified, and while this is helpful is the recertification process, it is 
not a CMS requirement. According to Medicare guidelines, the only requirement for 
hospice is that a patient’s prognosis is 6 months or less. 
• Documentation of a hospitalization can also help qualify a patient for hospice. The 
cause of hospitalization does not need to be related to the hospice diagnosis. 
References: 
• https://www.cms.gov/Hospice/Downloads/HospiceFace-to-FaceGuidance.pdf 
• http://www.geripal.org/2011/02/hospice-face-to-face-ftf-encounters-for.html
From 
Blogs 
to 
Boards: 
AAHPM 
2012 
Pre-­‐Course 
from 
the 
contributors 
of 
Pallimed 
& 
Geripal 
Notes Question HPM22a, 22b 
Page 27 of 48 
A young man was recently in a motor vehicle collision where he suffered a massive head injury 
and multi-trauma. He was resuscitated and survived in the ICU with a ventilator, continuous 
hemodialysis, and multiple pressors for the past 2 days, but now is declining, and he is not 
expected to survive this hospitalization. You receive a palliative care consult to help with the 
ventilator withdrawal. You head down to the unit and the nurse comes to you and says “I am 
not sure you should talk with the family – the organ procurement agency has just visited to 
discuss organ donation after cardiac death, and the family want to donate his organs – his liver 
and lungs may be transplantable.” 
What is the best next step? 
a) Thank the nurse, and back out of the consult 
b) Talk with the family about the patient, their grief, and counsel them about comfort care 
after cessation of life-support. 
c) Ask the attending physician of record who is going to manage the patient’s comfort 
care after cessation of life-support. 
d) Work with the family to help them realize this will only prolong the patient’s 
suffering. 
Part B 
A day later, the patient’s HCV test comes back positive and he is no longer a viable DCD 
candidate. The ICU attending asks you to ‘take care of the treatment withdrawal’. The family is 
very disappointed, and indicates their only goal at this point is for a comfortable death, without 
‘prolonging this any longer.’ His only symptom-directed med is intermittent fentanyl bolus 
(700mcg the last 24h). He is unresponsive on the vent, without any spontaneous movement. 
The best next step is to: 
a) Recommend rapidly stopping all life support including CRRT, ventilator, and pressors 
over the next hour or so, and starting a fentanyl and lorazepam infusion to keep the 
patient sedated. 
b) Recommend staggering withdrawal of life support over a couple days including 
stopping CRRT and pressors now in the hopes that the patient dies on the ventilator. 
c) Discuss with the family different approaches to life-support withdrawal. 
d) Switch the patient from fentanyl to morphine boluses as you extubate him, as 
morphine is more effective for air-hunger. 
Discussion: Correct answers are Part A(b) Part B (c) 
Part A Take Home Points 
• Donation after cardiac death is an important, and growing, public good, as it expands 
the pool of potential organ donors: it saves lives. Palliative consultants should support 
DCD programs, and palliative consultation alongside DCD is possible and in some 
institutions the standard of care. For most families, the real concern is not in 
prolonging suffering but the disappointment which can occur if the dying patient 
becomes ineligible to donate organs. DCD practices and the role palliative care 
consultants can play in them are spelled out nicely in this Fast Fact: 
http://www.eperc.mcw.edu/EPERC/FastFactsIndex/ff_242/htm. 
• While C is an important question as the palliative consultant may be asked to manage 
the patient’s comfort care after extubation in the window during which if the patient 
dies he will be able to donate his organs, that is a secondary concern right now to 
doing the good work of palliative care – meeting a patient and family and helping to 
meet their emotional and informational needs. 
Part B Take Home Points 
• A study in 2007 showed that family satisfaction was greater with a stuttered/prolonged
From 
Blogs 
to 
Boards: 
AAHPM 
2012 
Pre-­‐Course 
from 
the 
contributors 
of 
Pallimed 
& 
Geripal 
Page 28 of 48 
approach to discontinuing therapies if the patient had been in the ICU for more than 4 
days, but when the patient was in for 3days or less, a stuttered approach lowered 
satisfaction. Even with those clearly stated goals, careful discussions with the family 
are best about the options for cessation of life-prolonging treatments as families may 
have anticipated concerns about stopping certain treatments (fluids, tube feeds in 
particular). Given the modest evidence of improved satisfaction with staggered 
withdrawal, dicussing with family is the best policy. 
• Answer a) may end up being the best approach; however simply starting a fentanyl 
infusion is secondary to providing/ensuring adequate rapid, boluses of symptom 
medications are available around the time of extubation. 
• No opioid has been established as being superior over another for air hunger. In 
addition, the patient is in renal failure and morphine may accumulate whereas fentanyl 
will not; on the other hand, he is likely to live just a short amount of time making that 
a moot point generally. Morphine is more economical and familiar; on the other hand 
one knows how much fentanyl this patient has needed as a starting point. On top of 
this, this comatose patient probably cannot experience air hunger; the indication for 
opioids really is to palliate labored respirations. All this is to say d is both unnecessary 
but not contraindicated, but not for the reason stated. 
References: 
• http://www.pallimed.org/2008/10/stuttered-treatment-withdrawal-in-icu.html 
• http://cases.pallimed.org/2009/01/coordination-of-care-for-people-at-end.html 
• http://www.pallimed.org/2006/03/terminal-patients-in-icu-and-organ.html 
• Gerstel E, Engelberg RA, Koepsell T, & Curtis JR. (2008) Duration of withdrawal of 
life support in the intensive care unit and association with family satisfaction. 
American journal of respiratory and critical care medicine, 178(8), 798-804. PMID: 
18703787 
• Revelly JP, Imperatori L, Maravic P, Schaller MD, & Chioléro R. (2006) Are 
terminally ill patients dying in the ICU suitable for non-heart beating organ donation?. 
Intensive care medicine, 32(5), 708-12. PMID: 16534569 
• Lynn, J. (2001-2-21) Serving Patients Who May Die Soon and Their Families: The 
Role of Hospice and Other Services. JAMA: The Journal of the American Medical 
Association, 285(7), 925-932. DOI: 10.1001/jama.285.7.925 
• Vent withdrawal FF: http://www.eperc.mcw.edu/EPERC/FastFactsIndex/ff_033.htm 
• http://www.eperc.mcw.edu/EPERC/FastFactsIndex/ff_034.htm 
• DCD FF: http://www.eperc.mcw.edu/EPERC/FastFactsIndex/ff_242/htm 
• Anoxic Brain Injury FF: 
http://www.eperc.mcw.edu/EPERC/FastFactsIndex/ff_234.htm
From 
Blogs 
to 
Boards: 
AAHPM 
2012 
Pre-­‐Course 
from 
the 
contributors 
of 
Pallimed 
& 
Geripal 
Notes Question HPM23 
Page 29 of 48 
A 47 year old woman with a severe, idiopathic, dilated cardiomyopathy is receiving hospice 
care at home. She is ineligible for cardiac transplantation or a ventricular assist device. She has 
mild resting dyspnea but becomes severely dyspneic after just a few steps of ambulation. Her 
nurse measures her resting and ambulatory oxygen saturation while breathing ambient air: it is 
96 and 92%, respectively. The patient is taking digoxin, bumetamide, hydralazine, isosorbide 
dinitrate, albuterol MDI, warfarin, senna, and clonzepam. The patient requests home oxygen 
therapy to help alleviate her breathlessness. 
The best response is: 
a) Order home oxygen therapy for the patient 
b) Initiate lorazepam prn for dyspnea 
c) Recommend use of a hand-held fan and prn morphine for her dyspnea 
d) Request that the patient see her cardiologist for further optimization of her heart failure 
meds 
Discussion: Correct answer is (c) 
a) Home oxygen therapy is not recommended as first-line treatment for dyspnea in non-hypoxic 
patients. It has been shown to be equivalent to ‘sham’ delivery of ambient air 
via nasal cannula. While there is a role for it even in normoxic patients (it ‘works’, just 
no better than ambient air), it is not first-line. 
b) She is already on a benzodiazepine, and benzodiazepines are generally considered 2nd 
line agents to opioids 
c) Hand-held fans have been shown to improve dyspnea, and there is professional 
consensus that opioids are first-line agents for the symptomatic relief of refractory 
dyspnea that is not responding to treatment of the underlying cause. 
d) While there is a role for this strategy, the patient is already on multiple heart failure 
medications which clearly are not sufficient to palliative her dyspnea, and so c is the 
best answer. 
References: 
• http://www.pallimed.org/2010/09/rct-of-oxygen-vs-room-air-delivered-by.html 
• Viola R et al. The management of dyspnea in cancer patients: a systematic review. 
Supp Care Cancer. 2008; 16:329-337. 
• Galbraith S. Does the use of a handheld fan improve chronic dyspnea? A randomized, 
controlled, crossover trial. J Pain Symptom Manage. 2010 May;39(5):831-8.
From 
Blogs 
to 
Boards: 
AAHPM 
2012 
Pre-­‐Course 
from 
the 
contributors 
of 
Pallimed 
& 
Geripal 
Page 30 of 48 
Question HPM24 
Mr. L is a 52-year-old homeless man. One week ago, he was admitted to the ICU with respiratory distress 
and was intubated. A chest CT scan revealed a large necrotic mass filling the right hemithorax, obliterating 
the right and narrowing the left mainstem bronchi. Sputum cytology confirmed a diagnosis of non-small 
cell lung cancer. Oncology states that there is no role for chemotherapy or radiation unless he could be 
weaned off the ventilator, which was considered doubtful in the setting of his airway obstruction. 
Mr. L is unable to participate in medical decision-making. The patient’s mother, who is the authorized 
decision maker, meets with the palliative care team to discuss prognosis and treatment options, including 
withdrawal of life-sustaining treatments. The mother is adamant that all life-sustaining measures be 
continued despite a previous discussion that Mr. L’s disease severity will prevent him from ever leaving the 
ICU, let alone the hospital. Mr. L’s mother expresses hope that, despite the physician’s prediction, a miracle 
will occur that will allow her son to leave the hospital. 
The next best step is to: 
a) Schedule another family meeting to reiterate the prognosis of his current condition and the 
likelihood of recovery 
b) Involve an ethics committee as the mother’s belief in a miracle is far from a societal norm 
c) Tell the mother that hope for a miracle is unreasonable, but that she could still hope that her son is 
comfortable 
d) Ask the mother about her spiritual beliefs and how it influences her decision 
Discussion: Correct answer is (d) 
a) Although it would be reasonable to again discuss prognosis, the mother’s hope in a miracle can be 
considered at least an acknowledgement that she heard the original prognostic information. 
Furthermore, spiritual beliefs, including that of a belief in miracles, may trump a physician’s 
opinion of prognosis despite adequate communication of prognosis.(1) One study of surrogates of 
incapacitated critically ill patients at high risk for death found that only 2% based their views of 
prognosis solely on the physician’s prognostic estimate.(2) Rather, these surrogates used a 
combination of sources including knowledge of the patient’s intrinsic qualities and will to live; 
their observations of the patient; their own observations and beliefs in the power of their support 
and presence, and optimism, intuition, and faith (For 20% of surrogates, a faith in God overrode 
any other source of prognostic information).(1) While scheduling another meeting may be 
appropriate, the ‘problem’ in this scenario is not lack of DATA, and D is the better answer. 
b) According to a 2007 survey performed by the Pew Forum on Religion and Public Life, the majority 
of Americans believe in miracles with little difference based on the respondent’s age. In another 
survey, most public respondents (57.4%) believed that divine intervention from God could save a 
person even if the physician told them ‘‘futility had been reached.’’(3) Significant differences in 
belief in miracles were noted in this study between health care professionals and the general public. 
The majority of public respondents (61.3%) believed that a person in a persistent vegetative state 
could be saved by a miracle, although only a minority of trauma professionals had the same belief. 
c) Without knowing more about the mothers spiritual convictions and having a good understanding of 
what a “miracles means for her”, and reframing of her hope would be premature and could be 
perceived as condescending.(1) 
d) Most individuals would like physicians to ask about their spiritual/religious beliefs.(1) In addition, 
patients who report that their spiritual needs are supported by the medical team are more likely to 
receive hospice care than those who report their spiritual needs were unsupported.(2) 
References: 
• http://www.geripal.org/2011/06/lessons-i-learned-by-examining-miracles.html 
• Widera EW, Rosenfeld KE, Fromme EK, Sulmasy DP, Arnold RM. Approaching patients and 
family members who hope for a miracle. J Pain Symptom Manage. 2011 Jul; 42(1):119-25. 
• Boyd EA, Lo B, Evans LR, et al. ‘‘It’s not just what the doctor tells me:’’ factors that influence 
surrogate decision-makers’ perceptions of prognosis. Crit Care Med 2010;38:1270e1275. 
• Jacobs LM, Burns K, Bennett Jacobs B. Trauma death: views of the public and trauma 
professionals on death and dying from injuries. Arch Surg 2008; 143:730e735. 
• Balboni TA, Paulk ME, Balboni MJ, et al. Provision of spiritual care to patients with advanced 
cancer: associations with medical care and quality of life near death. J Clin Oncol 
2010;28:445e452
From 
Blogs 
to 
Boards: 
AAHPM 
2012 
Pre-­‐Course 
from 
the 
contributors 
of 
Pallimed 
& 
Geripal 
Notes Question HPM25 
Page 31 of 48 
Omar Johnson is a 64 year old man with cryptogenic cirrhosis in multiorgan system failure in 
your hospital’s ICU. He is ventilated, unresponsive, and on vasopressors. You and the ICU 
team agree his chances for surviving this hospitalization are minimal. He has no advance 
directive. 
You participate in an ICU family care conference with his wife (his legal decision maker based 
on state law), 2 sisters, and 3 adult sons. They are told he is dying with minimal chance of 
survival. 
His sons say they do not think the patient would want to die ‘like this – on machines,’ and 
describe several conversations with the patient to support that preference. His wife seems to 
agree with that, but also says, “I can’t give up on him. I can’t have that on my shoulders – I’ll 
always wonder if I did the right thing.” 
The best, next response would be: 
a) Request ethics consultation 
b) Along with the ICU physician, suggest to the family that you make the decision on 
behalf of the patient yourselves, to transition the patient to comfort-care. 
c) Ask the family to focus on what the patient himself would prefer in these 
circumstances. 
d) Express to the family acknowledgment of the emotional difficulty of this, and 
recommend another meeting the next day. 
Discussion: Correct answer is (b) 
a) Be wary of it as a board question answer. Ethics consultations have been shown to 
help with conflict although it wouldn’t be the “best, next response” in this situation. 
b) Making recommendations for what you believe to be the best plan of care is critically 
important in these situations. Surrogate decision making is uniquely traumatizing to 
family members (beyond routine bereavement); when a patient’s preferences are clear, 
physicians should clearly articulate an appropriate plan of care and not force family 
members into feeling they are responsible for a patient’s death. 
c) Generally a good idea – but they already have acknowledged those ‘facts’; the issue 
here is more comfort with decision-making roles and guilt. 
d) A 2nd best option to 2 
References: 
• http://www.pallimed.org/2011/03/trauma-of-surrogacy.html
From 
Blogs 
to 
Boards: 
AAHPM 
2012 
Pre-­‐Course 
from 
the 
contributors 
of 
Pallimed 
& 
Geripal 
Notes Question HPM26 
Mrs. Hassad is a 83 year old retired professor from who is being evaluated for a hospice admission. Her 4 sons live in 
adjacent homes with their families. She has metastatic breast cancer with bone, liver, and brain. Because of her 
underlying renal failure and moderate heart failure, she will not be receiving chemotherapy and her physician had 
arranged home hospice services now that she has completed palliative radiation. She is alert, oriented. 
The hospice intake nurse calls you because the family and patient state that she does not want to know anything about 
his diagnosis or severity of illness. Mrs. Hassad’s son tells the nurse not to speak with the patient about her prognosis, 
her illness, or about code status. Instead, she asks that you speak with her son about these matters. You are at the home 
with the nurse because she does not know how to get her to sign the paperwork to enroll in hospice. 
What do you do after confirming with Mrs. Hassad that she does not want to be involved in signing papers or knowing 
details of his medical condition, and would rather that you speak with her son? 
a) Explain to the son that you must gain consent from Mrs. Hassad in order to enroll her in hospice in respect of 
Page 32 of 48 
the principle of autonomy. 
b) Invoke the health care proxy and have Mrs. Hassad’s son sign the paperwork to enroll in hospice. 
c) Have the son sign the paperwork for hospice since Mrs. Hassad made the autonomous decision to defer 
decisions to her son. 
d) Refuse hospice enrollment for the patient since she is unwilling to accept to address her diagnosis and 
prognosis. 
e) Clarify to the patient that it is her responsibility to make the decision, based on autonomy, and to avoid trauma 
of surrogacy in her son. 
f) Teach the nurse that she should not have questioned the son’s request because that was disrespectful to their 
culture. 
Discussion: (c) is the correct answer 
• Patients can make the autonomous decision to know or not to know information. As outlined in the SPIKES 
protocol for giving bad news, the Invitation is to ask how much a patient wishes to know. Should patients 
choose not to be informed they should know that then consents to procedures and medical care must also be 
deferred to the person they request disclosure to. 
• Healthcare proxy is invoked only when patients lack the capacity to make medical decisions. Here Mrs. 
Hassad has capacity but chooses to have her son make decisions on her behalf. 
• Hospice enrollment does not require a patient’s acceptance of his or her disease and prognosis. 
• Research shows that most people, regardless of culture or country of origin, wish to have medical information 
disclosed to them. However, as age and illness advance, patients are more inclined to request less disclosure. 
One should not assume desire or lack of desire to be involved in medical decision-making and disclosure of 
information based on culture, religion or country of origin. 
References: 
§ http://www.pallimed.org/2011/03/trauma-of-surrogacy.html 
§ Oncotalk. (n.d.). Giving Bad News., http://depts.washington.edu/oncotalk/learn/modules/Modules_02.pdf 
§ Wendler, D., & Rid, A. (2011). Systematic review: the effect on surrogates of making treatment decisions for others. 
Annals of internal medicine, 154(5), 336-46. Retrieved from http://www.annals.org/cgi/content/abstract/154/5/336 
§ Elkin, E. B., Kim, S. H. M., Casper, E. S., Kissane, D. W., & Schrag, D. (2007). Desire for information and 
involvement in treatment decisions: elderly cancer patients’ preferences and their physicians' perceptions. Journal of 
clinical oncology : official journal of the American Society of Clinical Oncology, 25(33), 5275-80. 
§ Baile, W. F. (2000). SPIKES--A Six-Step Protocol for Delivering Bad News: Application to the Patient with Cancer. 
The Oncologist, 5(4), 302-311. 
§ Elkin, E. B., Kim, S. H. M., Casper, E. S., Kissane, D. W., & Schrag, D. (2007). Desire for information and 
involvement in treatment decisions: elderly cancer patients’ preferences and their physicians' perceptions. Journal of 
clinical oncology : official journal of the American Society of Clinical Oncology, 25(33), 5275-80. 
§ Asghari, F., Mirzazadeh, A., & Fotouhi, A. (2008). Patients’ preferences for receiving clinical information and 
participating in decision-making in Iran. Journal of medical ethics, 34(5), 348-52. 
§ Bushnaq, M. (2008). Palliative care in Jordan: culturally sensitive practice. Journal of palliative medicine, 11(10), 
1292-3. Mary Ann Liebert, Inc. 140 Huguenot Street, 3rd Floor New Rochelle, NY 10801-5215 USA.
From 
Blogs 
to 
Boards: 
AAHPM 
2012 
Pre-­‐Course 
from 
the 
contributors 
of 
Pallimed 
& 
Geripal 
Notes Question HPM27 
Page 33 of 48 
Dr. L is a 44 year old palliative care fellow about to complete two months of a busy inpatient 
consult rotation. You notice that over the last week she has become detached and disengaged 
when talking with patients and their family members. The fellow acknowledges feeling tired 
and drained most of the time, as well as having difficulty falling asleep. She also confides in 
you a personal sense of failure and self-doubt. 
The most appropriate interventions at this time is 
a) Recommend she see her primary doctor to discuss SSRI therapy 
b) Recommend she try bright light therapy 
c) Refer for a transient mirrectomy 
d) Recommend an educational program in mindful communication 
Discussion: Correct answer is (d) 
a) The fellow does not meet DSM-IV criteria for depression, although she does have 
some suggestive symptoms. Further exploration would be a correct answer, but 
starting an SSRI would not. 
b) There is no evidence that bright light therapy is helpful for symptoms of burnout. 
There is some evidence of a small benefit for depressive symptoms though. 
c) Transient mirrectomy is a fictional treatment described by Brad Stuart in an April fools 
day GeriPal post. It reportedly is a non-invasive method of numbing brain centers that 
may induce clinicians to identify with pain and suffering to a disabling degree. Sure 
sounds nice. 
d) The fellow has symptoms suggestive of burnout. Burnout encompasses 3 domains: 
feelings of emotional exhaustion, cynicism or depersonalization, and a low sense of 
personal accomplishment. The criterion standard for measuring burnout is the 
Maslach Burnout Inventory (MBI). There is scant high-quality evidence on the 
approach to treating burnout, however one study published in JAMA suggested that 
participation in a mindful communication program was associated with improvements 
in well-being, including burnout. 
References: 
• http://www.geripal.org/2011/04/doctor-develops-cure-for-burnout.html 
• Krasner MS et al. Among Primary Care Physicians Communication With Burnout, 
Empathy, and Attitudes Association of an Educational Program in Mindful. JAMA. 
2009;302(12):1284-1293 
• http://blog.vcu.edu/dpgray/JAMA%20self-care%20end%20of%20life.pdf 
• Kearney MK et al. Self-care of Physicians Care for Patients at the End of Life: “Being 
Connected...A Key to My Survival.” JAMA. 2009;301(11):1155-1164
From 
Blogs 
to 
Boards: 
AAHPM 
2012 
Pre-­‐Course 
from 
the 
contributors 
of 
Pallimed 
& 
Geripal 
Notes Question HPM28 
Page 34 of 48 
Your palliative care clinic team meets Nancy Bush a 46 year-old with newly diagnosed triple-negative 
metastatic breast cancer. She has 7 and 11 year old children. The children know 
Nancy has been ‘to the doctor’ a lot lately but nothing else. 
She is thinking about talking with the children and letting them know her diagnosis, but her 
mother thinks that telling them now will be too hard on them. 
You advise: 
a) It is best to wait until Nancy’s disease is obvious to the children so their interactions 
with their mother will not change. 
b) Telling the children now will make them too anxious. 
c) She should tell the older child, but the younger child is not at an appropriate 
development age that he will benefit from hearing his mother has cancer. 
d) Telling the children of the disease may make them less anxious 
Discussion: Correct answer is (d) 
For the purposes of the boards – your default position should be one of truthful disclosure to 
children of all ages. 
The highest quality longitudinal study of bereaved children showed: 
• The early loss of a parent was associated with poverty, - contributing factors may 
include the loss of income, as well as the burden of medical expenses 
• the increase in separation anxiety symptoms begins prior to death: this speaks to the 
need for preventive interventions when a family death is impending 
• Several studies have shown a higher incidence of substance abuse in bereaved 
children, up to at least 21 months after the death 
• Bereavement pattern associated with early loss of a parent was associated with 
poverty, substance abuse problems, and greater functional impairments. 
References 
• http://cases.pallimed.org/2009/05/what-do-i-say-to-my-kids.html 
• Rosenheim, E., Reicher, R. (1985). Informing children about a parent’s terminal 
illness. J Child Psychol Psychiatry Allied Disc. 26:995-998. 
• Siegel, K., Raveis, V., Karus, D. (1996). Pattern of communication with children when 
a parent has cancer. In L. Baider & L. Cooper (Eds) Cancer and the family, pp 109- 
128. John Wiley and Sons: New York. 
• Psychiatric symptoms in bereaved versus nonbereaved youth and young adults: a 
longitudinal epidemiological study. Kaplow JB. Saunders J. Angold A. Costello EJ. 
• Journal of the American Academy of Child & Adolescent Psychiatry. 49(11):1145-54, 
2010 Nov. 
• Sanchez L, Fristad M, Weller RA, Weller EB, Moye J. Anxiety inacutely bereaved 
prepubertal children. Ann Clin Psychiatry.1994;6:39-43. 
• Swadi H. A longitudinal perspective on adolescent substance abuse. Eur Child Adolesc 
Psychiatry. 1992;1:156-170.
From 
Blogs 
to 
Boards: 
AAHPM 
2012 
Pre-­‐Course 
from 
the 
contributors 
of 
Pallimed 
& 
Geripal 
Notes Question HPM29 
Page 35 of 48 
You receive a call from the hospice nurse about a new hospice patient, Mrs. Gardner, who had a large 
ischemic MCA stroke 4 months ago. She has not been able to eat, is unable to turn herself, and has 
developed a large stage IV decubiti on her low back. 
The wound measures 10cm x 8cm and 1.2cm deep. It has some limited undermining and no tunneling. At 
the wound bed, the spine is visible. The bed of the wound reveals malodorous, necrotic purplish muscle and 
tissue with extensive serosanguinous drainage. The surrounding skin is intact. 
Mr Gardner covers her wound with a cream but notes ‘It just keeps getting deeper.” The patient is turned q2 
hours. The goal of care is to keep her comfortable and at home – a promise he made to her. 
The hospice nurse asks you for orders to help manage the wound. She will order an air-mattress. 
After washing the bed of the wound with normal saline, applying a thin layer of metronidazole gel to the 
base of the wound, what do you recommend for a wound care dressing? 
a) Pack wound with wet-to-dry dressing and cover with ABD pad every 3 days. 
b) Pack wound with calcium alginate wafer and rope, cover with ABD pad every 3 days. 
c) Pack wound with hydrocolloid dressing and cover with ABD pad every 3 days 
Discussion: (b) is correct – non-occlusive, good for wet wounds 
References: 
• May 
macerate 
surrounding 
skin 
Foam 
Dressing 
++++ 
• Can 
be 
used 
in 
infected 
wounds 
Alginate 
dressing 
+++ 
Hydrogel 
++ 
• occlusive, 
should 
not 
use 
with 
venous/vascular 
compromise 
Hydrocolloid 
+ 
Transparent 
Lilm 
• Can 
cause 
debridement 
of 
healing 
tissue 
Gauze 
(wet 
to 
dry) 
Non-­‐ 
adherent 
dressings 
+ 
= 
degree 
of 
absorption 
• Ferris, F., & Pirrello, R. (n.d.). Palliative Wound Care. Retrieved February 25, 2012, from 
google.docs presentation 
• McDonald, A., & Lesage, P. (2006). Palliative management of pressure ulcers and malignant 
wounds in patients with advanced illness. Journal of palliative medicine, 9(2), 285-95. Mary Ann 
Liebert, Inc. 2 Madison Avenue Larchmont, NY 10538 USA.
From 
Blogs 
to 
Boards: 
AAHPM 
2012 
Pre-­‐Course 
from 
the 
contributors 
of 
Pallimed 
& 
Geripal 
Notes Question HPM30 
Page 36 of 48 
A 45 year old man with HIV-AIDS comes to your clinic for follow-up for HIV-related 
neuropathy pain. He has long declined any antiretroviral therapy, and has consistently stated he 
wants supportive-only care focused on maintaining his quality of life. He has a CD4 count of 90 
cells/mm3. 1 year ago it was 100. He reports worsening pain control which he relates to 
inability to swallow his morphine ER tabs (100 mg tid) much of the time. He reports mid-throat 
pain, and frequently chokes on the pills, ‘gags’ them back up. Examination reveals a thin 
man. Mouth demonstrates scattered white plaques on the palate which reveal a red base when 
scraped away. 
Best next step is to: 
a) Prescribe Nystatin ‘swish & swallow’; change morphine to 30mg elixir q4h scheduled. 
b) Prescribe fluconazole; change MorphineER pills to to MorphineER ‘granules’ in 
pudding (such as ‘Kadian’ or ‘Avinza’ morphine formulations). 
c) Prescribe fluconazole, change his morphine to methadone elixir, and recommend 
hospice care given his goals of care and prognosis. 
d) Prescribe Nystatin ‘swish & swallow’; change his morphineER to a fentanyl patch. 
Discussion: Correct answer is (b) 
This man has HIV, AIDS, thrush, and based on the history of dysphagia and odynophagia, 
esophageal candidiasis as well. Due to this, systemic antifungal agents are indicated such as 
fluconazole. Topical agents are ineffective for esophageal candidiasis; for thrush alone they are 
less effective but still used as first-line agents. 
All of the strategies to manage his pain while he is having pill dysphagia are within the realm of 
reason – use of scheduled immediate release morphine, Morphine ER granules which can be 
given in pudding or down a G-tube (‘Kadian’ or ‘Avinza’), or transdermal fentanyl. Methadone 
elixir or crushed pills is a possibility, however it’s a more complicated rotation, and interacts 
with fluconazole, and probably not as elegant as the other solutions. 
Hospice care is not appropriate for the patient based on prognosis. A CD4 count of 90 which is 
slowly declining, and no other major life-limiting complication of HIV, indicate his expected 
prognosis is well over 6 months. Indeed, if he chose to start antiretrovirals it could be decades. 
Hospice eligibility guidelines, while not very evidence-based, suggest a CD4 count 
<25cells/mm3 or a persistent viral load >100,000 copies/ml, as well as a serious HIV related 
comorbidity such as CNS lympthoma, MAC bacteremia untreated or unresponsive to treatment, 
Progressive multifocal leukoencephalopathy, systemic lympthoma, visceral Kaposi’s sarcoma, 
renal failure, cryptosporidium infection, or toxoplasmosis unresponsive to therapy. 
References: 
• http://www.eperc.mcw.edu/EPERC/FastFactsIndex/ff_213.htm 
• http://www.pallimed.org/2006/01/prognosis-in-end-stage-hiv-aids.html 
• http://www.eperc.mcw.edu/EPERC/FastFactsIndex/ff_147.htm
Guide to Palliative Care Board Review
Guide to Palliative Care Board Review
Guide to Palliative Care Board Review
Guide to Palliative Care Board Review
Guide to Palliative Care Board Review
Guide to Palliative Care Board Review
Guide to Palliative Care Board Review
Guide to Palliative Care Board Review
Guide to Palliative Care Board Review
Guide to Palliative Care Board Review
Guide to Palliative Care Board Review
Guide to Palliative Care Board Review

Más contenido relacionado

La actualidad más candente

Motivational Interviewing
Motivational InterviewingMotivational Interviewing
Motivational Interviewingheavensfield
 
Management of schizophrenia
Management of schizophreniaManagement of schizophrenia
Management of schizophreniaYasser Alzainy
 
Current concept for management of neuropathic pain
Current  concept  for management  of  neuropathic painCurrent  concept  for management  of  neuropathic pain
Current concept for management of neuropathic painNeurologyKota
 
Anti-epiliptic drugs
Anti-epiliptic drugsAnti-epiliptic drugs
Anti-epiliptic drugsUmair hanif
 
Brief interventions and motivational enhancement therapy for alcohol problems
Brief interventions and motivational enhancement therapy for alcohol problemsBrief interventions and motivational enhancement therapy for alcohol problems
Brief interventions and motivational enhancement therapy for alcohol problemskavroom
 
Ketamine for depression a walkthrough
Ketamine for depression a walkthroughKetamine for depression a walkthrough
Ketamine for depression a walkthroughKetamine Treatment
 
Cbt for anxiety2
Cbt for anxiety2Cbt for anxiety2
Cbt for anxiety2Pk Doctors
 

La actualidad más candente (12)

Motivational Interviewing
Motivational InterviewingMotivational Interviewing
Motivational Interviewing
 
Management of schizophrenia
Management of schizophreniaManagement of schizophrenia
Management of schizophrenia
 
Current concept for management of neuropathic pain
Current  concept  for management  of  neuropathic painCurrent  concept  for management  of  neuropathic pain
Current concept for management of neuropathic pain
 
Anti-epiliptic drugs
Anti-epiliptic drugsAnti-epiliptic drugs
Anti-epiliptic drugs
 
Motivational interviewing
Motivational interviewingMotivational interviewing
Motivational interviewing
 
Case presentation superfinale
Case presentation superfinaleCase presentation superfinale
Case presentation superfinale
 
MCA 2005 DoLs Briefing
MCA 2005 DoLs BriefingMCA 2005 DoLs Briefing
MCA 2005 DoLs Briefing
 
Tip 35: Motivational Interviewing with Patients with Substance Abuse Issues
Tip 35: Motivational Interviewing with Patients with Substance Abuse IssuesTip 35: Motivational Interviewing with Patients with Substance Abuse Issues
Tip 35: Motivational Interviewing with Patients with Substance Abuse Issues
 
Brief interventions and motivational enhancement therapy for alcohol problems
Brief interventions and motivational enhancement therapy for alcohol problemsBrief interventions and motivational enhancement therapy for alcohol problems
Brief interventions and motivational enhancement therapy for alcohol problems
 
Ketamine for depression a walkthrough
Ketamine for depression a walkthroughKetamine for depression a walkthrough
Ketamine for depression a walkthrough
 
Alcohol Withdrawal
Alcohol WithdrawalAlcohol Withdrawal
Alcohol Withdrawal
 
Cbt for anxiety2
Cbt for anxiety2Cbt for anxiety2
Cbt for anxiety2
 

Destacado

Insomnia in Hospice and Palliative Care
Insomnia in Hospice and Palliative CareInsomnia in Hospice and Palliative Care
Insomnia in Hospice and Palliative CareAndi Chatburn, DO, MA
 
Ewasko Galleries: Interiors
Ewasko Galleries: InteriorsEwasko Galleries: Interiors
Ewasko Galleries: InteriorsTommy Ewasko
 
Formato plano 7th week2_simp_prevsprecont
Formato plano 7th week2_simp_prevsprecontFormato plano 7th week2_simp_prevsprecont
Formato plano 7th week2_simp_prevsprecontEvelin Peña
 
Ipad apps used in pilot program
Ipad apps used in pilot programIpad apps used in pilot program
Ipad apps used in pilot programlnash
 
Take two tweets social media for doctors
Take two tweets social media for doctorsTake two tweets social media for doctors
Take two tweets social media for doctorsChristian Sinclair
 
Formato de clase 4 y 4 general review
Formato de clase 4 y 4 general reviewFormato de clase 4 y 4 general review
Formato de clase 4 y 4 general reviewEvelin Peña
 
2011 03 23 hpm tweetchat transcript
2011 03 23 hpm tweetchat transcript2011 03 23 hpm tweetchat transcript
2011 03 23 hpm tweetchat transcriptChristian Sinclair
 
2011 04 27 hpm tweetchat transcript
2011 04 27 hpm tweetchat transcript2011 04 27 hpm tweetchat transcript
2011 04 27 hpm tweetchat transcriptChristian Sinclair
 
ИС_Омега. Управление проектами ГЧП_2
ИС_Омега. Управление проектами ГЧП_2ИС_Омега. Управление проектами ГЧП_2
ИС_Омега. Управление проектами ГЧП_2Anna Grinenko
 
Social Media In Palliative Care Communities 3 of 3 - Widera
Social Media In Palliative Care Communities 3 of 3 - WideraSocial Media In Palliative Care Communities 3 of 3 - Widera
Social Media In Palliative Care Communities 3 of 3 - WideraChristian Sinclair
 
2011 04 13 hpm tweetchat transcript
2011 04 13 hpm tweetchat transcript2011 04 13 hpm tweetchat transcript
2011 04 13 hpm tweetchat transcriptChristian Sinclair
 
I pad pilot apps used
I pad pilot apps usedI pad pilot apps used
I pad pilot apps usedlnash
 
Sinclair end of-life presentation 1a (1)
Sinclair end of-life presentation 1a (1)Sinclair end of-life presentation 1a (1)
Sinclair end of-life presentation 1a (1)Christian Sinclair
 
Методика сопровождения проектов государственно-частного партнерства
Методика сопровождения проектов государственно-частного партнерстваМетодика сопровождения проектов государственно-частного партнерства
Методика сопровождения проектов государственно-частного партнерстваAnna Grinenko
 
2011 03 16 hpm tweetchat transcript
2011 03 16 hpm tweetchat transcript2011 03 16 hpm tweetchat transcript
2011 03 16 hpm tweetchat transcriptChristian Sinclair
 
Formato de clase 4y 5 simple present
Formato de clase 4y 5 simple presentFormato de clase 4y 5 simple present
Formato de clase 4y 5 simple presentEvelin Peña
 
MDA Retail Projects
MDA Retail ProjectsMDA Retail Projects
MDA Retail ProjectsMDA
 

Destacado (20)

Session 5.4 Moody
Session 5.4 MoodySession 5.4 Moody
Session 5.4 Moody
 
Insomnia in Hospice and Palliative Care
Insomnia in Hospice and Palliative CareInsomnia in Hospice and Palliative Care
Insomnia in Hospice and Palliative Care
 
Ewasko Galleries: Interiors
Ewasko Galleries: InteriorsEwasko Galleries: Interiors
Ewasko Galleries: Interiors
 
Formato plano 7th week2_simp_prevsprecont
Formato plano 7th week2_simp_prevsprecontFormato plano 7th week2_simp_prevsprecont
Formato plano 7th week2_simp_prevsprecont
 
Ipad apps used in pilot program
Ipad apps used in pilot programIpad apps used in pilot program
Ipad apps used in pilot program
 
Take two tweets social media for doctors
Take two tweets social media for doctorsTake two tweets social media for doctors
Take two tweets social media for doctors
 
Flowers by Ewasko
Flowers by EwaskoFlowers by Ewasko
Flowers by Ewasko
 
Formato de clase 4 y 4 general review
Formato de clase 4 y 4 general reviewFormato de clase 4 y 4 general review
Formato de clase 4 y 4 general review
 
2011 03 23 hpm tweetchat transcript
2011 03 23 hpm tweetchat transcript2011 03 23 hpm tweetchat transcript
2011 03 23 hpm tweetchat transcript
 
2011 04 27 hpm tweetchat transcript
2011 04 27 hpm tweetchat transcript2011 04 27 hpm tweetchat transcript
2011 04 27 hpm tweetchat transcript
 
ИС_Омега. Управление проектами ГЧП_2
ИС_Омега. Управление проектами ГЧП_2ИС_Омега. Управление проектами ГЧП_2
ИС_Омега. Управление проектами ГЧП_2
 
Social Media In Palliative Care Communities 3 of 3 - Widera
Social Media In Palliative Care Communities 3 of 3 - WideraSocial Media In Palliative Care Communities 3 of 3 - Widera
Social Media In Palliative Care Communities 3 of 3 - Widera
 
Danielle Chapla
Danielle ChaplaDanielle Chapla
Danielle Chapla
 
2011 04 13 hpm tweetchat transcript
2011 04 13 hpm tweetchat transcript2011 04 13 hpm tweetchat transcript
2011 04 13 hpm tweetchat transcript
 
I pad pilot apps used
I pad pilot apps usedI pad pilot apps used
I pad pilot apps used
 
Sinclair end of-life presentation 1a (1)
Sinclair end of-life presentation 1a (1)Sinclair end of-life presentation 1a (1)
Sinclair end of-life presentation 1a (1)
 
Методика сопровождения проектов государственно-частного партнерства
Методика сопровождения проектов государственно-частного партнерстваМетодика сопровождения проектов государственно-частного партнерства
Методика сопровождения проектов государственно-частного партнерства
 
2011 03 16 hpm tweetchat transcript
2011 03 16 hpm tweetchat transcript2011 03 16 hpm tweetchat transcript
2011 03 16 hpm tweetchat transcript
 
Formato de clase 4y 5 simple present
Formato de clase 4y 5 simple presentFormato de clase 4y 5 simple present
Formato de clase 4y 5 simple present
 
MDA Retail Projects
MDA Retail ProjectsMDA Retail Projects
MDA Retail Projects
 

Similar a Guide to Palliative Care Board Review

Ethical Issues Regarding Nutrition and Hydration in Advanced Illness
Ethical Issues Regarding Nutrition and Hydration in Advanced IllnessEthical Issues Regarding Nutrition and Hydration in Advanced Illness
Ethical Issues Regarding Nutrition and Hydration in Advanced IllnessMike Aref
 
giornate nefrologiche pisane: Quintaliani Presente e futuro della terapia nut...
giornate nefrologiche pisane: Quintaliani Presente e futuro della terapia nut...giornate nefrologiche pisane: Quintaliani Presente e futuro della terapia nut...
giornate nefrologiche pisane: Quintaliani Presente e futuro della terapia nut...Giuseppe Quintaliani
 
Nutrition and Hydration in the Hospice Patient
Nutrition and Hydration in the Hospice PatientNutrition and Hydration in the Hospice Patient
Nutrition and Hydration in the Hospice PatientVITAS Healthcare
 
450 Case Study Esophageal Cancer Treated with Surgery and Radiation
450 Case Study Esophageal Cancer Treated with Surgery and Radiation450 Case Study Esophageal Cancer Treated with Surgery and Radiation
450 Case Study Esophageal Cancer Treated with Surgery and RadiationJonathan Jeffrey
 
Chakras’ Energies Deficiencies as One of the Cause of Kidney Cancer
Chakras’ Energies Deficiencies as One of the Cause of Kidney CancerChakras’ Energies Deficiencies as One of the Cause of Kidney Cancer
Chakras’ Energies Deficiencies as One of the Cause of Kidney Cancerasclepiuspdfs
 
Case study- Endoscopic Gastrojejunostomy
Case study- Endoscopic GastrojejunostomyCase study- Endoscopic Gastrojejunostomy
Case study- Endoscopic Gastrojejunostomykja9641
 
Journal club nutrition in critically ill
Journal club nutrition in critically illJournal club nutrition in critically ill
Journal club nutrition in critically illv3venu
 
Clinical Toxicology by dr.tayyaba rphppt
Clinical Toxicology by dr.tayyaba rphpptClinical Toxicology by dr.tayyaba rphppt
Clinical Toxicology by dr.tayyaba rphpptBIANOOR123
 
Dr Catherine Hayle - Regional ELC - Complex decision making
Dr Catherine Hayle - Regional ELC - Complex decision making Dr Catherine Hayle - Regional ELC - Complex decision making
Dr Catherine Hayle - Regional ELC - Complex decision making Innovation Agency
 
Use of Supplements in the Elderly
Use of Supplements in the ElderlyUse of Supplements in the Elderly
Use of Supplements in the ElderlyMarc Evans Abat
 
Pancreatic Cancer Advances in Treatment Assignment.pdf
Pancreatic Cancer Advances in Treatment Assignment.pdfPancreatic Cancer Advances in Treatment Assignment.pdf
Pancreatic Cancer Advances in Treatment Assignment.pdfbkbk37
 
Palliative Care Boot Camp II
Palliative Care Boot Camp IIPalliative Care Boot Camp II
Palliative Care Boot Camp IIMike Aref
 
Nutrition Care process for Oncology Patients
Nutrition Care process for Oncology PatientsNutrition Care process for Oncology Patients
Nutrition Care process for Oncology PatientsSalmeh Bahmanpour
 
Palliative vs. Hospice Care - READ THIS
Palliative vs. Hospice Care - READ THISPalliative vs. Hospice Care - READ THIS
Palliative vs. Hospice Care - READ THISCynthia Merritt De Vor
 
Surgical Nutrition
Surgical NutritionSurgical Nutrition
Surgical Nutritionbsachs
 
Case presentation [autosaved]
Case presentation [autosaved]Case presentation [autosaved]
Case presentation [autosaved]bkvas
 

Similar a Guide to Palliative Care Board Review (20)

Ethical Issues Regarding Nutrition and Hydration in Advanced Illness
Ethical Issues Regarding Nutrition and Hydration in Advanced IllnessEthical Issues Regarding Nutrition and Hydration in Advanced Illness
Ethical Issues Regarding Nutrition and Hydration in Advanced Illness
 
EBM.ppt
EBM.pptEBM.ppt
EBM.ppt
 
giornate nefrologiche pisane: Quintaliani Presente e futuro della terapia nut...
giornate nefrologiche pisane: Quintaliani Presente e futuro della terapia nut...giornate nefrologiche pisane: Quintaliani Presente e futuro della terapia nut...
giornate nefrologiche pisane: Quintaliani Presente e futuro della terapia nut...
 
Nutrition and Hydration in the Hospice Patient
Nutrition and Hydration in the Hospice PatientNutrition and Hydration in the Hospice Patient
Nutrition and Hydration in the Hospice Patient
 
450 Case Study Esophageal Cancer Treated with Surgery and Radiation
450 Case Study Esophageal Cancer Treated with Surgery and Radiation450 Case Study Esophageal Cancer Treated with Surgery and Radiation
450 Case Study Esophageal Cancer Treated with Surgery and Radiation
 
Chakras’ Energies Deficiencies as One of the Cause of Kidney Cancer
Chakras’ Energies Deficiencies as One of the Cause of Kidney CancerChakras’ Energies Deficiencies as One of the Cause of Kidney Cancer
Chakras’ Energies Deficiencies as One of the Cause of Kidney Cancer
 
Case study- Endoscopic Gastrojejunostomy
Case study- Endoscopic GastrojejunostomyCase study- Endoscopic Gastrojejunostomy
Case study- Endoscopic Gastrojejunostomy
 
Mc qs
Mc qsMc qs
Mc qs
 
Journal club nutrition in critically ill
Journal club nutrition in critically illJournal club nutrition in critically ill
Journal club nutrition in critically ill
 
Clinical Toxicology by dr.tayyaba rphppt
Clinical Toxicology by dr.tayyaba rphpptClinical Toxicology by dr.tayyaba rphppt
Clinical Toxicology by dr.tayyaba rphppt
 
Dr Catherine Hayle - Regional ELC - Complex decision making
Dr Catherine Hayle - Regional ELC - Complex decision making Dr Catherine Hayle - Regional ELC - Complex decision making
Dr Catherine Hayle - Regional ELC - Complex decision making
 
Use of Supplements in the Elderly
Use of Supplements in the ElderlyUse of Supplements in the Elderly
Use of Supplements in the Elderly
 
Pancreatic Cancer Advances in Treatment Assignment.pdf
Pancreatic Cancer Advances in Treatment Assignment.pdfPancreatic Cancer Advances in Treatment Assignment.pdf
Pancreatic Cancer Advances in Treatment Assignment.pdf
 
Oncology ref. 2018
Oncology ref. 2018Oncology ref. 2018
Oncology ref. 2018
 
Palliative Care Boot Camp II
Palliative Care Boot Camp IIPalliative Care Boot Camp II
Palliative Care Boot Camp II
 
Nutrition Care process for Oncology Patients
Nutrition Care process for Oncology PatientsNutrition Care process for Oncology Patients
Nutrition Care process for Oncology Patients
 
Palliative vs. Hospice Care - READ THIS
Palliative vs. Hospice Care - READ THISPalliative vs. Hospice Care - READ THIS
Palliative vs. Hospice Care - READ THIS
 
Palliative vs Hospice Care
Palliative vs Hospice CarePalliative vs Hospice Care
Palliative vs Hospice Care
 
Surgical Nutrition
Surgical NutritionSurgical Nutrition
Surgical Nutrition
 
Case presentation [autosaved]
Case presentation [autosaved]Case presentation [autosaved]
Case presentation [autosaved]
 

Más de Christian Sinclair

Communication and Prognosis 2017
Communication and Prognosis 2017Communication and Prognosis 2017
Communication and Prognosis 2017Christian Sinclair
 
Hospice and Palliative Care Online: From clutter to curation
Hospice and Palliative Care Online: From clutter to curationHospice and Palliative Care Online: From clutter to curation
Hospice and Palliative Care Online: From clutter to curationChristian Sinclair
 
Presentation Skills Workshop - KUMC Fellowship 2014
Presentation Skills Workshop - KUMC Fellowship 2014Presentation Skills Workshop - KUMC Fellowship 2014
Presentation Skills Workshop - KUMC Fellowship 2014Christian Sinclair
 
Before I Die - Advance Care Planning
Before I Die - Advance Care PlanningBefore I Die - Advance Care Planning
Before I Die - Advance Care PlanningChristian Sinclair
 
Radiation Therapy in Palliative Care Spring 2012
Radiation Therapy in Palliative Care Spring 2012Radiation Therapy in Palliative Care Spring 2012
Radiation Therapy in Palliative Care Spring 2012Christian Sinclair
 
Prolonged dying phase aahpm 2012
Prolonged dying phase aahpm 2012Prolonged dying phase aahpm 2012
Prolonged dying phase aahpm 2012Christian Sinclair
 
Prolonged dying phase handouts march 2012
Prolonged dying phase handouts march 2012Prolonged dying phase handouts march 2012
Prolonged dying phase handouts march 2012Christian Sinclair
 
2011 05 11 hpm tweetchat transcript
2011 05 11 hpm tweetchat transcript2011 05 11 hpm tweetchat transcript
2011 05 11 hpm tweetchat transcriptChristian Sinclair
 
2011 05 04 hpm tweetchat transcript
2011 05 04 hpm tweetchat transcript2011 05 04 hpm tweetchat transcript
2011 05 04 hpm tweetchat transcriptChristian Sinclair
 
2011 04 20 hpm tweetchat transcript
2011 04 20 hpm tweetchat transcript2011 04 20 hpm tweetchat transcript
2011 04 20 hpm tweetchat transcriptChristian Sinclair
 
2011 04 06 hpm tweetchat transcript
2011 04 06 hpm tweetchat transcript2011 04 06 hpm tweetchat transcript
2011 04 06 hpm tweetchat transcriptChristian Sinclair
 
2011 03 30 hpm tweetchat transcript
2011 03 30 hpm tweetchat transcript2011 03 30 hpm tweetchat transcript
2011 03 30 hpm tweetchat transcriptChristian Sinclair
 
2011 05 18 hpm tweetchat transcript
2011 05 18 hpm tweetchat transcript2011 05 18 hpm tweetchat transcript
2011 05 18 hpm tweetchat transcriptChristian Sinclair
 
2011 06 22 hpm tweetchat transcript
2011 06 22 hpm tweetchat transcript2011 06 22 hpm tweetchat transcript
2011 06 22 hpm tweetchat transcriptChristian Sinclair
 
2011 06 15 hpm tweetchat transcript
2011 06 15 hpm tweetchat transcript2011 06 15 hpm tweetchat transcript
2011 06 15 hpm tweetchat transcriptChristian Sinclair
 
2011 06 01 hpm tweetchat transcript
2011 06 01 hpm tweetchat transcript2011 06 01 hpm tweetchat transcript
2011 06 01 hpm tweetchat transcriptChristian Sinclair
 
2011 05 25 hpm tweetchat transcript
2011 05 25 hpm tweetchat transcript2011 05 25 hpm tweetchat transcript
2011 05 25 hpm tweetchat transcriptChristian Sinclair
 
2011 06 08 hpm tweetchat transcript
2011 06 08 hpm tweetchat transcript2011 06 08 hpm tweetchat transcript
2011 06 08 hpm tweetchat transcriptChristian Sinclair
 

Más de Christian Sinclair (20)

#PallimedValentines
#PallimedValentines#PallimedValentines
#PallimedValentines
 
Communication and Prognosis 2017
Communication and Prognosis 2017Communication and Prognosis 2017
Communication and Prognosis 2017
 
Hospice and Palliative Care Online: From clutter to curation
Hospice and Palliative Care Online: From clutter to curationHospice and Palliative Care Online: From clutter to curation
Hospice and Palliative Care Online: From clutter to curation
 
Presentation Skills Workshop - KUMC Fellowship 2014
Presentation Skills Workshop - KUMC Fellowship 2014Presentation Skills Workshop - KUMC Fellowship 2014
Presentation Skills Workshop - KUMC Fellowship 2014
 
Before I Die - Advance Care Planning
Before I Die - Advance Care PlanningBefore I Die - Advance Care Planning
Before I Die - Advance Care Planning
 
Social Media = Public Health
Social Media = Public HealthSocial Media = Public Health
Social Media = Public Health
 
Radiation Therapy in Palliative Care Spring 2012
Radiation Therapy in Palliative Care Spring 2012Radiation Therapy in Palliative Care Spring 2012
Radiation Therapy in Palliative Care Spring 2012
 
Prolonged dying phase aahpm 2012
Prolonged dying phase aahpm 2012Prolonged dying phase aahpm 2012
Prolonged dying phase aahpm 2012
 
Prolonged dying phase handouts march 2012
Prolonged dying phase handouts march 2012Prolonged dying phase handouts march 2012
Prolonged dying phase handouts march 2012
 
2011 05 11 hpm tweetchat transcript
2011 05 11 hpm tweetchat transcript2011 05 11 hpm tweetchat transcript
2011 05 11 hpm tweetchat transcript
 
2011 05 04 hpm tweetchat transcript
2011 05 04 hpm tweetchat transcript2011 05 04 hpm tweetchat transcript
2011 05 04 hpm tweetchat transcript
 
2011 04 20 hpm tweetchat transcript
2011 04 20 hpm tweetchat transcript2011 04 20 hpm tweetchat transcript
2011 04 20 hpm tweetchat transcript
 
2011 04 06 hpm tweetchat transcript
2011 04 06 hpm tweetchat transcript2011 04 06 hpm tweetchat transcript
2011 04 06 hpm tweetchat transcript
 
2011 03 30 hpm tweetchat transcript
2011 03 30 hpm tweetchat transcript2011 03 30 hpm tweetchat transcript
2011 03 30 hpm tweetchat transcript
 
2011 05 18 hpm tweetchat transcript
2011 05 18 hpm tweetchat transcript2011 05 18 hpm tweetchat transcript
2011 05 18 hpm tweetchat transcript
 
2011 06 22 hpm tweetchat transcript
2011 06 22 hpm tweetchat transcript2011 06 22 hpm tweetchat transcript
2011 06 22 hpm tweetchat transcript
 
2011 06 15 hpm tweetchat transcript
2011 06 15 hpm tweetchat transcript2011 06 15 hpm tweetchat transcript
2011 06 15 hpm tweetchat transcript
 
2011 06 01 hpm tweetchat transcript
2011 06 01 hpm tweetchat transcript2011 06 01 hpm tweetchat transcript
2011 06 01 hpm tweetchat transcript
 
2011 05 25 hpm tweetchat transcript
2011 05 25 hpm tweetchat transcript2011 05 25 hpm tweetchat transcript
2011 05 25 hpm tweetchat transcript
 
2011 06 08 hpm tweetchat transcript
2011 06 08 hpm tweetchat transcript2011 06 08 hpm tweetchat transcript
2011 06 08 hpm tweetchat transcript
 

Último

Call Girl Koramangala | 7001305949 At Low Cost Cash Payment Booking
Call Girl Koramangala | 7001305949 At Low Cost Cash Payment BookingCall Girl Koramangala | 7001305949 At Low Cost Cash Payment Booking
Call Girl Koramangala | 7001305949 At Low Cost Cash Payment Bookingnarwatsonia7
 
Call Girls Viman Nagar 7001305949 All Area Service COD available Any Time
Call Girls Viman Nagar 7001305949 All Area Service COD available Any TimeCall Girls Viman Nagar 7001305949 All Area Service COD available Any Time
Call Girls Viman Nagar 7001305949 All Area Service COD available Any Timevijaych2041
 
VIP Call Girls Mumbai Arpita 9910780858 Independent Escort Service Mumbai
VIP Call Girls Mumbai Arpita 9910780858 Independent Escort Service MumbaiVIP Call Girls Mumbai Arpita 9910780858 Independent Escort Service Mumbai
VIP Call Girls Mumbai Arpita 9910780858 Independent Escort Service Mumbaisonalikaur4
 
Call Girls Hebbal Just Call 7001305949 Top Class Call Girl Service Available
Call Girls Hebbal Just Call 7001305949 Top Class Call Girl Service AvailableCall Girls Hebbal Just Call 7001305949 Top Class Call Girl Service Available
Call Girls Hebbal Just Call 7001305949 Top Class Call Girl Service Availablenarwatsonia7
 
Call Girls Thane Just Call 9910780858 Get High Class Call Girls Service
Call Girls Thane Just Call 9910780858 Get High Class Call Girls ServiceCall Girls Thane Just Call 9910780858 Get High Class Call Girls Service
Call Girls Thane Just Call 9910780858 Get High Class Call Girls Servicesonalikaur4
 
Call Girls In Andheri East Call 9920874524 Book Hot And Sexy Girls
Call Girls In Andheri East Call 9920874524 Book Hot And Sexy GirlsCall Girls In Andheri East Call 9920874524 Book Hot And Sexy Girls
Call Girls In Andheri East Call 9920874524 Book Hot And Sexy Girlsnehamumbai
 
call girls in Connaught Place DELHI 🔝 >༒9540349809 🔝 genuine Escort Service ...
call girls in Connaught Place  DELHI 🔝 >༒9540349809 🔝 genuine Escort Service ...call girls in Connaught Place  DELHI 🔝 >༒9540349809 🔝 genuine Escort Service ...
call girls in Connaught Place DELHI 🔝 >༒9540349809 🔝 genuine Escort Service ...saminamagar
 
Call Girl Lucknow Mallika 7001305949 Independent Escort Service Lucknow
Call Girl Lucknow Mallika 7001305949 Independent Escort Service LucknowCall Girl Lucknow Mallika 7001305949 Independent Escort Service Lucknow
Call Girl Lucknow Mallika 7001305949 Independent Escort Service Lucknownarwatsonia7
 
97111 47426 Call Girls In Delhi MUNIRKAA
97111 47426 Call Girls In Delhi MUNIRKAA97111 47426 Call Girls In Delhi MUNIRKAA
97111 47426 Call Girls In Delhi MUNIRKAAjennyeacort
 
Low Rate Call Girls Mumbai Suman 9910780858 Independent Escort Service Mumbai
Low Rate Call Girls Mumbai Suman 9910780858 Independent Escort Service MumbaiLow Rate Call Girls Mumbai Suman 9910780858 Independent Escort Service Mumbai
Low Rate Call Girls Mumbai Suman 9910780858 Independent Escort Service Mumbaisonalikaur4
 
See the 2,456 pharmacies on the National E-Pharmacy Platform
See the 2,456 pharmacies on the National E-Pharmacy PlatformSee the 2,456 pharmacies on the National E-Pharmacy Platform
See the 2,456 pharmacies on the National E-Pharmacy PlatformKweku Zurek
 
Call Girls Frazer Town Just Call 7001305949 Top Class Call Girl Service Avail...
Call Girls Frazer Town Just Call 7001305949 Top Class Call Girl Service Avail...Call Girls Frazer Town Just Call 7001305949 Top Class Call Girl Service Avail...
Call Girls Frazer Town Just Call 7001305949 Top Class Call Girl Service Avail...narwatsonia7
 
Russian Call Girl Brookfield - 7001305949 Escorts Service 50% Off with Cash O...
Russian Call Girl Brookfield - 7001305949 Escorts Service 50% Off with Cash O...Russian Call Girl Brookfield - 7001305949 Escorts Service 50% Off with Cash O...
Russian Call Girl Brookfield - 7001305949 Escorts Service 50% Off with Cash O...narwatsonia7
 
Noida Sector 135 Call Girls ( 9873940964 ) Book Hot And Sexy Girls In A Few C...
Noida Sector 135 Call Girls ( 9873940964 ) Book Hot And Sexy Girls In A Few C...Noida Sector 135 Call Girls ( 9873940964 ) Book Hot And Sexy Girls In A Few C...
Noida Sector 135 Call Girls ( 9873940964 ) Book Hot And Sexy Girls In A Few C...rajnisinghkjn
 
Russian Call Girls Chickpet - 7001305949 Booking and charges genuine rate for...
Russian Call Girls Chickpet - 7001305949 Booking and charges genuine rate for...Russian Call Girls Chickpet - 7001305949 Booking and charges genuine rate for...
Russian Call Girls Chickpet - 7001305949 Booking and charges genuine rate for...narwatsonia7
 
Call Girls Service Nandiambakkam | 7001305949 At Low Cost Cash Payment Booking
Call Girls Service Nandiambakkam | 7001305949 At Low Cost Cash Payment BookingCall Girls Service Nandiambakkam | 7001305949 At Low Cost Cash Payment Booking
Call Girls Service Nandiambakkam | 7001305949 At Low Cost Cash Payment BookingNehru place Escorts
 
call girls in munirka DELHI 🔝 >༒9540349809 🔝 genuine Escort Service 🔝✔️✔️
call girls in munirka  DELHI 🔝 >༒9540349809 🔝 genuine Escort Service 🔝✔️✔️call girls in munirka  DELHI 🔝 >༒9540349809 🔝 genuine Escort Service 🔝✔️✔️
call girls in munirka DELHI 🔝 >༒9540349809 🔝 genuine Escort Service 🔝✔️✔️saminamagar
 
College Call Girls Vyasarpadi Whatsapp 7001305949 Independent Escort Service
College Call Girls Vyasarpadi Whatsapp 7001305949 Independent Escort ServiceCollege Call Girls Vyasarpadi Whatsapp 7001305949 Independent Escort Service
College Call Girls Vyasarpadi Whatsapp 7001305949 Independent Escort ServiceNehru place Escorts
 
Call Girls Service Chennai Jiya 7001305949 Independent Escort Service Chennai
Call Girls Service Chennai Jiya 7001305949 Independent Escort Service ChennaiCall Girls Service Chennai Jiya 7001305949 Independent Escort Service Chennai
Call Girls Service Chennai Jiya 7001305949 Independent Escort Service ChennaiNehru place Escorts
 
Call Girls Kanakapura Road Just Call 7001305949 Top Class Call Girl Service A...
Call Girls Kanakapura Road Just Call 7001305949 Top Class Call Girl Service A...Call Girls Kanakapura Road Just Call 7001305949 Top Class Call Girl Service A...
Call Girls Kanakapura Road Just Call 7001305949 Top Class Call Girl Service A...narwatsonia7
 

Último (20)

Call Girl Koramangala | 7001305949 At Low Cost Cash Payment Booking
Call Girl Koramangala | 7001305949 At Low Cost Cash Payment BookingCall Girl Koramangala | 7001305949 At Low Cost Cash Payment Booking
Call Girl Koramangala | 7001305949 At Low Cost Cash Payment Booking
 
Call Girls Viman Nagar 7001305949 All Area Service COD available Any Time
Call Girls Viman Nagar 7001305949 All Area Service COD available Any TimeCall Girls Viman Nagar 7001305949 All Area Service COD available Any Time
Call Girls Viman Nagar 7001305949 All Area Service COD available Any Time
 
VIP Call Girls Mumbai Arpita 9910780858 Independent Escort Service Mumbai
VIP Call Girls Mumbai Arpita 9910780858 Independent Escort Service MumbaiVIP Call Girls Mumbai Arpita 9910780858 Independent Escort Service Mumbai
VIP Call Girls Mumbai Arpita 9910780858 Independent Escort Service Mumbai
 
Call Girls Hebbal Just Call 7001305949 Top Class Call Girl Service Available
Call Girls Hebbal Just Call 7001305949 Top Class Call Girl Service AvailableCall Girls Hebbal Just Call 7001305949 Top Class Call Girl Service Available
Call Girls Hebbal Just Call 7001305949 Top Class Call Girl Service Available
 
Call Girls Thane Just Call 9910780858 Get High Class Call Girls Service
Call Girls Thane Just Call 9910780858 Get High Class Call Girls ServiceCall Girls Thane Just Call 9910780858 Get High Class Call Girls Service
Call Girls Thane Just Call 9910780858 Get High Class Call Girls Service
 
Call Girls In Andheri East Call 9920874524 Book Hot And Sexy Girls
Call Girls In Andheri East Call 9920874524 Book Hot And Sexy GirlsCall Girls In Andheri East Call 9920874524 Book Hot And Sexy Girls
Call Girls In Andheri East Call 9920874524 Book Hot And Sexy Girls
 
call girls in Connaught Place DELHI 🔝 >༒9540349809 🔝 genuine Escort Service ...
call girls in Connaught Place  DELHI 🔝 >༒9540349809 🔝 genuine Escort Service ...call girls in Connaught Place  DELHI 🔝 >༒9540349809 🔝 genuine Escort Service ...
call girls in Connaught Place DELHI 🔝 >༒9540349809 🔝 genuine Escort Service ...
 
Call Girl Lucknow Mallika 7001305949 Independent Escort Service Lucknow
Call Girl Lucknow Mallika 7001305949 Independent Escort Service LucknowCall Girl Lucknow Mallika 7001305949 Independent Escort Service Lucknow
Call Girl Lucknow Mallika 7001305949 Independent Escort Service Lucknow
 
97111 47426 Call Girls In Delhi MUNIRKAA
97111 47426 Call Girls In Delhi MUNIRKAA97111 47426 Call Girls In Delhi MUNIRKAA
97111 47426 Call Girls In Delhi MUNIRKAA
 
Low Rate Call Girls Mumbai Suman 9910780858 Independent Escort Service Mumbai
Low Rate Call Girls Mumbai Suman 9910780858 Independent Escort Service MumbaiLow Rate Call Girls Mumbai Suman 9910780858 Independent Escort Service Mumbai
Low Rate Call Girls Mumbai Suman 9910780858 Independent Escort Service Mumbai
 
See the 2,456 pharmacies on the National E-Pharmacy Platform
See the 2,456 pharmacies on the National E-Pharmacy PlatformSee the 2,456 pharmacies on the National E-Pharmacy Platform
See the 2,456 pharmacies on the National E-Pharmacy Platform
 
Call Girls Frazer Town Just Call 7001305949 Top Class Call Girl Service Avail...
Call Girls Frazer Town Just Call 7001305949 Top Class Call Girl Service Avail...Call Girls Frazer Town Just Call 7001305949 Top Class Call Girl Service Avail...
Call Girls Frazer Town Just Call 7001305949 Top Class Call Girl Service Avail...
 
Russian Call Girl Brookfield - 7001305949 Escorts Service 50% Off with Cash O...
Russian Call Girl Brookfield - 7001305949 Escorts Service 50% Off with Cash O...Russian Call Girl Brookfield - 7001305949 Escorts Service 50% Off with Cash O...
Russian Call Girl Brookfield - 7001305949 Escorts Service 50% Off with Cash O...
 
Noida Sector 135 Call Girls ( 9873940964 ) Book Hot And Sexy Girls In A Few C...
Noida Sector 135 Call Girls ( 9873940964 ) Book Hot And Sexy Girls In A Few C...Noida Sector 135 Call Girls ( 9873940964 ) Book Hot And Sexy Girls In A Few C...
Noida Sector 135 Call Girls ( 9873940964 ) Book Hot And Sexy Girls In A Few C...
 
Russian Call Girls Chickpet - 7001305949 Booking and charges genuine rate for...
Russian Call Girls Chickpet - 7001305949 Booking and charges genuine rate for...Russian Call Girls Chickpet - 7001305949 Booking and charges genuine rate for...
Russian Call Girls Chickpet - 7001305949 Booking and charges genuine rate for...
 
Call Girls Service Nandiambakkam | 7001305949 At Low Cost Cash Payment Booking
Call Girls Service Nandiambakkam | 7001305949 At Low Cost Cash Payment BookingCall Girls Service Nandiambakkam | 7001305949 At Low Cost Cash Payment Booking
Call Girls Service Nandiambakkam | 7001305949 At Low Cost Cash Payment Booking
 
call girls in munirka DELHI 🔝 >༒9540349809 🔝 genuine Escort Service 🔝✔️✔️
call girls in munirka  DELHI 🔝 >༒9540349809 🔝 genuine Escort Service 🔝✔️✔️call girls in munirka  DELHI 🔝 >༒9540349809 🔝 genuine Escort Service 🔝✔️✔️
call girls in munirka DELHI 🔝 >༒9540349809 🔝 genuine Escort Service 🔝✔️✔️
 
College Call Girls Vyasarpadi Whatsapp 7001305949 Independent Escort Service
College Call Girls Vyasarpadi Whatsapp 7001305949 Independent Escort ServiceCollege Call Girls Vyasarpadi Whatsapp 7001305949 Independent Escort Service
College Call Girls Vyasarpadi Whatsapp 7001305949 Independent Escort Service
 
Call Girls Service Chennai Jiya 7001305949 Independent Escort Service Chennai
Call Girls Service Chennai Jiya 7001305949 Independent Escort Service ChennaiCall Girls Service Chennai Jiya 7001305949 Independent Escort Service Chennai
Call Girls Service Chennai Jiya 7001305949 Independent Escort Service Chennai
 
Call Girls Kanakapura Road Just Call 7001305949 Top Class Call Girl Service A...
Call Girls Kanakapura Road Just Call 7001305949 Top Class Call Girl Service A...Call Girls Kanakapura Road Just Call 7001305949 Top Class Call Girl Service A...
Call Girls Kanakapura Road Just Call 7001305949 Top Class Call Girl Service A...
 

Guide to Palliative Care Board Review

  • 1. From Blogs to Boards: AAHPM 2012 Pre-­‐Course from the contributors of Pallimed & Geripal From Blogs to Boards: Answer Key Editors: • Suzana Makowski • Drew Rosielle • Paul Tatum • Eric Widera • Christian Sinclair Page 1 of 48
  • 2. From Blogs to Boards: AAHPM 2012 Pre-­‐Course from the contributors of Pallimed & Geripal From Blogs to Boards Goals of this session: § Prepare for the boards with an audience response quizz hosted by Pallimed & Geripal bloggers § Review recent, clinically relevant literature on Palliative Care § Discover three methods to obtain quality up-to-date information on palliative care Board Review Content: Page 2 of 48 Board Content 8% 7% 6% 45% 11% 9% 5% 9% Approach to Care Psychosocial & Spiritual Concerns Impending Death Grief & Bereavement Medical Management Communication & Teamwork Ethical & Legal Decision-Making Prognostication
  • 3. From Blogs to Boards: AAHPM 2012 Pre-­‐Course from the contributors of Pallimed & Geripal Notes Question HPM1 Page 3 of 48 Ms. V is a 68 year old with metastatic non-small cell lung cancer, congestive heart failure, and mild renal insufficiency residing in an inpatient palliative care unit for management of bone pain. Her medications include morphine IR, fentanyl transdermal patch, furosemide, senna, and Fleet enema’s prn. Ms. V did not have a bowel movement in 4 days. Basic labs were ordered for the next morning as well as a two of her prn enemas, although they failed to result in a bowel movement. The labs the next day reveal a serum sodium of 124, potassium of 3.0, creatinine of 1.4 (baseline of 1), low calcium of 6.5, and a very elevated phosphate of 17 mg/dl. What is the most likely cause of her electrolyte abnormalities? a) A medication adverse event b) Tumor lysis syndrome c) Bowel Impaction d) Osteolytic metastases Discussion: Correct answer is (a) a) Sodium phosphate preparations should never be given to patients with renal insufficiency, heart failure, cirrhosis, or elderly frail individuals due to significant risks of adverse effect. Both oral and rectal sodium phosphate preparations can cause significant fluid shifts within the colon resulting in intravascular volume depletion. Furthermore, these preparations can cause electrolyte disturbances including significant hyperphosphatemia, hypocalcemia, and hypokalemia. A significant clinically important rise in serum phosphate can even be seen in elderly patients with normal renal function. (J Gastroenterol Hepatol. 2004;19(1):68). Lastly, phosphate nephropathy may occur due to the transient and potentially severe increase in serum phosphate combined with volume depletion from the fluid shifts. b) Tumor lysis may indeed cause hyperphosphatemia and hypocalcemia, although it is generally seen in with cytotoxic therapy in patients with a large tumor burden with rapid cell turnover (ie. Non-Hodgkins Lymphoma or certain leukemias). It is also associated with hyperkalemia. c) Bowel impaction alone should not cause these electrolyte disturbances d) Osteolytic metastases generally cause hypercalcemia. References: • http://www.geripal.org/2012/02/dangers-of-fleet-enemas.html
  • 4. From Blogs to Boards: AAHPM 2012 Pre-­‐Course from the contributors of Pallimed & Geripal Notes Question HPM2 Page 4 of 48 Walking into a room at your hospice inpatient unit you see a tired appearing female patient lying in bed with soft moaning, holding her abdomen. She has end stage CHF and no history of cancer. Review of your notes show decreasing oral intake and increased time in bed. Her nurse reports she disimpacted her yesterday after suppositories and enemas were ineffective for worsening constipation. Medications include: Fentanyl 50mcg patch (on for several weeks), Senna 2 tabs BID, Colace daily, Recent enema, and docusate suppository Exam: Cachectic female, Scaphoid abdomen, hypoactive bowel sounds, formed (but not hard) stool on rectal exam. What is the next best step? a) Write an order for methylnaltrexone 8mg subcutaneously x1 now. b) Switch her from a fentanyl patch to a morphine pump so you can better manage her abdominal pain. c) Write an order for octreotide 200mcg subcutaneously twice daily for three days d) Place an NG and give her polyethylene glycol daily until she has a bowel movement or regains ability to swallow and you can remove the NG tube Discussion: Correct answer is (a) a) The patient likely has opioid induced constipation (OIC). Methylnaltrexone is a mu-opioid receptor antagonist and is related to naloxone. After ruling out bowel obstruction, fecal impaction and any other abdominal process, you give methylnaltrexone at 0.15mg/kg subcutaneously, usually 8 (patients < 136lbs) or 12 mg (patients over 136lbs). About 60 percent of patients will have a BM in under 4 hours. Usually within 30 minutes of the first dose. Number needed to treat was 2.2 (pretty darn good). One barrier is cost. At $48 per 8mg dose this is a costly way to manage constipation. b) While controlling abdominal pain is important relieving the cause of the abdominal pain takes precedence. Opioids may be the cause of her pain – increasing them is not indicated. With the exception of imminently dying patients, proper treatment of OIC will lead to its resolution and function can be improved. c) Octreotide has a role in palliative care for malignant bowel obstruction (MBO), not constipation. This patient does not have a cancer history and sudden onset nausea and vomiting that may be signs for a MBO. Octreotide also is expensive-costing between $40 and $80 per dose. d) Placing a nasogastric tube should be avoided whenever possible when there are less invasive measures available. The patient can swallow oral laxatives, and does not have an MBO and so does not have any minimal indications for an NGT in any case. Polyethylene glycol is helpful as an osmotic laxative and is often employed as a first line option for OIC. It is often more helpful as part of a maintenance regimen or for mild to moderate constipation. References: • Thomas, Jay et. al. Methylnaltrexone for Opioid Induced Constipation in Advanced Illness. 2008. NEJM 358 (22): 2332-2343. • Yuan, Chun-Su. Methylnaltrexone Mechanisms of Action and Effects on Opioid Bowel Dysfuction and Other Opioid Adverse Side Effects. The Annals of Pharmacotherapy, 2007. 41: 984- 993
  • 5. From Blogs to Boards: AAHPM 2012 Pre-­‐Course from the contributors of Pallimed & Geripal Notes Question HPM3 Page 5 of 48 During a hospice interdisciplinary team meeting, you hear about a 53 year old resident of the local nursing home. He has ALS with bulbar attributes, and is starting to have difficulty swallowing and speaking. He is bedbound most of the day. He has had two episodes of aspiration pneumonia in the last month. His nurse describes the scene with the patient’s wife, Sally, at his side, squeezing his hand with one hand and her rosary with the other. He explained to the nurse, “I told Sally that I don’t want a feeding tube. I’ve had a good life and have few regrets. I saw my father-in-law die on a feeding tube and I would not want to go through that, or put my wife through that. But I am Catholic. Our friend at the parish said that I have to ‘do everything’ to prolong my life – especially when it comes to nutrition - or I will go hell. I don’t want to go to hell.” His wife nods emphatically. During the interdisciplinary care meeting, the chaplain (in his role as teacher) asks you to explain to the team what your understanding of the Catholic doctrine is as pertaining to this patient. What do you say? a) My understanding is that medically assisted nutrition is obligatory for patients who are unable to take food by mouth. b) My understanding is that medically assisted nutrition is morally optional for most patients at the end of life. Discussion: Correct answer is (b) a) Some interpret the teachings of the church to mandate artificial nutrition at the end of life, especially with the media coverage of Terri Schiavo. But the doctrine is more nuanced than that. “58. In principle, there is an obligation to provide patients with food and water, including medically assisted nutrition and hydration for those who cannot take food orally. This obligation extends to patients in chronic and presumably irreversible conditions (e.g., the “persistent vegetative state”) who can reasonably be expected to live indefinitely if given such care.” (from: section 58. Ethical and Religious Directives for Catholic Health Care Services.) b) The discussion is actually more complex then that: Medically-assisted nutrition and hydration become morally optional when they cannot reasonably be expected to prolong life or when they would be “excessively burdensome for the patient or [would] cause significant physical discomfort, for example resulting from complications in the use of the means employed. “59. The free and informed judgment made by a competent adult patient concerning the use or withdrawal of life-sustaining procedures should always be respected and normally complied with, unless it is contrary to Catholic moral teaching.” (from: section 59. Ethical and Religious Directives for Catholic Health Care Services.) References: http://www.pallimed.org/2010/01/catholic-directives-on-artificial.html http://www.pallimed.org/2008/08/media-coverage-of-terri-schiavo.html
  • 6. From Blogs to Boards: AAHPM 2012 Pre-­‐Course from the contributors of Pallimed & Geripal Notes Question HPM4 Page 6 of 48 Mrs Dole, a 68 year old with 20 year history of Diabetes Mellitus Type II is referred to Palliative Care from Oncology with Stage III Nasopharyngeal carcinoma. Nausea is the key concern. For last 3 years she has had early satiety but maintained weight. Since initiating chemotherapy, she has had nausea for the first 2 days of her chemotherapy cycle, which then resolves. 1 week after the last round of chemotherapy she required intravenous fluids for dehydration. Now 2 weeks later is having intermittent severe nausea. It can be provoked by sudden changes in body position. She fell once because she lost her balance. Usually she does not vomit, but occasionally does. She describes a feeling of the room spinning associated with the nausea. Of the following options, which drug is most targeted to this patient’s specific nausea type: a) Ondansetron b) Prochlorperazine c) Metoclopramide d) Diazepam e) Meclizine Discussion: (e) are the correct answers This patient has had multiple types of nausea, however currently her major nausea type seems to be vestibular. She may have developed an otolith while dehydrated. Some chemotherapeutic agents are ototoxic and can cause vestibular symptoms including hearing loss, tinnitus, vertigo/nausea. She also has had chemotherapy induced nausea, as well as diabetic gastroparsis. For the boards, probably the default choice for nausea will be D2 blockers, however there are certain types of nausea for which D2 blockers are not the best choice. a) Ondansetron and the other ‘-setrons’ are HT3 receptor blockers and have excellent evidence for the treatment of chemotherapy induced nausea, and post-operative nausea. While used widely for other types of nausea including opioid-associated, there is less evidence to support them for these practices. They are exceedingly safe and well-tolerated; they are constipating. *** Chemotherapy induced nausea/vomiting is considered acute when it occurs <24h after chemo infusion, and delayed if >24h. Delayed n/v usually occurs in the several days after chemotherapy, but not weeks. First line treatments to prevent acute CINV including 5HT3 blockers and steroids. NK-1 blockers such as aprepitant and gluclocorticoids are also used, especially for mod-highly emetogenic chemo. NK-1 blockers and steroids also prevent delayed N/V; 5HT3 blockers less so. D2 blockers are no longer first line agents as 5HT3 blockers have clearly shown superior efficacy and safety. Doses of metoclopramide needed to be effective are 1-2mg/kg IV! b) Prochlorperazine and other D2 blockers such as haloperidol target the Chemoreceptor trigger zone and D2 receptor. They are the work-horses of nausea treatment. c) While the patient has some component of diabetic gastroparesis suggested by satiety and long history of DM, he is not bothered by emesis with meals. Metoclopramide targets D2 receptors primarily in the gut, and has some prokinetic features, but its role long-term for gastroparesis is controversial as it causes EPS such as tardive dyskinesia. d) Diazepam and benzodiazepines are effective for anticipatory nausea/vomiting which occurs in ~25% of chemo patients. Behavorial/cognitive treatments, and integrative modalities are probably helpful too. Aggressive prevention of CINV can help prevent anticipatory n/v. e) She has what seems to be vestibular symptoms. Anticholinergic drugs such as meclizine, scopolamine, promethazine, and even diphenhydramine are potential drugs. CNS side effects such as sedation, confusion; as well as orthostatis and xerostomia are worrisome side effects. References: • http://www.pallimed.org/2007/09/vatican-tube-feeding-more-on-abigail.html • http://jama.ama-assn.org/content/298/10/1196.full.pdf+html • Hain TC, Uddin M. Pharmacological treatment of vertigo. CNS Drugs. 2003;17:85–100. • http://www.oncolink.org/resources/article.cfm?c=16&s=59&ss=224&id=1004
  • 7. From Blogs to Boards: AAHPM 2012 Pre-­‐Course from the contributors of Pallimed & Geripal Notes Question HPM5 Page 7 of 48 In hospice IDT, you discuss the case of a 68 year old female with ovarian cancer with abdominal pain and sudden onset nausea and vomiting. She has had no recent bowel movements and is on minimal opioids. You suggest a trial of octreotide for a likely malignant bowel obstruction and the nurses say “Doctor! You say we can use octreotide for everything! Is there anything octreotide can’t be used for in hospice?” Which one of the following is not a potential scenario to use octreotide? Choose the best answer. Answers a) A 37 year old male with end stage alcoholic hepatitis who starts vomiting blood b) A 90 year old with a severe diarrhea with a history of a rectal tumor and radiation burns to the perineal area c) A 42 year old female with a tense distended abdomen leaking a small amount from a previous paracentesis site. d) A 27 year old male with a malignant wound with copious drainage e) A 31 year old female with abdominal pain from opioid-induced constipation Discussion: Answer is (e) a) Octreotide is the Swiss Army Knife of palliative medications. It is a synthetic analog of somatostatin and has many mechanisms of action: in general, it has a global effect to decrease secretions primarily in the GI tract It can be costly as a medication alone but it could reduce the system cost by avoiding hospitalizations. You should talk with your local pharmacist to see about availability and cost in your local programs. It is typically administered via intermittent subcutaneous dosing. Study published in 2000 compared octreotide infusion with sclerotherapy and found that octreotide to be as effective as sclerotherapy regarding hemostasis at 48 hours and on day 7 after the index bleeding episode. So for the patient looking to avoid hospitalization with acute variceal bleed this may be a helpful (but expensive) medication. b) While it does not work as a prophylactic treatment to prevent chemo and radiation induced diarrhea a few studies have shown that it can treat existing diarrhea related to these two common cancer treatments. c) Rapidly accumulating ascites or situations where repeat paracentesis or drain may not be readily available have been shown to be responsive to octreotide. It also has been reported for use in pleural effusions related to cirrhosis. d) Tumor related secretions have been show to respond to octreotide e) Indications for octreotide include (via palliativedrugs.com) : symptoms associated with unresectable hormone-secreting tumors, e.g. carcinoid, VIPomas, glucagonomas and acromegaly; prevention of complications after elective pancreatic surgery; †bleeding esophageal varices; †salivary, pancreatic and enterocutaneous fistulas; †intractable diarrhea related to high output ileostomies, AIDS, radiotherapy, chemotherapy or bone marrow transplant;†inoperable bowel obstruction in patients with cancer; †hypertrophic pulmonary osteo-arthopathy;†ascites in cirrhosis and cancer; †buccal fistula; †death rattle (noisy respiratory secretions); †bronchorrhea;†reduction of tumor-related secretions. References: § http://cases.pallimed.org/2008/06/am-i-really-going-to-have-to-live-like.html § http://www.pallimed.org/2008/11/octreotide-for-radiation-induced.html § Freitas DS, Sofia C, Pontes JM, Gregório C, Cabral JP, Andrade P, Rosa A, Camacho E, Ferreira M, Portela F.... (2000) Octreotide in acute bleeding esophageal varices: a prospective randomized study. Hepato-gastroenterology, 47(35), 1310-4. PMID: 11100339 § Kalambokis, G. (2006-01) Octreotide in the treatment of refractory ascites of cirrhosis. Scandinavian Journal of Gastroenterology, 14(1), 199-121. DOI: 10.1080/00365520510024043 M § Martenson et al. The efficacy of octreotide in the therapy of acute radiation-induced diarrhea: a randomized controlled study. International Journal of Radiation OncologyBiologyPhysics, 54(1), 195-202. DOI: 10.1016/S0360-3016(02)02870-5
  • 8. From Blogs to Boards: AAHPM 2012 Pre-­‐Course from the contributors of Pallimed & Geripal Notes Question HPM6 Page 8 of 48 You visit a patient at home receiving hospice care for cancer. Her pain has been well controlled with long acting morphine 60mg BID and occasional PRN doses of short acting liquid morphine (10mg) over the past few weeks: she had been tolerating this well. She has had recent progressive functional decline and is currently at a PPS of 20%. In the last 24 hours the patient has vomited and has been more lethargic and having difficulty swallowing pills. She appears uncomfortable. In your examination you see a very thin patient who appears to be dying with a prognosis in the few days to a week range. The patient’s son is a respiratory therapist at a hospital and is insisting you change the patient’s opioid to a fentanyl patch because “it is less sedating than morphine.” The best response is: a) Because the patient is cachectic, you tell the family that fentanyl transdermal patches are not indicated because the medication will not be absorbed. b) Agree with the son and convert the patient to a 37.5mcg/hr fentanyl patch with oral morphine liquid 10mg q1 hour PRN c) Because the fentanyl will not be effective for over 24 hours, continue the long acting morphine sulfate 60mg BID but give it rectally instead of by mouth d) Suggest starting a morphine infusion via her port at 1.7mg/hr basal with a 3mg q30min bolus PRN after talking with the son about his concerns about sedation. Discussion: Answer is (d) a) Cachexia has not been show to be a CLINICALLY RELEVALANT factor in absorption of transdermal fentanyl. Cachexia will decrease the amount of subcutaneous fat which is where fentanyl is stored AFTER absorption through the dermal layers. In 2009 Heiskanen did a study comparing blood levels between cachectic and non-cachectic volunteers and found no significant difference, although cachectic patients had a slightly lower mean concentration. There was no difference in VAS score. b) Fentanyl is not less sedating than morphine at equianalgesic doses. Also there is no 37.5mcg/hr patch or 12.5mcg/hr patch. As written, and described by the manufacturer, the “12.5mcg/hr patch” is labeled and Rx’d as a “12mcg/hr” patch to prevent confusion with Rx’ing 125mcg/hr. As for the conversion, it could be acceptable to use a 25mcg/hr & 12mcg/hr patch (total 37mcg/hr) per the Fentanyl transdermal product insert. It recommends 25mcg/hr for someone on OMDD of 60-134mg and 50mcg/hr for someone on OMDD 135- 224, so this is right in the middle. The Breitbart/Donner conversion of 2mg morphine = 1mcg/hr transdermal fentanyl which would be 60mcg/hr of fentanyl (You could choose 50 or 75 depending on other clinical circumstances). c) The pharmacokinetics of fentanyl do not warrant switching to it if otherwise indicated. Morphine still has time to circulate and get out of her system, and fentanyl begins to reach significant blood concentrations 8-12 hours after application. If needed, she can be bridged with a few doses of liquid morphine. In addition, people do not prefer rectal administration if it could be avoided. d) A morphine continuous infusion allows for the continuation of the current effective opioid in a patient who is likely not going to regain swallowing function. The conversion is most direct (120mg OMDD = 40mg daily IV = 1.7mg/hr (1.5 if your pumps are limited in decimal rates). A 3 mg IV morphine bolus most closely replicates the 10mg oral morphine doses that were effective prior. If you did not choose this answer because your hospice doesn’t use continuous infusions (expense, nurse familiarity, not available from local pharmacy) then start talking with your hospice to decrease these barriers to an effective and essential tool to good pain management. References: • http://www.pallimed.org/2009/05/cachexia-and-absorption-of-transdermal.html • Heiskanen, Tarja. (2009-7) Transdermal fentanyl in cachectic cancer patients. PAIN, 70(1-2), 928-222. DOI: 10.1016/j.pain.2009.04.012 • Mercadante, Sebastiano. (2012-01-09) Sustained-release oral morphine versus transdermal fentanyl and oral methadone in cancer pain management. European Journal of Pain, 7(Suppl.
  • 9. From Blogs to Boards: AAHPM 2012 Pre-­‐Course from the contributors of Pallimed & Geripal Page 9 of 48 A), 320-1046. DOI: 10.1016/j.ejpain.2008.01.013 • Weissman DE. Converting to/from Transdermal Fentanyl, 2nd Edition. Fast Facts and Concepts. July 2005; 2. Available at: http://www.eperc.mcw.edu/fastfact/ff_002.htm. • Tatum IV WO. (2002) Adult patient perceptions of emergency rectal medications for refractory seizures. Epilepsy & behavior : E&B, 3(6), 535-538. PMID: 12609248 • Colbert SA, O'Hanlon D, McAnena O, & Flynn N. (1998) The attitudes of patients and health care personnel to rectal drug administration following day case surgery. European journal of anaesthesiology, 15(4), 422-6. PMID: 9699099 • Mercadante, Sebastiano. (2012-01-09) Sustained-release oral morphine versus transdermal fentanyl and oral methadone in cancer pain management. European Journal of Pain, 7(Suppl. A), 320-1046. DOI: 10.1016/j.ejpain.2008.01.013
  • 10. From Blogs to Boards: AAHPM 2012 Pre-­‐Course from the contributors of Pallimed & Geripal Notes Question HPM7 Page 10 of 48 JY, a 28 year old woman with advanced cystic fibrosis and Burkholderia cenocepacia colonization is hospitalized for a cystic fibrosis exacerbation. She has chronic chest wall pain from coughing and pleurisy, and recently broke 2 ribs from coughing. She is on IV glucocorticoids, IV ketorolac, IV ketamine prior to vest treatments, and lorazepam. Prior to her hospitalization, she took oxycodone ER 30mg q12h. Currently she is on a hydromorphone IV PCA at 2mg/hour, with 2mg q30 minute boluses. She used 72mg of IV dilaudid in the last 24h. Despite this she is becoming drowsy, and reports her pain is minimally improved and still severe for most of the day: 7-8/10, and ‘nearly intolerable’ during vest therapy The best next step is to: a) Increase her PCA basal and ‘bolus’ doses by 50% and monitor for 24 hours. b) Add a 5% lidocaine patch to her chest wall over her rib fractures c) Discontinue hydromorphone and switch the patient to another opioid d) Advise the primary team to stop vest therapies Discussion: Correct answer (c) a) Indications for opioid rotation are 1) dose-limiting side effects such as sedation, nausea, pruritus, myoclonus from the patient’s current opioid, 2) need for a new dosing route (patient cannot swallow), 3) costs/insurance changes, 4) inadequate analgesia despite ‘adequate’ dose-escalation of the current opioid. There is no consensus on what 4 actually means, however rapidly escalating someone by an order of magnitude (as in this case) without good response, is generally a scenario in which you’d consider rotation (if not long before). Is not best next step given the above discussion b) No data at all suggesting the lidocaine patch is effective for pain from fractures c) Is the correct answer: Morphine, methadone, or fentanyl are all reasonable options. Some prefer methadone in these sorts of settings, but no actual data to support that and probably not tested on the boards. Another reasonable approach in this situation would be to consult a pain interventionalist for regional options. d) Opioid rotation is reasonable first, before advising this, as it will likely affect the patient’s ability to recover. References: • http://www.pallimed.org/2008/07/methadone-methadone-methadone.html • http://www.pallimed.org/2010/01/outpatient-rotations-to-methadone.html • http://www.pallimed.org/2005/07/transdermal-fentanyl-to-methadone.html
  • 11. From Blogs to Boards: AAHPM 2012 Pre-­‐Course from the contributors of Pallimed & Geripal Notes Question HPM8 Page 11 of 48 Mr. Smith is a 72 year old patient was admitted to hospital from his nursing home for respiratory distress due to CHF exacerbation. Despite aggressive diuresis attempts, his respiratory distress continued and his urine output remained minimal (~30ml/day). PMH: heart failure, moderate dementia, renal insufficiency Home medications: furosemide 40mg po bid, metoprolol 25mg bid, donepezil 10mg daily, olanzapine 5mg qhs. After a conversation with his son (health care proxy) the patient was "made CMO" (comfort measures only) by the hospitalist service and resident team two days ago. He was then started on a morphine drip “titrate by 1mg as needed for pain or shortness of breath”, his donepezil, olanzapine and diuretics continued, other medications stopped. His intern calls in a panic: “We promised to make him comfortable, that he would die in 2 days, but he is still alive and the family does not know why he is in such pain – even with light touch – crying out & jerking.” What is your recommendation? a) Stop morphine drip and start fentanyl and lorazepam prn b) Increase morphine and olanzapine c) Increase morphine and add lorazepam prn d) Stop morphine drip and start fentanyl, increase olanzapine Discussion: Correct answer (a) Key points: • Opioid neurotoxicity in the setting of renal failure/azotemia is the most likely answer. Morphine metabolites build up disproportionately in the setting of renal failure. Morphine 3-glucoronide is a neurostimulant that can lead to agitated delirium, myoclonus, hyperalgesia, and even seizures. Morphine and hydromorphone are the most common culprits. Morphine 6-glucoronide is a metabolite that is active on the mu-opioid receptor, and thus is not a major player in terms of inducing agitated neurotoxicity. • Fentanyl does not have the same metabolites and thus has a lower risk of agitated neurotoxicity. Since there are no active metabolites that build up in renal failure, it is the safest of the “pure” opioids for patients on dialysis or who are oliguric. Methadone is another opioid that is nearly ~100% excreted in the stool • The treatment for this is to rotate off current opioid. Fentanyl is safer option in renal failure. • Antipsychotics can worsen the symptoms • Benzodiazepines can help treat myoclonus and prevent seizures References: Robin K Wilson, David E Weissman; Neuroexcitatory effects of opioids: patient assessment, 2nd ed. EPERC# 057 http://www.eperc.mcw.edu/EPERC/FastFactsIndex/ff_057.htm http://www.aahpm.org/apps/blog/?tag=boards Smith, H. S. (2009). Opioid metabolism. Mayo Clinic proceedings. Mayo Clinic, 84(7), 613-24.
  • 12. From Blogs to Boards: AAHPM 2012 Pre-­‐Course from the contributors of Pallimed & Geripal Notes Question HPM9 Page 12 of 48 BJ, a 65 yo woman with known non-small cell lung cancer, metastatic to her mediastinum, contralateral lung, and supraclavicular lymph nodes, returns to your clinic for follow-up for her cancer-related pain. She is getting chemotherapy, and has always expressed a desire for ‘the most aggressive’ treatments available for her cancer. She complains of 2 weeks of worsening, midline low back pain. She has noticed difficulty in rising from chairs/toilet, and needed a wheelchair to make it into the clinic area today from the parking garage due to weakness. Examination is notable for an unremarkable back/spine exam, and 4/5 strength bilaterally in her lower extremities both proximally and distally. You obtain a stat MRI which shows a T12 vertebral metastasis and cord compression. In addition to administering glucocorticoids, then next best step is to: a) Arrange an urgent radiation oncology consultation for the next day. b) Admit her to the hospital, and arrange a stat radiation oncology consultation. c) Admit her to the hospital, and arrange a stat spine surgery consultation. d) Adjust her pain medications appropriately, and instruct her to contact you immediately if her pain or disability worsens Discussion: Correct answer is (c) This is a medical emergency. • Vertebral metastases, putting a patient at risk for cord compression, should be considered in any patient with new back pain and cancer. New or otherwise suspicious back-pain can be evaluated urgently with a non-contrast MRI of the entire spine. • If patients have neurologic symptoms of LE weakness and/or bladder, bowel dysfunction, it is a medical emergency and patients needs stat imaging, steroids, and intervention. Neurologic deficits, once present, can rapidly progress to permanent paraplegia within 24h. • The role of steroids + XRT vs steroids + surgery is unclear. A recent trial indicated better outcomes with immediate surgery, especially for patients who came in with severe weakness. 84% of patients vs 54% were ambulatory after treatment course with surgery vs radiation without surgery. Actual practice has not necessarily caught up with this, and will depend on local, institutional resources. References: • http://www.pallimed.org/2005/08/surgery-better-than-radiation-steroids.html • http://www.pallimed.org/2008/03/spinal-cord-compression-copd-prognosis.html • http://www.eperc.mcw.edu/EPERC/FastFactsIndex/ff_237.htm • http://www.eperc.mcw.edu/EPERC/FastFactsIndex/ff_238.htm
  • 13. From Blogs to Boards: AAHPM 2012 Pre-­‐Course from the contributors of Pallimed & Geripal Notes Question HPM10 Page 13 of 48 Mr. G. Da Salva is a 68 year old construction worker who has metastatic non-small cell lung cancer involving his right femur and pelvis. Medications include: Morphine ER 200mg bid, Morphine IR 30-60mg PO q2 hours prn, and dexamethasone 8mg daily. At rest his pain is well managed, 2/10. However, he fears movement due to severe pain and spends most of the day in his recliner, avoiding showering or changing or helping with the meals. He uses approximately 5 doses daily of 60mg short-acting morphine for this pain but once it starts to work the pain has often spontaneously subsided and he becomes sleepy and confused. Which of the following is LEAST appropriate? a) Take a short-acting morphine prior to a clustering his activities: showering, changing, fixing a meal. b) Add sublingual fentanyl 200mcg to take prior to his activities. c) Increase his long-acting morphine to 200mg tid. d) Single-fraction radiation therapy to his pelvis and femur. e) Intrathecal pump with morphine and low-dose bupivacaine. Discussion: Correct answer is (c) This is an example of incidental pain. It differs from breakthrough pain in that it is associated with movement, and diminishes as soon as the activity ends. The challenge with this form of pain management is that the pharmacology of systemic opioids does not tend to match the timing of this type of pain. a) Clustering his activities together so that they all take longer may better match the t1/2 of the short-acting morphine, but he will still need to take the medicine approximately 40 minutes prior to starting the activities. b) Sublingual or buccal fentanyl has a shorter half-life and shorter time to onset than other oral opioids and is a better option. c) Is the correct answer: Increasing the long-acting morphine is the least appropriate because this will not help the incidental pain and may worsen his confusion when he is not moving around. d) Single fraction radiation therapy would be very appropriate in this setting and would likely be one of the most preferred interventions as long as he had not previously been irradiated at the site of pain. e) Intrathecal pain medication delivery is another good option. However, this is an expensive procedure and requires a prognosis of at least 3 months to assure coverage by insurance plans. Because the dose of opioid is a fraction of systemic opioid delivery, its risk of side-effects is lower and is a more effective means of managing incidental pain in the lower back and lower extremities. References: • http://www.aahpm.org/apps/blog/?p=809 • http://www.pallimed.org/2009/12/poll-results-palliative-care-experience.html • Bruera, E., & Kim, H. N. (2003). Cancer pain. JAMA : the journal of the American Medical Association, 290(18), 2476-9. • Chow, E., Harris, K., Fan, G., Tsao, M., & Sze, W. M. (2007). Palliative radiotherapy trials for bone metastases: a systematic review. Journal of clinical oncology : official journal of the American Society of Clinical Oncology, 25(11), 1423-36. • Smith, T. J., Swainey, C., & Coyne, P. J. (2005). Pain management, including intrathecal pumps. Current Pain and Headache Reports, 9(4), 243-248. Current Medicine Group LLC.
  • 14. From Blogs to Boards: AAHPM 2012 Pre-­‐Course from the contributors of Pallimed & Geripal Notes Question HPM11 Page 14 of 48 Mr. Z is a 87 year old with advanced dementia living in a nursing home. At baseline he cannot recognize family members, is dependent on all ADLs (dressing, toileting, bathing) but does not have urinary or fecal incontinence. He speaks about 1-2 intelligible words per day and he has had progressive loss of ability to ambulate. He is now admitted to the hospital after sustaining a hip fracture from a fall. When discussing treatment options for his hip fracture, his wife asks you how long he likely has to live. Given his current state of health, what would be the most appropriate answer: a) Given that he does not meet FAST 7C criteria his prognosis is likely greater than 6 months b) He meets NHPCO Guidelines for hospice eligibility which means he likely has less than a 6 month prognosis c) Given his advanced dementia and recent hip fracture, his 6 month mortality risk exceeds 50% d) As with most individuals with advanced dementia, his life expectancy is likely weeks to months Discussion: Correct answer is (c) References: a) The FAST scale measures functional status in dementia and consists of 7 major stages split into 16 different sub-stages. Hospice eligibility criteria for dementia are based largely on whether a patient meets or exceeds Stage 7c on the FAST and whether they have at least one complication from their dementia. Unfortunately, these criteria do not accurately predict 6-month survival. b) The current National Hospice and Palliative Care Organization (NHPCO) guidelines for hospice eligibility are of limited accuracy in predicting death within 6 months. In addition, NHPCO guidelines relies on the FAST staging, which fails to account for the observation that dementia often does not progress in a sequential pattern. The patient is dependent on ADLs (dressing, toileting, bathing) but does not have urinary or fecal incontinence (FAST Stage 6d and 6e). His speech has declined from less than 6 intelligible words per day (7a) to one or less (7b), and he has had progressive loss of ability to ambulate (7C), however since he does not have 6d and 6e, Mr. Z is not considered Fast Stage 7c, rather he is Fast 6C. c) Is the correct answer: Individuals with advanced dementia that are either hospitalized for either pneumonia or for hip fracture have a very poor prognosis. In one study, six-month mortality for patients with end-stage dementia and hip fracture was 55% compared with 12% for cognitively intact patients. d) Advanced dementia is a terminal condition; however estimating prognosis is difficult due to the prolonged period of severe functional and cognitive impairment that occurs prior to death. For those with advanced disease who reside in a nursing home, the 6-month mortality rate is 25% with a median survival in one study of only 478 days. References: § http://www.geripal.org/2009/10/there-is-important-article-in-current.html § www.eprognosis.org § Mitchell SL, Miller SC, Teno JM, Kiely DK, Davis RB, Shaffer ML. Prediction of 6-month survival of nursing home residents with advanced dementia using ADEPT vs hospice eligibility guidelines. JAMA. Nov 3 2010;304(17):1929-1935. § Morrison RS, Siu AL. Survival in end-stage dementia following acute illness. Jama. Jul 5 2000;284(1):47-52. § Mitchell SL, Teno JM, Kiely DK, et al. The clinical course of advanced dementia. N Engl J Med. Oct 15 2009;361(16):1529-1538.
  • 15. From Blogs to Boards: AAHPM 2012 Pre-­‐Course from the contributors of Pallimed & Geripal Notes Question HPM12 Page 15 of 48 Mrs. A is an 88 year old with advanced dementia who lives in a nursing home. She has at baseline some difficulty with eating as she pockets food in her mouth and occasionally coughs after swallowing. She is now hospitalized for an aspiration pneumonia. In addition to the antibiotics she is on in the hospital, her only other medications include HCTZ for hypertension and a baby aspirin. She has never taken a cholinesterase inhibitor . What is the best next step? a) A trial of both a cholinesterase inhibitors and memantine b) Feeding tube insertion c) Careful hand feeding and good oral care d) Addition of olanzapine to treat her pocketing of food behavior Discussion: Correct answer is (c) a) Acetylcholinesterase inhibitors, such as donepezil, galantamine, and rivastigmine, and the N-methyl-D-aspartate (NMDA) antagonist memantine have some evidence for a statistically significant improvement in cognitive, functional, and behavioral outcomes in indivudals with moderate-to-severe dementia. However, these improvements have marginal clinical significance. Adverse events are common with these agents, most commonly nausea, vomiting, and diarrhea. There is no evidence to suggest that they decrease eating problems or risk for aspirations. b) Placement of PEG tubes often occur after transfer to an acute care facility for eating problems or pneumonia, despite the fact that feeding tubes have not been shown to improve survival for individuals with dementia. There is also no evidence that tube feeding prevents aspiration pneumonia, decreases the risk for pressure ulcers, improves patient comfort, or prolongs life. c) Oral care has been shown to decrease incidence of pneumonia, number of febrile days, and death from pneumonia in nursing home residents. d) Antipsychotics have not been shown to improve eating behaviors in dementia. There is a moderate short-term efficacy when treating agitation, serious side effects that include risk of stroke and death limit their clinical use. References: • http://www.geripal.org/2011/07/decisions-on-feeding-tubes-in-advanced.html • Finucane TE, Christmas C, Travis K. Tube feeding in patients with advanced dementia: a review of the evidence. JAMA. Oct 13 1999;282(14):1365-1370. • Yoneyama T, Yoshida M, Ohrui T, et al. Oral care reduces pneumonia in older patients in nursing homes. J Am Geriatr Soc. Mar 2002;50(3):430-433. • Schneider LS, Tariot PN, Dagerman KS, et al. Effectiveness of atypical antipsychotic drugs in patients with Alzheimer's disease. N Engl J Med. Oct 12 2006;355(15):1525- 1538
  • 16. From Blogs to Boards: AAHPM 2012 Pre-­‐Course from the contributors of Pallimed & Geripal Notes Question HPM12 – Part 2 Page 16 of 48 The family is concerned that Mrs. A’s aspirations will continue if she continues to be fed by hand in the nursing home. They would like to know about more about the risks of a feeding tube placement. The most appropriate risk to include in the discussion is: a) She will have a 1 in 10 chance of a major surgical complication in the perioperative period. b) She is unlikely to have a tube related complication after the perioperative period c) Once the tube is placed, it would be technically difficult to electively remove the tube d) She will have a 1 in 3 chance of requiring chemical or physical restraints to prevent tube removal Discussion: Correct answer is (d) a) Up to one-third experience transient gastrointestinal adverse effects (ie, vomiting, diarrhea) but major complications like bowel perforations are rare (1%) b) Tube dislodgement, blockage, and leakage are common (4%-11%). One in 5 tube-fed residents experiencing a tube related complication necessitating a hospital transfer in the year following insertion. c) Removal of feeding tubes is not technically difficult. An important pearl to know is that if a feeding tube is inadvertently removed, the stoma site will close in a few hours, so put in a Foley catheter to keep it open until a new one can be placed. d) Over a quarter of tube fed dementia patients are physically restrained after feeding tube placement, and nearly a third are placed on sedating medications to prevent them from pulling out the feeding tube. References: • http://www.geripal.org/2011/07/decisions-on-feeding-tubes-in-advanced.html • Teno JM et al. Decision-making and outcomes of feeding tube insertion: a five-state study. J Am Geriatr Soc. 2011 May;59(5):881-6
  • 17. From Blogs to Boards: AAHPM 2012 Pre-­‐Course from the contributors of Pallimed & Geripal Notes Question HPM13 Page 17 of 48 A 54 yo man with a 7 month history of metastatic bladder cancer presents to the cancer center’s palliative care clinic. He complains of low mood, anhedonia, feelings of guilt, shame, and worthlessness most days for the last 2 months. He says, “Of course I’m depressed – who wouldn’t be? I’ve got a cancer that the doctors tell me is terminal. What good am I to my family? They’d be better off without me.” The best next step would be to: a) Tell the patient that he is depressed and recommend a treatment plan for it. b) Ask your team’s social worker to see the patient for grief counseling. c) Provide emotional support and counseling with the patient that what he is experiencing is part of the expected adjustment to having a terminal illness. d) Refer the patient to psychiatry for complicated depression. Discussion: Correct answer is (a) a) Psychiatric disorders including depression and anxiety disorders occur in only a minority of patients at the end of life. Published numbers have ranged from 10-40%; while higher than the general population, they are not the norm. Persistent low mood, feelings of worthlessness, guilt, and shame are highly suggestive of depression. The patient should be counseled about this and offered treatment b) While this may end up being appropriate for this patient, his symptomotology is most c/w depression and a) is the better answer. c) See (a). This is not ‘normal.’ d) Complicated depression is not a diagnosis. He has untreated depression, and HPM specialists should be able to initiate appropriate therapy! References: • Rayner L et al. Antidepressants for the treatment of depression in palliative care: systematic review and metaanalysis. Palliat Med. 25(1):36-51. DOI: 10.1177/0269216310380764 • http://www.pallimed.org/2011/02/what-i-learned-at-aahpmhpna-annual.html • Irwin SA. Oral ketamine for the rapid treatment of depression and anxiety in patients receiving hospice care. JPM. 2010; 13:903-8. http://www.ncbi.nlm.nih.gov/pubmed/20636166
  • 18. From Blogs to Boards: AAHPM 2012 Pre-­‐Course from the contributors of Pallimed & Geripal Notes Question HPM14 Page 18 of 48 The patient agrees to pharmacologic therapy for his depression, and declines offers of counseling/therapy. Your best estimate is that he has 4-8 weeks to live based on performance status and tempo of decline. Which of the following are appropriate drug approaches for his depression? a) Methylphenidate b) Ketamine c) Dronabinol d) Sertraline Discussion: Correct answer are (a) and (b) a) Is correct: Methylphenidate and other psychostimulants are rapidly-acting, with onset of mood elevation occurring ~immediately (if they are going to be effective at all). b) Is correct: Ketamine has been described as a depression therapy for decades, albeit one on the margins of accepted medical practice; there is a recent resurgence in interest for its use at life’s end, because its effects are immediate. Patients usually receive a single infusion, which stabilizes mood for weeks at a time. Can be used orally too. c) Dronabinol has no defined role as an antidepressant. d) Most SSRIs take at least 4 weeks to become effective, which is 50-100% of the patient’s anticipated survival. While SSRIs are first-line antidepressants in the general population as well as for patients with advanced illness, this is not as true as prognosis shortens. References: • Rayner L et al. Antidepressants for the treatment of depression in palliative care: systematic review and metaanalysis. Palliat Med. 25(1):36-51. DOI: 10.1177/0269216310380764 • http://www.pallimed.org/2011/02/what-i-learned-at-aahpmhpna-annual.html • Irwin SA. Oral ketamine for the rapid treatment of depression and anxiety in patients receiving hospice care. JPM. 2010; 13:903-8. http://www.ncbi.nlm.nih.gov/pubmed/20636166
  • 19. From Blogs to Boards: AAHPM 2012 Pre-­‐Course from the contributors of Pallimed & Geripal Notes Antidepressant Pop Quiz Page 19 of 48 I say depression, insomnia, anorexia, nausea – You say: a) Trazodone b) Paroxetine c) Mirtazipine d) Escitalopram I say depression, anxiety, insomnia, neuropathy, You say: a) nortriptyline b) duloxetine c) fluoxetine d) venlafaxine I say activating antidepressants, You say a) fluoxetine b) paroxetine c) buproprion d) citalopram I say depression, anxiety, neuropathic pain, advanced age, You say: a) duloxetine b) nortriptyline c) paroxetine d) mirtazapine Discussion: Correct answers in order are: (c) – (a) – (a, c) – (a) • Mirtazapine has side effects which include drowsiness and weight gain, plus some antiemetic effects • TCAs are the only drug class that directly treat all those symptoms (SNRIs – serotonin norepinephrine reuptake inhibitors - like duloxetine and venlafaxine aren’t known to be directly effective for insomnia). • Paroxetine can be sedating; citalopram more neutral • Duloxetine seems to be better tolerated than TCAs, especially in the elderly. For the boards, would avoid giving elderly TCAs. Mirtazapine’s role in pain is not well defined, especially compared to SNRIs and TCAs.
  • 20. From Blogs to Boards: AAHPM 2012 Pre-­‐Course from the contributors of Pallimed & Geripal Notes Question HPM15 Page 20 of 48 Mrs. Phillips is a 91-year-old hospitalized patient who is now actively dying due to end-stage pulmonary fibrosis and asbestosis. She has been well palliated during the last several months at home where she lived independently, until she developed a pneumonia and was hospitalized. Her home medications had not been adjusted in over six weeks. This included: albuterol and atropine nebulizers, dexamethasone 2mg every morning, 25mcg/hour fentanyl patch for dyspnea, oxycodone concentrate (20mg/ml) 10mg q2 hours prn dyspnea or pain, senna and Colace. She is on day 7 of oral antibiotics for presumed pneumonia. She is on oxygen 6 liters via nasal cannula. Her last bowel movement was yesterday, and her urine output has been good (250ml or more daily.) Yesterday she was still oriented, between periods of increasing fatigue and sleep. She showed signs of mottling and new secretions causing respiratory rattle. A scopolamine patch 1.5mg was started for her increased secretions. You are called by the resident who explains to you that this morning Mrs. Phillips is now agitated, moaning, and even thrashing at times. This is causing family and floor nurses distress. He asks you for advice. Which of the following is appropriate? a) Stop scopolamine b) Start lorazepam c) Increase the fentanyl d) Stop the fentanyl e) Counsel family about the inevitability terminal delirium f) Order soft restraints Discussion: (a) is correct Delirium is a common condition at the end-of-life. It often is considered “terminal” even if reversible, however. Terminal delirium should be considered a diagnosis of exclusion or even one made in hindsight. While conducting a battery of exhaustive tests to evaluate the cause is not usually appropriate or necessary, causes of delirium should be addressed if possible. The most common causes of delirium in this setting remain constipation, urinary retention, medications, infection, electrolyte abnormalities. Constipation, urinary retention can be ruled out in this patient. a) With this patient, the addition of scopolamine is the most likely cause. This is a tertiary amine anticholinergic agent, and commonly causes confusion in the elderly. b) Lorazepam is not the best option for delirium; neuroleptics, in addition to treating the underlying cause (if feasible) are appropriate. c) Fentanyl, on the other hand, is less likely the cause. She has been on a stable dose since home and was previously tolerating it well. Stopping the fentanyl will likely increase delirium, dyspnea and withdrawal symptoms. Increasing the fentanyl, similarly, is unlikely to address the agitation – unless the patient has been responding to breakthrough oxycodone. d) As above e) Family members should be comforted, but not that it is an inevitable part of dying. f) Restraints should be avoided. References: • http://www.geripal.org/2010/06/ny-times-article-on-delirium.html • http://www.eperc.mcw.edu/EPERC/FastFactsIndex/ff_001.htm
  • 21. From Blogs to Boards: AAHPM 2012 Pre-­‐Course from the contributors of Pallimed & Geripal Notes Question HPM16 Page 21 of 48 Mr. J is 58 year old diagnosed with ALS 6 months ago. He is referred to your clinic by his primary care doctor to help discuss options to treat a progressive weight loss. He currently lives alone in an apartment, is independent of ADLs although he has been having difficulty feeding himself due to proximal arm weakness. He complains that he occasionally bursts out crying or laughing, but denies feeling depressed. His forced vital capacity (FVC) has remained at 70% for the last 3 months. The best next step to help treat his progressive weight loss? a) Riluzole b) PEG Placement c) Mobile arm supports and modified cutlery d) Non Invasive Positive Pressure Ventilation (NIPPV) Discussion: (c) is the correct answer a) Riluzole is the only available disease-modifying therapy for ALS. Based on clinical trials, riluzole likely prolongs median survival in patients with ALS by 2-3 months compared to patients taking placebo.(1) It does little to improve functional outcomes or bulbar symptoms. There is no evidence to suggest that it is beneficial for weight loss. b) PEG placement should be discussed with any individual diagnosed with ALS, although attempt to reverse other common reasons for weight loss is warranted before PEG placement. Ideally, PEG tubes should be placed before FVC falls below 50%. While PEG may be indicated for this patient in the future, currently C is the best option. c) Individuals living with ALS may have difficulty with the mechanics of both cooking and putting food from the plate to the mouth. This may often contribute to weight loss, especially for those individuals living alone. Occupational therapy may help maintain adequate nutrition by supplying devices such as mobile arm supports and modified cutlery. d) NIPPV confers a survival benefit and improves quality of life in patients with normal or moderately impaired bulbar function, although it does not improve weight References: • Miller RG, Mitchell JD, Lyon M, Moore DH. Riluzole for amyotrophic lateral sclerosis (ALS)/motor neuron disease (MND). Cochrane Database Syst Rev. 2007 Jan 24;(1):CD001447. • Mitsumoto H, Rabkin, JG. Palliative Care for Patients With Amyotrophic Lateral Sclerosis “Prepare for the Worst and Hope for the Best”. JAMA. 2007;298(2):207-216
  • 22. From Blogs to Boards: AAHPM 2012 Pre-­‐Course from the contributors of Pallimed & Geripal Notes Question HPM17 Page 22 of 48 Mr G is a 74-year-old nursing home resident with coronary artery disease and end-stage renal failure (eGFR of 12). He is considering starting treatment with dialysis but would like to know more about what life will be like after starting dialysis. What would be the most accurate statement in regards to his prognosis a) His functional status is likely to improve with renal replacement therapy b) His functional status is likely to be maintained at his pre-dialysis level c) He is unlikely to have significant symptom burden if he elects not to initiate dialysis d) The majority of nursing home residents die within one year of starting dialysis Discussion: (d) is the correct answer a) Based on a NEJM paper (1) that linked dialysis registry data to activities of daily living measures reported by nursing homes in 3,702 patients, patients similar to Mr. G did poorly. Within 3 months after the start of dialysis, 61% of the nursing home residents had died or had a decrease in functional status as compared with their functional status before dialysis. Only 39% had the same functional status that they had before dialysis. By 12 months, almost all (87%) nursing home residents had died or had a decrease in functional status after starting dialysis. b) See a above c) The last month of life for individuals who elect not to undergo renal replacement therapy is associated with relatively high symptom burden, similar to that of advanced cancer.(2) Common symptoms include lack of energy, itching, feeling drowsy, shortness of breath, difficulty concentrating, pain, lack of appetite, and swelling of arms/legs. Therefore, clinicians should attend to these symptoms as aggressively as they do for patients with advanced cancer. We don’t know though whether symptom burden is improve or worsened for elderly patients with multiple chronic conditions who do elect for renal replacement therapy. d) In the NEJM cited above, 58% of these nursing home residents had died 1 year after initiating dialysis References: • http://www.geripal.org/2009/10/how-should-we-counsel-frail-nursing.html • http://www.geripal.org/2010/09/dying-without-dialysis.html • Kurella Tamura M, Covinsky KE, Chertow GM, Yaffe K, Landefeld CS, McCulloch CE. Functional status of elderly adults before and after initiation of dialysis. N Engl J Med 2009;361:1539-1547 • Murtagh FE, et al. Symptoms in the Month Before Death for Stage 5 Chronic Kidney Disease Patients Managed Without Dialysis. J Pain Symptom Manage. 2010 Sep;40(3):342-52.
  • 23. From Blogs to Boards: AAHPM 2012 Pre-­‐Course from the contributors of Pallimed & Geripal Notes Question HPM18 Page 23 of 48 George Condi is a 68 y/o male is admitted to ICU for respiratory crisis and found to have renal cell carcinoma with a 13 cm mass in the R upper abdomen. He has severe pain, and dyspnea with large R sided pleural effusion. With drainage of effusion his dyspnea is improved; a tunneled pleural catheter is placed, and he is discharged to home hospice with a PPS of 50. The next day his wife calls saying she can’t manage the catheter and she is in tears because his pain is 6/10 and he is more short of breath. “You promised me it wouldn’t be like this!” She wants to take him to the emergency room for IV furosemide and a pulmonologist visit. The best approach is to: a) Arrange for a hospice nurse to meet the patient in the emergency room to disenroll him from hospice b) Set up in home continuous care to manage his catheter c) Immediately prepare a respite stay d) Admit the patient to a qualified skilled nursing facility for General Inpatient stay for pain control Discussion: (d) is best a) The family needs are symptom management and training about catheter care. Both are best provided in a controlled environment, not in the emergency room. This case represents a failure in transitions of care, and disenrollment from hospice will only lead to one more transition b) Continuous care could be a reasonable choice if the issue were only the catheter. The requirement for skilled care is the same as for gip and it is a good option for a patient who really does not want to be in a facility but in this case the patient’s intensive symptom need may need 24 hour nursing care and continuous care is not entirely provided by nursing level care c) Respite is for the benefit of the family and is generally a planned event, it is also appropriate to use in cases of caregiver breakdown when there is not a skilled care need requirement for the patient d) In this case, the patient has severe dyspnea and pain and is requiring both catheter drainage of an effusion and rapid titration of opioids to control his symptoms in a manner that his care givers are not capable of providing currently. GIP is to provide skilled care for the patient that cannot be provided in the home. Documentation for GIP based on pain must include: • Frequent evaluation • Frequent medication adjustment • Aggressive interventions to control the pain References: • http://www.aahpm.org/apps/blog/?p=1133
  • 24. From Blogs to Boards: AAHPM 2012 Pre-­‐Course from the contributors of Pallimed & Geripal Notes Question HPM19 Page 24 of 48 George is admitted to GIP status in a skilled nursing facility with 24 hour RN availability. He has had a marked decline since he was seen 2 days ago. The hospice nurse is asking whether the plan should be to send him back home after the symptoms are controlled. The social worker doesn’t want to bring that up because it might upset the wife and because it might give George false hope. The entire Interprofessional group thinks he might die in the next week or two You reply that: a) Since he’ll likely die in 7-10 days, it will be fine to continue on General Inpatient Status for imminently dying criteria so discharge discussions don’t need to be raised b) Due to the wife’s burden of caregiver distress, the patient will be maintained on General Inpatient Status for caregiver breakdown so discharge discussions don’t need to be raised c) Once admitted to General Inpatient Status, one of the goals must be transition to a lower level of care d) Since General Inpatient status should only last 7 days, discharge discussions will start after the first 3 days to let the family have some relief. Discussion: Correct answer is (c) a) There is no GIP status for ‘imminently dying’. There must be some symptom that requires management b) CMS has clarified that GIP should only be used based on the patient condition and should not be used due to caregiver “breakdown”. (CMS Quarterly Provider Update April 2007, http://www.cms.hhs.gov/quarterlyproviderupdates/downloads/cms1539p.pdf) c) Direct wording from quarterly. The goal may not be achievable, but needs to be a part of planning and discussion. d) There is no specified time limit to GIP status, although some fiscal intermediaries do appear to increase audits after the first 7 days References: • http://digital.ipcprintservices.com/publication/?i=93241 (login required using aahpm membership id) • Medicare Benefit Policy Manual Chapter 9 - Coverage of Hospice Services Under Hospital Insurance 40.1.5 - Short-Term Inpatient Care pp15 and 16/32 http://www.cms.gov/manuals/downloads/bp102c09.pdf
  • 25. From Blogs to Boards: AAHPM 2012 Pre-­‐Course from the contributors of Pallimed & Geripal Notes Question HPM20 Page 25 of 48 Mrs. Tagliatelli is a 76 year old Italian immigrant and widow who has not missed a day of mass in her adult life until this past month. She comes to see her primary care physician in clinic because she missed mass, asking whether she should get hospice. She has heart failure, mild hypertension, and sleep apnea. She has noted that over the last month, her legs are more swollen and she is having increased difficulty walking to church and the grocery store. She still keeps an impeccable home, managing her housecleaning herself, but now is sitting down for a longer period of time after carrying the vacuum up and downstairs. She is also able to maintain her daily rituals of reading the NYTimes Health and Travel sections, cooking three small meals each day. She no longer wishes to return to hospital, and has not been admitted since her myocardial infarction 5 years ago, which preceded her diagnosis of heart failure. At that time, she had a successful resuscitation and wishes to remain full code. She uses CPAP at night for her sleep apnea, but otherwise does not require oxygen. She also tells you that because she lives alone, she keeps a gun in her home for self-protection. Her home medications include: Furosemide 10mg BID, Atenolol 50mg daily, lisinopril 10mg daily, simvastatin 5mg daily, aspirin 81mg daily. She also has nitroglycerine 0.4mg sl prn (which she has not used since her MI), and acetaminophen 325mg which she takes “once in a while for an ache.” Why would this patient not be admitted to hospice? a) She is full code. b) She lives alone. c) She has greater than a six-month prognosis. d) She is not homebound. e) She has firearms in the home. Discussion: answer is (c) Take home points: • Prognostication for heart failure is one of the more scrutinized and difficult under the hospice guidelines. Her current NYHA class is 2, due to increased symptoms with activity. In addition, her medical management can still be adjusted, likely with good response. General guidelines for heart failure include hospitalization within the last 6- months to a year, dyspnea with minimal exertion or at rest. Since she still is independent in all ADLs including thorough house-cleaning, without becoming dyspneic, it is not reasonable to say she has a <6mo prognosis. • While to qualify for visiting nursing services, a patient must be homebound, no such requirement exists for hospice. • Additionally, patients may choose to be full code on hospice. Medicare does not require a 24 hour caregiver to be present in order for the patient to receive hospice services. While firearms are concerning for patient and staff safety, they do not prevent hospice admission. Some hospices have adopted a stance to request patients to have their firearms in locked safe or gun closet. References: See Q21
  • 26. From Blogs to Boards: AAHPM 2012 Pre-­‐Course from the contributors of Pallimed & Geripal Notes Question HPM21 Page 26 of 48 A couple of years and hospitalizations later, Mrs. Tagliatelli was admitted to hospice. At the time of admission to hospice, she was breathless with minimal exertion. Neighbors and members of her church visited her often offering her food, company, and rides to church. She required oxygen all the time. Even with this, at the time of admission to hospice, she experienced constant dyspnea. Her cardiac medications were continued, morphine ER and IR were added for her dyspnea. After six months on hospice, she is now well palliated, especially since she has been able to have her medications as prescribed and no longer spaces out her medications in order to make them last. However, she continues to require help from her friends and neighbors, and oxygen with minimal activity. She fell once and required a trip to the emergency department. You go to see her for recertification visit. What do you write in your recertification note? a) She meets criteria for recertification because her prognosis remains 6-months or less. b) She does not meet criteria for recertification because she has not shown decline in her condition. c) She does not meet criteria for recertification because her last hospitalization was unrelated to her hospice diagnosis Discussion (a) is the correct answer Take home points: • Some intermediaries recommend the demonstration of decline in clinical condition for a patient to be recertified, and while this is helpful is the recertification process, it is not a CMS requirement. According to Medicare guidelines, the only requirement for hospice is that a patient’s prognosis is 6 months or less. • Documentation of a hospitalization can also help qualify a patient for hospice. The cause of hospitalization does not need to be related to the hospice diagnosis. References: • https://www.cms.gov/Hospice/Downloads/HospiceFace-to-FaceGuidance.pdf • http://www.geripal.org/2011/02/hospice-face-to-face-ftf-encounters-for.html
  • 27. From Blogs to Boards: AAHPM 2012 Pre-­‐Course from the contributors of Pallimed & Geripal Notes Question HPM22a, 22b Page 27 of 48 A young man was recently in a motor vehicle collision where he suffered a massive head injury and multi-trauma. He was resuscitated and survived in the ICU with a ventilator, continuous hemodialysis, and multiple pressors for the past 2 days, but now is declining, and he is not expected to survive this hospitalization. You receive a palliative care consult to help with the ventilator withdrawal. You head down to the unit and the nurse comes to you and says “I am not sure you should talk with the family – the organ procurement agency has just visited to discuss organ donation after cardiac death, and the family want to donate his organs – his liver and lungs may be transplantable.” What is the best next step? a) Thank the nurse, and back out of the consult b) Talk with the family about the patient, their grief, and counsel them about comfort care after cessation of life-support. c) Ask the attending physician of record who is going to manage the patient’s comfort care after cessation of life-support. d) Work with the family to help them realize this will only prolong the patient’s suffering. Part B A day later, the patient’s HCV test comes back positive and he is no longer a viable DCD candidate. The ICU attending asks you to ‘take care of the treatment withdrawal’. The family is very disappointed, and indicates their only goal at this point is for a comfortable death, without ‘prolonging this any longer.’ His only symptom-directed med is intermittent fentanyl bolus (700mcg the last 24h). He is unresponsive on the vent, without any spontaneous movement. The best next step is to: a) Recommend rapidly stopping all life support including CRRT, ventilator, and pressors over the next hour or so, and starting a fentanyl and lorazepam infusion to keep the patient sedated. b) Recommend staggering withdrawal of life support over a couple days including stopping CRRT and pressors now in the hopes that the patient dies on the ventilator. c) Discuss with the family different approaches to life-support withdrawal. d) Switch the patient from fentanyl to morphine boluses as you extubate him, as morphine is more effective for air-hunger. Discussion: Correct answers are Part A(b) Part B (c) Part A Take Home Points • Donation after cardiac death is an important, and growing, public good, as it expands the pool of potential organ donors: it saves lives. Palliative consultants should support DCD programs, and palliative consultation alongside DCD is possible and in some institutions the standard of care. For most families, the real concern is not in prolonging suffering but the disappointment which can occur if the dying patient becomes ineligible to donate organs. DCD practices and the role palliative care consultants can play in them are spelled out nicely in this Fast Fact: http://www.eperc.mcw.edu/EPERC/FastFactsIndex/ff_242/htm. • While C is an important question as the palliative consultant may be asked to manage the patient’s comfort care after extubation in the window during which if the patient dies he will be able to donate his organs, that is a secondary concern right now to doing the good work of palliative care – meeting a patient and family and helping to meet their emotional and informational needs. Part B Take Home Points • A study in 2007 showed that family satisfaction was greater with a stuttered/prolonged
  • 28. From Blogs to Boards: AAHPM 2012 Pre-­‐Course from the contributors of Pallimed & Geripal Page 28 of 48 approach to discontinuing therapies if the patient had been in the ICU for more than 4 days, but when the patient was in for 3days or less, a stuttered approach lowered satisfaction. Even with those clearly stated goals, careful discussions with the family are best about the options for cessation of life-prolonging treatments as families may have anticipated concerns about stopping certain treatments (fluids, tube feeds in particular). Given the modest evidence of improved satisfaction with staggered withdrawal, dicussing with family is the best policy. • Answer a) may end up being the best approach; however simply starting a fentanyl infusion is secondary to providing/ensuring adequate rapid, boluses of symptom medications are available around the time of extubation. • No opioid has been established as being superior over another for air hunger. In addition, the patient is in renal failure and morphine may accumulate whereas fentanyl will not; on the other hand, he is likely to live just a short amount of time making that a moot point generally. Morphine is more economical and familiar; on the other hand one knows how much fentanyl this patient has needed as a starting point. On top of this, this comatose patient probably cannot experience air hunger; the indication for opioids really is to palliate labored respirations. All this is to say d is both unnecessary but not contraindicated, but not for the reason stated. References: • http://www.pallimed.org/2008/10/stuttered-treatment-withdrawal-in-icu.html • http://cases.pallimed.org/2009/01/coordination-of-care-for-people-at-end.html • http://www.pallimed.org/2006/03/terminal-patients-in-icu-and-organ.html • Gerstel E, Engelberg RA, Koepsell T, & Curtis JR. (2008) Duration of withdrawal of life support in the intensive care unit and association with family satisfaction. American journal of respiratory and critical care medicine, 178(8), 798-804. PMID: 18703787 • Revelly JP, Imperatori L, Maravic P, Schaller MD, & Chioléro R. (2006) Are terminally ill patients dying in the ICU suitable for non-heart beating organ donation?. Intensive care medicine, 32(5), 708-12. PMID: 16534569 • Lynn, J. (2001-2-21) Serving Patients Who May Die Soon and Their Families: The Role of Hospice and Other Services. JAMA: The Journal of the American Medical Association, 285(7), 925-932. DOI: 10.1001/jama.285.7.925 • Vent withdrawal FF: http://www.eperc.mcw.edu/EPERC/FastFactsIndex/ff_033.htm • http://www.eperc.mcw.edu/EPERC/FastFactsIndex/ff_034.htm • DCD FF: http://www.eperc.mcw.edu/EPERC/FastFactsIndex/ff_242/htm • Anoxic Brain Injury FF: http://www.eperc.mcw.edu/EPERC/FastFactsIndex/ff_234.htm
  • 29. From Blogs to Boards: AAHPM 2012 Pre-­‐Course from the contributors of Pallimed & Geripal Notes Question HPM23 Page 29 of 48 A 47 year old woman with a severe, idiopathic, dilated cardiomyopathy is receiving hospice care at home. She is ineligible for cardiac transplantation or a ventricular assist device. She has mild resting dyspnea but becomes severely dyspneic after just a few steps of ambulation. Her nurse measures her resting and ambulatory oxygen saturation while breathing ambient air: it is 96 and 92%, respectively. The patient is taking digoxin, bumetamide, hydralazine, isosorbide dinitrate, albuterol MDI, warfarin, senna, and clonzepam. The patient requests home oxygen therapy to help alleviate her breathlessness. The best response is: a) Order home oxygen therapy for the patient b) Initiate lorazepam prn for dyspnea c) Recommend use of a hand-held fan and prn morphine for her dyspnea d) Request that the patient see her cardiologist for further optimization of her heart failure meds Discussion: Correct answer is (c) a) Home oxygen therapy is not recommended as first-line treatment for dyspnea in non-hypoxic patients. It has been shown to be equivalent to ‘sham’ delivery of ambient air via nasal cannula. While there is a role for it even in normoxic patients (it ‘works’, just no better than ambient air), it is not first-line. b) She is already on a benzodiazepine, and benzodiazepines are generally considered 2nd line agents to opioids c) Hand-held fans have been shown to improve dyspnea, and there is professional consensus that opioids are first-line agents for the symptomatic relief of refractory dyspnea that is not responding to treatment of the underlying cause. d) While there is a role for this strategy, the patient is already on multiple heart failure medications which clearly are not sufficient to palliative her dyspnea, and so c is the best answer. References: • http://www.pallimed.org/2010/09/rct-of-oxygen-vs-room-air-delivered-by.html • Viola R et al. The management of dyspnea in cancer patients: a systematic review. Supp Care Cancer. 2008; 16:329-337. • Galbraith S. Does the use of a handheld fan improve chronic dyspnea? A randomized, controlled, crossover trial. J Pain Symptom Manage. 2010 May;39(5):831-8.
  • 30. From Blogs to Boards: AAHPM 2012 Pre-­‐Course from the contributors of Pallimed & Geripal Page 30 of 48 Question HPM24 Mr. L is a 52-year-old homeless man. One week ago, he was admitted to the ICU with respiratory distress and was intubated. A chest CT scan revealed a large necrotic mass filling the right hemithorax, obliterating the right and narrowing the left mainstem bronchi. Sputum cytology confirmed a diagnosis of non-small cell lung cancer. Oncology states that there is no role for chemotherapy or radiation unless he could be weaned off the ventilator, which was considered doubtful in the setting of his airway obstruction. Mr. L is unable to participate in medical decision-making. The patient’s mother, who is the authorized decision maker, meets with the palliative care team to discuss prognosis and treatment options, including withdrawal of life-sustaining treatments. The mother is adamant that all life-sustaining measures be continued despite a previous discussion that Mr. L’s disease severity will prevent him from ever leaving the ICU, let alone the hospital. Mr. L’s mother expresses hope that, despite the physician’s prediction, a miracle will occur that will allow her son to leave the hospital. The next best step is to: a) Schedule another family meeting to reiterate the prognosis of his current condition and the likelihood of recovery b) Involve an ethics committee as the mother’s belief in a miracle is far from a societal norm c) Tell the mother that hope for a miracle is unreasonable, but that she could still hope that her son is comfortable d) Ask the mother about her spiritual beliefs and how it influences her decision Discussion: Correct answer is (d) a) Although it would be reasonable to again discuss prognosis, the mother’s hope in a miracle can be considered at least an acknowledgement that she heard the original prognostic information. Furthermore, spiritual beliefs, including that of a belief in miracles, may trump a physician’s opinion of prognosis despite adequate communication of prognosis.(1) One study of surrogates of incapacitated critically ill patients at high risk for death found that only 2% based their views of prognosis solely on the physician’s prognostic estimate.(2) Rather, these surrogates used a combination of sources including knowledge of the patient’s intrinsic qualities and will to live; their observations of the patient; their own observations and beliefs in the power of their support and presence, and optimism, intuition, and faith (For 20% of surrogates, a faith in God overrode any other source of prognostic information).(1) While scheduling another meeting may be appropriate, the ‘problem’ in this scenario is not lack of DATA, and D is the better answer. b) According to a 2007 survey performed by the Pew Forum on Religion and Public Life, the majority of Americans believe in miracles with little difference based on the respondent’s age. In another survey, most public respondents (57.4%) believed that divine intervention from God could save a person even if the physician told them ‘‘futility had been reached.’’(3) Significant differences in belief in miracles were noted in this study between health care professionals and the general public. The majority of public respondents (61.3%) believed that a person in a persistent vegetative state could be saved by a miracle, although only a minority of trauma professionals had the same belief. c) Without knowing more about the mothers spiritual convictions and having a good understanding of what a “miracles means for her”, and reframing of her hope would be premature and could be perceived as condescending.(1) d) Most individuals would like physicians to ask about their spiritual/religious beliefs.(1) In addition, patients who report that their spiritual needs are supported by the medical team are more likely to receive hospice care than those who report their spiritual needs were unsupported.(2) References: • http://www.geripal.org/2011/06/lessons-i-learned-by-examining-miracles.html • Widera EW, Rosenfeld KE, Fromme EK, Sulmasy DP, Arnold RM. Approaching patients and family members who hope for a miracle. J Pain Symptom Manage. 2011 Jul; 42(1):119-25. • Boyd EA, Lo B, Evans LR, et al. ‘‘It’s not just what the doctor tells me:’’ factors that influence surrogate decision-makers’ perceptions of prognosis. Crit Care Med 2010;38:1270e1275. • Jacobs LM, Burns K, Bennett Jacobs B. Trauma death: views of the public and trauma professionals on death and dying from injuries. Arch Surg 2008; 143:730e735. • Balboni TA, Paulk ME, Balboni MJ, et al. Provision of spiritual care to patients with advanced cancer: associations with medical care and quality of life near death. J Clin Oncol 2010;28:445e452
  • 31. From Blogs to Boards: AAHPM 2012 Pre-­‐Course from the contributors of Pallimed & Geripal Notes Question HPM25 Page 31 of 48 Omar Johnson is a 64 year old man with cryptogenic cirrhosis in multiorgan system failure in your hospital’s ICU. He is ventilated, unresponsive, and on vasopressors. You and the ICU team agree his chances for surviving this hospitalization are minimal. He has no advance directive. You participate in an ICU family care conference with his wife (his legal decision maker based on state law), 2 sisters, and 3 adult sons. They are told he is dying with minimal chance of survival. His sons say they do not think the patient would want to die ‘like this – on machines,’ and describe several conversations with the patient to support that preference. His wife seems to agree with that, but also says, “I can’t give up on him. I can’t have that on my shoulders – I’ll always wonder if I did the right thing.” The best, next response would be: a) Request ethics consultation b) Along with the ICU physician, suggest to the family that you make the decision on behalf of the patient yourselves, to transition the patient to comfort-care. c) Ask the family to focus on what the patient himself would prefer in these circumstances. d) Express to the family acknowledgment of the emotional difficulty of this, and recommend another meeting the next day. Discussion: Correct answer is (b) a) Be wary of it as a board question answer. Ethics consultations have been shown to help with conflict although it wouldn’t be the “best, next response” in this situation. b) Making recommendations for what you believe to be the best plan of care is critically important in these situations. Surrogate decision making is uniquely traumatizing to family members (beyond routine bereavement); when a patient’s preferences are clear, physicians should clearly articulate an appropriate plan of care and not force family members into feeling they are responsible for a patient’s death. c) Generally a good idea – but they already have acknowledged those ‘facts’; the issue here is more comfort with decision-making roles and guilt. d) A 2nd best option to 2 References: • http://www.pallimed.org/2011/03/trauma-of-surrogacy.html
  • 32. From Blogs to Boards: AAHPM 2012 Pre-­‐Course from the contributors of Pallimed & Geripal Notes Question HPM26 Mrs. Hassad is a 83 year old retired professor from who is being evaluated for a hospice admission. Her 4 sons live in adjacent homes with their families. She has metastatic breast cancer with bone, liver, and brain. Because of her underlying renal failure and moderate heart failure, she will not be receiving chemotherapy and her physician had arranged home hospice services now that she has completed palliative radiation. She is alert, oriented. The hospice intake nurse calls you because the family and patient state that she does not want to know anything about his diagnosis or severity of illness. Mrs. Hassad’s son tells the nurse not to speak with the patient about her prognosis, her illness, or about code status. Instead, she asks that you speak with her son about these matters. You are at the home with the nurse because she does not know how to get her to sign the paperwork to enroll in hospice. What do you do after confirming with Mrs. Hassad that she does not want to be involved in signing papers or knowing details of his medical condition, and would rather that you speak with her son? a) Explain to the son that you must gain consent from Mrs. Hassad in order to enroll her in hospice in respect of Page 32 of 48 the principle of autonomy. b) Invoke the health care proxy and have Mrs. Hassad’s son sign the paperwork to enroll in hospice. c) Have the son sign the paperwork for hospice since Mrs. Hassad made the autonomous decision to defer decisions to her son. d) Refuse hospice enrollment for the patient since she is unwilling to accept to address her diagnosis and prognosis. e) Clarify to the patient that it is her responsibility to make the decision, based on autonomy, and to avoid trauma of surrogacy in her son. f) Teach the nurse that she should not have questioned the son’s request because that was disrespectful to their culture. Discussion: (c) is the correct answer • Patients can make the autonomous decision to know or not to know information. As outlined in the SPIKES protocol for giving bad news, the Invitation is to ask how much a patient wishes to know. Should patients choose not to be informed they should know that then consents to procedures and medical care must also be deferred to the person they request disclosure to. • Healthcare proxy is invoked only when patients lack the capacity to make medical decisions. Here Mrs. Hassad has capacity but chooses to have her son make decisions on her behalf. • Hospice enrollment does not require a patient’s acceptance of his or her disease and prognosis. • Research shows that most people, regardless of culture or country of origin, wish to have medical information disclosed to them. However, as age and illness advance, patients are more inclined to request less disclosure. One should not assume desire or lack of desire to be involved in medical decision-making and disclosure of information based on culture, religion or country of origin. References: § http://www.pallimed.org/2011/03/trauma-of-surrogacy.html § Oncotalk. (n.d.). Giving Bad News., http://depts.washington.edu/oncotalk/learn/modules/Modules_02.pdf § Wendler, D., & Rid, A. (2011). Systematic review: the effect on surrogates of making treatment decisions for others. Annals of internal medicine, 154(5), 336-46. Retrieved from http://www.annals.org/cgi/content/abstract/154/5/336 § Elkin, E. B., Kim, S. H. M., Casper, E. S., Kissane, D. W., & Schrag, D. (2007). Desire for information and involvement in treatment decisions: elderly cancer patients’ preferences and their physicians' perceptions. Journal of clinical oncology : official journal of the American Society of Clinical Oncology, 25(33), 5275-80. § Baile, W. F. (2000). SPIKES--A Six-Step Protocol for Delivering Bad News: Application to the Patient with Cancer. The Oncologist, 5(4), 302-311. § Elkin, E. B., Kim, S. H. M., Casper, E. S., Kissane, D. W., & Schrag, D. (2007). Desire for information and involvement in treatment decisions: elderly cancer patients’ preferences and their physicians' perceptions. Journal of clinical oncology : official journal of the American Society of Clinical Oncology, 25(33), 5275-80. § Asghari, F., Mirzazadeh, A., & Fotouhi, A. (2008). Patients’ preferences for receiving clinical information and participating in decision-making in Iran. Journal of medical ethics, 34(5), 348-52. § Bushnaq, M. (2008). Palliative care in Jordan: culturally sensitive practice. Journal of palliative medicine, 11(10), 1292-3. Mary Ann Liebert, Inc. 140 Huguenot Street, 3rd Floor New Rochelle, NY 10801-5215 USA.
  • 33. From Blogs to Boards: AAHPM 2012 Pre-­‐Course from the contributors of Pallimed & Geripal Notes Question HPM27 Page 33 of 48 Dr. L is a 44 year old palliative care fellow about to complete two months of a busy inpatient consult rotation. You notice that over the last week she has become detached and disengaged when talking with patients and their family members. The fellow acknowledges feeling tired and drained most of the time, as well as having difficulty falling asleep. She also confides in you a personal sense of failure and self-doubt. The most appropriate interventions at this time is a) Recommend she see her primary doctor to discuss SSRI therapy b) Recommend she try bright light therapy c) Refer for a transient mirrectomy d) Recommend an educational program in mindful communication Discussion: Correct answer is (d) a) The fellow does not meet DSM-IV criteria for depression, although she does have some suggestive symptoms. Further exploration would be a correct answer, but starting an SSRI would not. b) There is no evidence that bright light therapy is helpful for symptoms of burnout. There is some evidence of a small benefit for depressive symptoms though. c) Transient mirrectomy is a fictional treatment described by Brad Stuart in an April fools day GeriPal post. It reportedly is a non-invasive method of numbing brain centers that may induce clinicians to identify with pain and suffering to a disabling degree. Sure sounds nice. d) The fellow has symptoms suggestive of burnout. Burnout encompasses 3 domains: feelings of emotional exhaustion, cynicism or depersonalization, and a low sense of personal accomplishment. The criterion standard for measuring burnout is the Maslach Burnout Inventory (MBI). There is scant high-quality evidence on the approach to treating burnout, however one study published in JAMA suggested that participation in a mindful communication program was associated with improvements in well-being, including burnout. References: • http://www.geripal.org/2011/04/doctor-develops-cure-for-burnout.html • Krasner MS et al. Among Primary Care Physicians Communication With Burnout, Empathy, and Attitudes Association of an Educational Program in Mindful. JAMA. 2009;302(12):1284-1293 • http://blog.vcu.edu/dpgray/JAMA%20self-care%20end%20of%20life.pdf • Kearney MK et al. Self-care of Physicians Care for Patients at the End of Life: “Being Connected...A Key to My Survival.” JAMA. 2009;301(11):1155-1164
  • 34. From Blogs to Boards: AAHPM 2012 Pre-­‐Course from the contributors of Pallimed & Geripal Notes Question HPM28 Page 34 of 48 Your palliative care clinic team meets Nancy Bush a 46 year-old with newly diagnosed triple-negative metastatic breast cancer. She has 7 and 11 year old children. The children know Nancy has been ‘to the doctor’ a lot lately but nothing else. She is thinking about talking with the children and letting them know her diagnosis, but her mother thinks that telling them now will be too hard on them. You advise: a) It is best to wait until Nancy’s disease is obvious to the children so their interactions with their mother will not change. b) Telling the children now will make them too anxious. c) She should tell the older child, but the younger child is not at an appropriate development age that he will benefit from hearing his mother has cancer. d) Telling the children of the disease may make them less anxious Discussion: Correct answer is (d) For the purposes of the boards – your default position should be one of truthful disclosure to children of all ages. The highest quality longitudinal study of bereaved children showed: • The early loss of a parent was associated with poverty, - contributing factors may include the loss of income, as well as the burden of medical expenses • the increase in separation anxiety symptoms begins prior to death: this speaks to the need for preventive interventions when a family death is impending • Several studies have shown a higher incidence of substance abuse in bereaved children, up to at least 21 months after the death • Bereavement pattern associated with early loss of a parent was associated with poverty, substance abuse problems, and greater functional impairments. References • http://cases.pallimed.org/2009/05/what-do-i-say-to-my-kids.html • Rosenheim, E., Reicher, R. (1985). Informing children about a parent’s terminal illness. J Child Psychol Psychiatry Allied Disc. 26:995-998. • Siegel, K., Raveis, V., Karus, D. (1996). Pattern of communication with children when a parent has cancer. In L. Baider & L. Cooper (Eds) Cancer and the family, pp 109- 128. John Wiley and Sons: New York. • Psychiatric symptoms in bereaved versus nonbereaved youth and young adults: a longitudinal epidemiological study. Kaplow JB. Saunders J. Angold A. Costello EJ. • Journal of the American Academy of Child & Adolescent Psychiatry. 49(11):1145-54, 2010 Nov. • Sanchez L, Fristad M, Weller RA, Weller EB, Moye J. Anxiety inacutely bereaved prepubertal children. Ann Clin Psychiatry.1994;6:39-43. • Swadi H. A longitudinal perspective on adolescent substance abuse. Eur Child Adolesc Psychiatry. 1992;1:156-170.
  • 35. From Blogs to Boards: AAHPM 2012 Pre-­‐Course from the contributors of Pallimed & Geripal Notes Question HPM29 Page 35 of 48 You receive a call from the hospice nurse about a new hospice patient, Mrs. Gardner, who had a large ischemic MCA stroke 4 months ago. She has not been able to eat, is unable to turn herself, and has developed a large stage IV decubiti on her low back. The wound measures 10cm x 8cm and 1.2cm deep. It has some limited undermining and no tunneling. At the wound bed, the spine is visible. The bed of the wound reveals malodorous, necrotic purplish muscle and tissue with extensive serosanguinous drainage. The surrounding skin is intact. Mr Gardner covers her wound with a cream but notes ‘It just keeps getting deeper.” The patient is turned q2 hours. The goal of care is to keep her comfortable and at home – a promise he made to her. The hospice nurse asks you for orders to help manage the wound. She will order an air-mattress. After washing the bed of the wound with normal saline, applying a thin layer of metronidazole gel to the base of the wound, what do you recommend for a wound care dressing? a) Pack wound with wet-to-dry dressing and cover with ABD pad every 3 days. b) Pack wound with calcium alginate wafer and rope, cover with ABD pad every 3 days. c) Pack wound with hydrocolloid dressing and cover with ABD pad every 3 days Discussion: (b) is correct – non-occlusive, good for wet wounds References: • May macerate surrounding skin Foam Dressing ++++ • Can be used in infected wounds Alginate dressing +++ Hydrogel ++ • occlusive, should not use with venous/vascular compromise Hydrocolloid + Transparent Lilm • Can cause debridement of healing tissue Gauze (wet to dry) Non-­‐ adherent dressings + = degree of absorption • Ferris, F., & Pirrello, R. (n.d.). Palliative Wound Care. Retrieved February 25, 2012, from google.docs presentation • McDonald, A., & Lesage, P. (2006). Palliative management of pressure ulcers and malignant wounds in patients with advanced illness. Journal of palliative medicine, 9(2), 285-95. Mary Ann Liebert, Inc. 2 Madison Avenue Larchmont, NY 10538 USA.
  • 36. From Blogs to Boards: AAHPM 2012 Pre-­‐Course from the contributors of Pallimed & Geripal Notes Question HPM30 Page 36 of 48 A 45 year old man with HIV-AIDS comes to your clinic for follow-up for HIV-related neuropathy pain. He has long declined any antiretroviral therapy, and has consistently stated he wants supportive-only care focused on maintaining his quality of life. He has a CD4 count of 90 cells/mm3. 1 year ago it was 100. He reports worsening pain control which he relates to inability to swallow his morphine ER tabs (100 mg tid) much of the time. He reports mid-throat pain, and frequently chokes on the pills, ‘gags’ them back up. Examination reveals a thin man. Mouth demonstrates scattered white plaques on the palate which reveal a red base when scraped away. Best next step is to: a) Prescribe Nystatin ‘swish & swallow’; change morphine to 30mg elixir q4h scheduled. b) Prescribe fluconazole; change MorphineER pills to to MorphineER ‘granules’ in pudding (such as ‘Kadian’ or ‘Avinza’ morphine formulations). c) Prescribe fluconazole, change his morphine to methadone elixir, and recommend hospice care given his goals of care and prognosis. d) Prescribe Nystatin ‘swish & swallow’; change his morphineER to a fentanyl patch. Discussion: Correct answer is (b) This man has HIV, AIDS, thrush, and based on the history of dysphagia and odynophagia, esophageal candidiasis as well. Due to this, systemic antifungal agents are indicated such as fluconazole. Topical agents are ineffective for esophageal candidiasis; for thrush alone they are less effective but still used as first-line agents. All of the strategies to manage his pain while he is having pill dysphagia are within the realm of reason – use of scheduled immediate release morphine, Morphine ER granules which can be given in pudding or down a G-tube (‘Kadian’ or ‘Avinza’), or transdermal fentanyl. Methadone elixir or crushed pills is a possibility, however it’s a more complicated rotation, and interacts with fluconazole, and probably not as elegant as the other solutions. Hospice care is not appropriate for the patient based on prognosis. A CD4 count of 90 which is slowly declining, and no other major life-limiting complication of HIV, indicate his expected prognosis is well over 6 months. Indeed, if he chose to start antiretrovirals it could be decades. Hospice eligibility guidelines, while not very evidence-based, suggest a CD4 count <25cells/mm3 or a persistent viral load >100,000 copies/ml, as well as a serious HIV related comorbidity such as CNS lympthoma, MAC bacteremia untreated or unresponsive to treatment, Progressive multifocal leukoencephalopathy, systemic lympthoma, visceral Kaposi’s sarcoma, renal failure, cryptosporidium infection, or toxoplasmosis unresponsive to therapy. References: • http://www.eperc.mcw.edu/EPERC/FastFactsIndex/ff_213.htm • http://www.pallimed.org/2006/01/prognosis-in-end-stage-hiv-aids.html • http://www.eperc.mcw.edu/EPERC/FastFactsIndex/ff_147.htm